Sei sulla pagina 1di 142

Page 1

CHAPTER 1
General Provisions

Republic v. Heirs of Sin, GR No. 157485, March 16, 2014


REPUBLIC OF THE PHILIPPINES represented by AKLAN NATIONAL COLLEGE OF FISHERIES (ANCF)
and DR. ELENITA R. ANDRADE, in her capacity as ANCF Superintendent, Petitioner,
vs. HEIRS OF MAXIMA LACHICA SIN, namely: SALVACION L. SIN, ROSARIO S. ENRIQUEZ,
FRANCISCO L. SIN, MARIA S. YUCHINTAT, MANUEL L. SIN, JAIME CARDINAL SIN, RAMON L.
SIN, and CEFERINA S. VITA, Respondents.

Facts: Respondents claim that they are the lawful heirs of the late Maxima Lachica Sin who was the owner of a
parcel of land situated at Barangay Tambac, New Washington, Aklan. On August 26, 1991, they respondent
heirs instituted in the RTC of Kalibo, Aklan a complaint against Aklan National College of Fisheries (ANCF)
for recovery of possession, quieting of title, and declaration of ownership with damages claiming that the latter
usurped their rights over the property.
ANCF countered that the subject land was the subject of Proclamation No. 2074 of then President Ferdinand E.
Marcos allocating the area of said property as civil reservation for educational purposes of ANCF. The ANCF
Superintendent furthermore averred that the subject parcel of land is timberland and therefore not susceptible of
private ownership.
The respondents presented evidence that they inherited a bigger parcel of land from their mother who acquired
it by virtue of a deed of sale. That in 1988 a potion thereof was occupied by ANCF and converted into a
fishpond for educational purpose. Respondent heirs asserted that they were previously in possession of the
disputed land in the concept of an owner. To prove possession, respondents presented several tax declarations,
the earliest of which was in the year 1945.
The MCTC, the RTC and the Court of Appeals unanimously held that respondents retain private rights to the
disputed property by virtue of their and their predecessors’ open, continuous, exclusive and notorious
possession amounts to an imperfect title, which should be respected and protected.

Issue: Whether or not the claim of the respondents amounts to judicial confirmation of imperfect title.

Held: No. At the outset, it must be noted that respondents have not filed an application for judicial
confirmation of imperfect title under the Public Land Act or the Property Registration Decree. Section 48(b) of
the Public Land Act and Section 14(1) of the Property Registration Decree provide the requisites for judicial
confirmation of imperfect title: (1) open, continuous, exclusive, and notorious possession and occupation of the
subject land by himself or through his predecessors–in–interest under a bona fide claim of ownership since time
immemorial or from June 12, 1945; and (2) the classification of the land as alienable and disposable land of the
public domain.

Under the Regalian doctrine, which is embodied in our Constitution, all lands of the public domain belong to
the State, which is the source of any asserted right to any ownership of land. All lands not appearing to be
clearly within private ownership are presumed to belong to the State. Accordingly, public lands not shown to
have been reclassified or released as alienable agricultural land or alienated to a private person by the State
remain part of the inalienable public domain. Unless public land is shown to have been reclassified as alienable
or disposable to a private person by the State, it remains part of the inalienable public domain. Property of the
public domain is beyond the commerce of man and not susceptible of private appropriation and acquisitive
prescription. Occupation thereof in the concept of owner no matter how long cannot ripen into ownership and
be registered as a title.
Page 2

The burden of proof in overcoming the presumption of State ownership of the lands of the public domain is on
the person applying for registration (or claiming ownership), who must prove that the land subject of the
application is alienable or disposable. To overcome this presumption, incontrovertible evidence must be
established that the land subject of the application (or claim) is alienable or disposable.
There must be a positive act declaring land of the public domain as alienable and disposable. To prove that the
land subject of an application for registration is alienable, the applicant must establish the existence of a positive
act of the government, such as a presidential proclamation or an executive order; an administrative action;
investigation reports of Bureau of Lands investigators; and a legislative act or a statute. The applicant may also
secure a certification from the government that the land claimed to have been possessed for the required number
of years is alienable and disposable.

In the case at bar, it are therefore the respondents which have the burden to identify a positive act of the
government, such as an official proclamation, declassifying inalienable public land into disposable land for
agricultural or other purposes. Since respondents failed to do so, the alleged possession by them and by their
predecessors–in–interest is inconsequential and could never ripen into ownership. Accordingly, respondents
cannot be considered to have private rights within the purview of Proclamation No. 2074 as to prevent the
application of said proclamation to the subject property.
Republic v. Remnan Enterprises. Inc., GR No. 199310, Feb. 19, 2014
REPUBLIC OF THE PHILIPPINES, Petitioner, V. REMMAN ENTERPRISES, INC., REPRESENTED BY
RONNIE P. INOCENCIO, Respondent.

Facts: On December 3, 2001, Remman Enterprises, Inc. (respondent), filed an application with the RTC for
judicial confirmation of title over two parcels of land, Lot Nos. 3068 and 3077 situated in Barangay Napindan,
Taguig, Metro Manila.

On December 13, 2001, the RTC granted respondent's application for registration. Thereafter, following the
required publication and posting, a scheduled hearing was set. However, on May 30, 2002, only the Laguna
Lake Development Authority (LLDA) appeared as oppositor. Hence, the RTC issued an order of general default
except LLDA, which was given 15 days to submit its comment/opposition to the respondent's application for
registration.
On June 4, 2002, the LLDA filed its Opposition to the respondent's application for registration, asserting that
the lots are not part of the alienable and disposable lands of the public domain. On the other hand, the Republic
of the Philippines (petitioner), on July 16, 2002, likewise filed its Opposition,alleging that the respondent failed
to prove that it and its predecessors-in-interest have been in open, continuous, exclusive, and notorious
possession of the subject parcels of land since June 12, 1945 or earlier.

Respondent's witnesses showed that the respondent and its predecessors-in-interest have been in open,
continuous, exclusive, and notorious possession of the said parcels of land long before June 12, 1945. The
respondent purchased Lot Nos. 3068 and 3077 from Conrado Salvador (Salvador) and Bella Mijares (Mijares),
respectively, in 1989. The subject properties were originally owned and possessed by Veronica Jaime (Jaime),
who cultivated and planted different kinds of crops in the said lots, through her caretaker and hired farmers,
since 1943. Sometime in 1975, Jaime sold the said parcels of land to Salvador and Mijares, who continued to
cultivate the lots until the same were purchased by the respondent in 1989.

The respondent likewise alleged that the subject properties are within the alienable and disposable lands of the
public domain, as evidenced by the certifications issued by the Department of Environment and Natural
Resources (DENR).
Page 3

On the other hand, the LLDA alleged that the respondent's application for registration should be denied since
the subject parcels of land are not part of the alienable and disposable lands of the public domain; it pointed out
that pursuant to Section 41(11) of Republic Act No. 4850(R.A. No. 4850), lands, surrounding the Laguna de
Bay, located at and below the reglementary elevation of 12.50 meters are public lands which form part of the
bed of the said lake. Engr. Magalonga, testifying for the oppositor LLDA, he found out that the elevations of
Lot Nos. 3068 and 3077 are below 12.50 m. That upon actual area verification of the subject properties on
September 25, 2002, Engr. Magalonga confirmed that the elevations of the subject properties range from 11.33
m to 11.77 m.

On rebuttal, the respondent presented Engr. Flotildes, who claimed that, based on the actual topographic survey
of the subject properties he conducted upon the request of the respondent, the elevations of the subject
properties, contrary to LLDA's claim, are above 12.50 m.
The RTC granted the respondent's application for registration of title to the subject properties. The RTC found
that the respondent was able to prove that the subject properties form part of the alienable and disposable lands
of the public domain.

The RTC opined that the elevations of the subject properties are very much higher than the reglementary
elevation of 12.50 m and, thus, not part of the bed of Laguna Lake.

The RTC likewise found that the respondent was able to prove that it and its predecessors-in-interest have been
in open, continuous, exclusive, and notorious possession of the subject properties as early as 1943.
The petitioner appealed to the CA. The CA affirmed the decision of the RTC. The CA likewise pointed out that
the respondent was able to present certifications issued by the DENR, attesting that the subject properties form
part of the alienable and disposable lands of the public domain, which was not disputed by the petitioner.
Hence, the instant petition.

Issue: Did the CA err in affirming the RTC Decision which granted the application for registration filed
by the respondent?

Held: Section 14(1) of P.D. No. 1529 refers to the judicial confirmation of imperfect or incomplete titles to
public land acquired under Section 48(b) of Commonwealth Act (C.A.) No. 141, or the Public Land Act, as
amended by P.D. No. 1073.Under Section 14(1) of P.D. No. 1529, applicants for registration of title must
sufficiently establish: first, that the subject land forms part of the disposable and alienable lands of the public
domain; second, that the applicant and his predecessors-in-interest have been in open, continuous, exclusive,
and notorious possession and occupation of the same; and third, that it is under a bona fide claim of ownership
since June 12, 1945, or earlier.

The first requirement was not satisfied in this case. To prove that the subject property forms part of the alienable
and disposable lands of the public domain, the respondent presented two certifications issued by Calamno,
attesting that Lot Nos. 3068 and 3077 form part of the alienable and disposable lands of the public domain.
However, the said certifications presented by the respondent are insufficient to prove that the subject properties
are alienable and disposable.

In Republic of the Philippines v. T.A.N. Properties, Inc., 578 Phil. 441 (2008).The Court clarified that, in
addition to the certification issued by the proper government agency that a parcel of land is alienable and
disposable, applicants for land registration must prove that the DENR Secretary had approved the land
classification and released the land of public domain as alienable and disposable. They must present a copy of
the original classification approved by the DENR Secretary and certified as true copy by the legal custodian of
the records.
Page 4

Respondent failed to do so because the certifications presented by respondent do not, by themselves, prove that
the land is alienable and disposable.

Anent the second and third requirements, the Court finds that the respondent failed to present sufficient
evidence to prove that it and its predecessors-in-interest have been in open, continuous, exclusive, and notorious
possession and occupation of the subject properties since June 12, 1945, or earlier.
Cerquena testified for the respondents that the subject properties were originally owned by Jaime who
supposedly possessed and cultivated the same since 1943; that sometime in 1975, Jaime sold the subject
properties to Salvador and Mijares who, in turn, sold the same to the respondent in 1989.
The foregoing are but unsubstantiated and self-serving assertions of the possession and occupation of the
subject properties by the respondent and its predecessors-in-interest; they do not constitute the well-nigh
incontrovertible evidence of possession and occupation of the subject properties required by Section 14(1) of
P.D. No. 1529.

For purposes of land registration under Section 14(1) of P.D. No. 1529, proof of specific acts of ownership must
be presented to substantiate the claim of open, continuous, exclusive, and notorious possession and occupation
of the land subject of the application. Applicants for land registration cannot just offer general statements which
are mere conclusions of law rather than factual evidence of possession. Actual possession consists in the
manifestation of acts of dominion over it of such a nature as a party would actually exercise over his own
property. Valiao v. Republic, G.R. No. 170757, November 28, 2011.

"A mere casual cultivation of portions of the land by the claimant does not constitute possession under claim of
ownership. For him, possession is not exclusive and notorious so as to give rise to a presumptive grant from the
state. The possession of public land, however long the period thereof may have extended, never confers title
thereto upon the possessor because the statute of limitations with regard to public land does not operate against
the state, unless the occupant can prove possession and occupation of the same under claim of ownership for the
required number of years." Del Rosario v. Republic of the Philippines, 432 Phil. 824

Further, the Court notes that the tax declarations over the subject properties presented by the respondent were
only for 2002. The respondent failed to explain why, despite its claim that it acquired the subject properties as
early as 1989, and that its predecessors-in-interest have been in possession of the subject property since 1943, it
was only in 2002 that it started to declare the same for purposes of taxation. "While tax declarations are not
conclusive evidence of ownership, they constitute proof of claim of ownership." Aide v. Bernal, G.R. No.
169336, March 18, 2010 GRANTED.
Lozada v. Bracewell. GR No. 179155, April 2, 2014
NICOMEDES J. LOZADA, Petitioner,vs. EULALIA BRACEWELL, EDDIE BRACEWELL, ESTELLITA
BRACEWELL, JAMES BRACEWELL, JOHN BRACEWELL, EDWIN BRACEWELL, ERIC
BRACEWELL, and HEIRS OF GEORGE BRACEWELL, Respondents.

Case law instructs that for “as long as a final decree has not been entered by the (Land Registration Authority
[LRA]) and the period of one (1) year has not elapsed from the date of entry of such decree, the title is not
finally adjudicated and the decision in the registration proceeding continues to be under the control and sound
discretion of the court rendering it

Facts: Petitioner filed an application for registration and confirmation of title over a parcel of land which was
granted by the RTC of Makati City acting as a land registration court. Consequently, on July 10, 1997, the LRA
issued a Decree in the name of petitioner, who later obtained an OCT. On February 6, 1998, within a year from
the issuance of the aforementioned decree, James Bracewell, Jr. (Bracewell) filed a petition for review of a
decree of registration under Section 32 of Presidential Decree No. (PD) 1529, otherwise known as the “Property
Page 5

Registration Decree,” before the RTC of Las Piñas City claiming that a portion of such land was his as absolute
owner and possessor and us fraudulently included in the Decree. He further averred that petitioner deliberately
concealed the fact that he (Bracewell) is one of the adjoining owners, and left him totally ignorant of the
registration proceedings involving said lots. Instead of impleading him, petitioner listed Bracewell’s
grandmother, Maria Cailles, as an adjoining owner, although she had already died by that time. Finding that
petitioner obtained Decree and OCT in bad faith, the Las Piñas City-RTC rendered a Decision in favor of
Bracewell, who had died during the pendency of the case and was substituted by Eulalia Bracewell and his
heirs. The Las Piñas City-RTC faulted petitioner for deliberately preventing respondents from participating and
objecting to his application for registration when the documentary evidence showed that, as early as 1962,
Bracewell had been paying taxes for the subject lot; and that he (Bracewell) was recognized as the owner
thereof in the records of the Bureau of Lands way back in 1965, as well as in the City Assessor’s Office.
Petitioner argues that the Las Piñas City-RTC had no jurisdiction over a petition for review of a decree of
registration under Section 32 of PD 1529, which should be filed in the same branch of the court that rendered
the decision and ordered the issuance of the decree (Makati City) The CA held that, since the petition for review
was filed within one (1) year from the issuance of the questioned decree, and considering that the subject lot is
located in Las Piñas City, the RTC of said city had jurisdiction over the case.

Issue: Whether or not the Las Piñas City-RTC has jurisdiction over the petition for review of decree,
which was issued as a result of the judgment rendered by the RTC of Makati City.

Held: Under the “Land Registration Act,” which was the law in force at the time of the commencement by both
parties of their respective registration proceedings — jurisdiction over all applications for registration of title
was conferred upon the Courts of First Instance (CFIs, now RTCs) of the respective provinces in which the land
sought to be registered is situated. Subsequently, Batas Pambansa Bilang (BP) 129,[39] otherwise known as
“The Judiciary Reorganization Act of 1980,” was enacted and took effect on August 14, 1981, authorizing the
creation of RTCs in different judicial regions, including the RTC of Las Piñas City as part of the National
Capital Judicial Region. As pointed out by the court, the RTC of Las Piñas City was established “in or about
1994.” Understandably, in February 1998, Bracewell sought the review of the Decree before the Las Piñas City-
RTC, considering that the lot subject of this case is situated in Las Piñas City. It should be pointed out, however,
that with the passage of PD 1529, the distinction between the general jurisdiction vested in the RTC and the
limited jurisdiction conferred upon it as a cadastral court was eliminated. Section 32. Review of decree of
registration; Innocent purchaser for value.—The decree of registration shall not be reopened or revised by
reason of absence, minority, or other disability of any person adversely affected thereby, nor by any proceeding
in any court for reversing judgments, subject, however, to the right of any person, including the government and
the branches thereof, deprived of land or of any estate or interest therein by such adjudication or confirmation of
title obtained by actual fraud, to file in the proper Court of First Instance a petition for reopening and review of
the decree of registration not later than one year from and after the date of the entry of such decree of
registration, but in no case shall such petition be entertained by the court where an innocent purchaser for value
has acquired the land or an interest therein, whose rights may be prejudiced. Whenever the phrase “innocent
purchaser for value” or an equivalent phrase occurs in this Decree, it shall be deemed to include an innocent
lessee, mortgagee, or other encumbrancer for value. Upon the expiration of said period of one year, the decree
of registration and the certificate of title issued shall become incontrovertible. Any person aggrieved by such
decree of registration in any case may pursue his remedy by action for damages against the applicant or any
other persons responsible for the fraud. As such, case law instructs that for “as long as a final decree has not
been entered by the [LRA] and the period of one (1) year has not elapsed from the date of entry of such decree,
the title is not finally adjudicated and the decision in the registration proceeding continues to be under the
control and sound discretion of the court rendering it.
Valiao v. Republic, GR No. 110757, Nov. 28, 2011
Page 6

PACIFICO M. VALIAO, for himself and in behalf of his co-heirs LODOVICO, RICARDO, BIENVENIDO, all
Surnamed VALIAO and NEMESIO M. GRANDEA, Petitioners, vs. REPUBLIC OF THE PHILIPPINES,
MACARIO ZAFRA, and MANUEL YUSAY, Respondents,

Facts: On August 11, 1987, petitioners filed with the RTC an application for registration of a parcel of land
situated in Barrio Galicia, Municipality of Ilog, Negros Occidental.

On June 20, 1988, private oppositors filed their Motion to Dismiss the application on the following grounds:
(1) the land applied for has not been declared alienable and disposable; (2) res judicata has set in to bar the
application for registration; and (3) the application has no factual or legal basis.

On August 24, 1988, the Republic of the Philippines (Republic), through the Office of the Solicitor General
(OSG), opposed the application for registration.

On July 3, 1989, the RTC denied private oppositors' Motion to Dismiss. Trial thereafter ensued.

In support of their application for registration, petitioners alleged that they acquired the subject property in
1947, upon the death of their uncle Basilio who purchased the land from a certain Fermin Payogao, pursuant to
a Deed of Sale dated May 19, 1916 entirely handwritten in Spanish language. Basilio possessed the land in
question from May 19, 1916 until his death in 1947. Basilio's possession was open, continuous, peaceful,
adverse, notorious, uninterrupted and in the concept of an owner. Upon Basilio's death, the applicants as co-
heirs possessed the said land until 1966, whenoppositor Zafra unlawfully and violently dispossessed them of
their property, which compelled them to file complaints of Grave Coercion and Qualified Theft against Zafra.
The RTC, in its Decision dated December 15, 1995, granted petitioners' application for registration of the
subject property.

Aggrieved by the Decision, the private oppositors and the Republic, through Assistant Prosecutor Josue A.
Gatin, filed an appeal with the CA, which reversed the trial court's findings in its Decision dated June 23, 2005.
Petitioners filed a motion for reconsideration, which was denied by the CA. Hence, the present petition.

Issue: Is the piece of land in question alienable and disposable land of the public domain.

Held: Under Rule 45, the principle is well-established that this Court is not a trier of facts and that only
questions of law may be raised. This rule, however, is subject to certain exceptions. One of these is when the
findings of the appellate court are contrary to those of the trial court. Due to the divergence of the findings of
the CA and the RTC, the Court will now re-examine the facts and evidence adduced before the lower courts.
Under Section 14 (1) of Presidential Decree No. (PD) 1529, otherwise known as the Property Registration
Decree, petitioners need to prove that: (1) the land forms part of the alienable and disposable land of the public
domain; and (2) they, by themselves or through their predecessors-in-interest, have been in open, continuous,
exclusive, and notorious possession and occupation of the subject land under a bona fide claim of ownership
from June 12, 1945 or earlier.

No such evidence was offered by the petitioners to show that the land in question has been classified as
alienable and disposable land of the public domain. In the absence of incontrovertible evidence to prove that the
subject property is already classified as alienable and disposable, we must consider the same as still inalienable
public domain. Verily, the rules on the confirmation of imperfect title do not apply unless and until the land
subject thereof is released in an official proclamation to that effect so that it may form part of the disposable
agricultural lands of the public domain.
Page 7

Palanca v. Republic. GR No. 151312, Aug. 30, 2006


HEIRS OF THE LATE SPOUSES PEDRO S. PALANCA AND SOTERRANEA RAFOLS VDA. DE
PALANCA namely: IMELDA R. PALANCA, MAMERTA R. PALANCA, OFELIA P. MIGUEL, ESTEFANIA
P. PE, CANDELARIA P. PUNZALAN, NICOLAS R. PALANCA, CONSTANTINO R. PALANCA,
EDMUNDO PALANCA, LEOCADIA R. PALANCA and OLIVERIO R. PALANCA, represented by their
attorney-in-fact, OFELIA P. MIGUEL, Petitioners, v. REPUBLIC OF THE PHILIPPINES, (represented by the
Lands Management Bureau), REGIONAL TRIAL COURT OF PALAWAN (Office of the Executive Judge) and
the REGISTER OF DEEDS OF PALAWAN, Respondents.

“the government should exercise a positive act to re-classify inalienable land to alienable for proper
disposition; action to recover property of public domain never prescribes”

Facts:
 The subject properties in this case are namely: a 239,980 sqm land in Busuanga and a 176,588 sqm land
in New Busuanga
 These parcels of land are claimed by the heirs of Palanca on the grounds of open possession without
opposition for 40yrs and a CFI decision naming herein petitioners as owners.
 After 23yrs, herein respondents pray for the annulment of aforementioned decision and reversion.
 Petitioners presented tax declarations and witnesses of their possession while respondents presented a
Land Classification Map and CENRO certification.
 CA ruled in favor of the respondents.
 Petitioner’s contention:
 The subject properties in this case are namely: a 239,980 sqm land in Busuanga and a 176,588
sqm land in New Busuanga
 These parcels of land are claimed by the heirs of Palanca on the grounds of open possession
without opposition for 40yrs and a CFI decision naming herein petitioners as owners.
 After 23yrs, herein respondents pray for the annulment of aforementioned decision and
reversion.
 Petitioners presented tax declarations and witnesses of their possession while respondents
presented a Land Classification Map and CENRO certification.
 CA ruled in favor of the respondents.
 Respondent’s contention
 The subject properties in this case are namely: a 239,980 sqm land in Busuanga and a 176,588
sqm land in New Busuanga
 These parcels of land are claimed by the heirs of Palanca on the grounds of open possession
without opposition for 40yrs and a CFI decision naming herein petitioners as owners.
 After 23yrs, herein respondents pray for the annulment of aforementioned decision and
reversion.
 Petitioners presented tax declarations and witnesses of their possession while respondents
presented a Land Classification Map and CENRO certification.
 CA ruled in favor of the respondents.
Issue/s: Whether or not the heirs of Pedro S. Palanca have ownership rights over the subject properties in
the instant case?

Held: The petitioners failed to provide proof that the disputed parcels of land were of an alienable nature,
therefore res judicata does not apply in this case due to the absence of the power of the CFI to dispose lands of
the same nature. The courts lost the authority to classify lands of public domain upon the effectivity of CA 141
which clearly stated that such power exclusively rests with the President.
The petition is denied for lack of merit.
Page 8

Sec. of DENR v. Yap, GR No. 167707, Oct. 8, 2008


THE SECRETARY OF THE DEPARTMENT OF ENVIRONMENT AND NATURAL RESOURCES, THE
REGIONAL EXECUTIVE DIRECTOR, DENR-REGION VI, REGIONAL TECHNICAL DIRECTOR FOR
LANDS, LANDS MANAGEMENT BUREAU, REGION VI PROVINCIAL ENVIRONMENT AND
NATURAL RESOURCES OFFICER OF KALIBO, AKLAN, REGISTER OF DEEDS, DIRECTOR OF LAND
REGISTRATION AUTHORITY, DEPARTMENT OF TOURISM SECRETARY, DIRECTOR OF PHILIPPINE
TOURISM AUTHORITY, petitioners, vs. MAYOR JOSE S. YAP, LIBERTAD TALAPIAN, MILA Y.
SUMNDAD, and ANICETO YAP, in their behalf and in behalf of all those similarly situated, respondents.

Facts: On November 10, 1978, then President Marcos issued Proc. No. 1801 declaring Boracay Island, among
other islands, caves and peninsulas in the Philippines, as tourist zones and marine reserves under the
administration of the Philippine Tourism Authority (PTA). President Marcos later approved the issuance
of PTA Circular 3-82 dated September 3, 1982, to implement Proclamation No. 1801.
Claiming that Proclamation No. 1801 and PTA Circular No 3-82 precluded them from filing an application for
judicial confirmation of imperfect title or survey of land for titling purposes, respondents-claimants Mayor .
Yap, Jr., and others filed a petition for declaratory relief with the RTC in Kalibo, Aklan
In their petition, respondents-claimants alleged that Proc. No. 1801 and PTA Circular No. 3-82 raised doubts on
their right to secure titles over their occupied lands. They declared that they themselves, or through their
predecessors-in-interest, had been in open, continuous, exclusive, and notorious possession and occupation in
Boracay since June 12, 1945, or earlier since time immemorial. They declared their lands for tax purposes and
paid realty taxes on them. Respondents-claimants posited that Proclamation No. 1801 and its implementing
Circular did not place Boracay beyond the commerce of man. Since the Island was classified as a tourist zone,
it was susceptible of private ownership. Under Section 48(b) of the Public Land Act, they had the right to have
the lots registered in their names through judicial confirmation of imperfect titles.
The Republic, through the OSG, opposed the petition for declaratory relief. The OSG countered that Boracay
Island was an unclassified land of the public domain. It formed part of the mass of lands classified as “public
forest,” which was not available for disposition pursuant to Section 3(a) of the Revised Forestry Code, as
amended. The OSG maintained that respondents-claimants’ reliance on PD No. 1801 and PTA Circular No. 3-82
was misplaced. Their right to judicial confirmation of title was governed by Public Land Act and Revised
Forestry Code, as amended. Since Boracay Island had not been classified as alienable and disposable, whatever
possession they had cannot ripen into ownership.
On July 14, 1999, the RTC rendered a decision in favor of respondents-claimants, declaring that, “PD 1810 and
PTA Circular No. 3-82 Revised Forestry Code, as amended.
The OSG moved for reconsideration but its motion was denied. The Republic then appealed to the CA. On In
2004, the appellate court affirmed in toto the RTC decision. Again, the OSG sought reconsideration but it was
similarly denied. Hence, the present petition under Rule 45.
On May 22, 2006, during the pendency the petition in the trial court, President Gloria Macapagal-Arroyo issued
Proclamation No. 1064 classifying Boracay Island partly reserved forest land (protection purposes) and partly
agricultural land (alienable and disposable).
On August 10, 2006, petitioners-claimants Sacay,and other landowners in Boracay filed with this Court an
original petition for prohibition, mandamus, and nullification of Proclamation No. 1064. They allege that the
Proclamation infringed on their “prior vested rights” over portions of Boracay. They have been in continued
possession of their respective lots in Boracay since time immemorial.
On November 21, 2006, this Court ordered the consolidation of the two petitions

Issue: the main issue is whether private claimants have a right to secure titles over their occupied
portions in Boracay.

Held: petitions DENIED. The CA decision is reversed.


Page 9

Except for lands already covered by existing titles, Boracay was an unclassified land of the public domain
prior to Proclamation No. 1064. Such unclassified lands are considered public forest under PD No. 705.
PD No. 705 issued by President Marcos categorized all unclassified lands of the public domain as public forest.
Section 3(a) of PD No. 705 defines a public forest as “a mass of lands of the public domain which has not
been the subject of the present system of classification for the determination of which lands are needed for forest
purpose and which are not.” Applying PD No. 705, all unclassified lands, including those in Boracay Island,
are ipso facto considered public forests. PD No. 705, however, respects titles already existing prior to its
effectivity.
The 1935 Constitution classified lands of the public domain into agricultural, forest or timber, such
classification modified by the 1973 Constitution. The 1987 Constitution reverted to the 1935 Constitution
classification with one addition: national parks. Of these, only agricultural lands may be alienated. Prior to
Proclamation No. 1064 of May 22, 2006, Boracay Island had never been expressly and administratively
classified under any of these grand divisions. Boracay was an unclassified land of the public domain.
A positive act declaring land as alienable and disposable is required. In keeping with the presumption of
State ownership, the Court has time and again emphasized that there must be a positive act of the
government, such as a presidential proclamation or an executive order; an administrative action; investigation
reports of Bureau of Lands investigators; and a legislative act or a statute. The applicant may also secure a
certification from the government that the land claimed to have been possessed for the required number of years
is alienable and disposable. The burden of proof in overcoming such presumption is on the person applying for
registration (or claiming ownership), who must prove that the land subject of the application is alienable or
disposable.
In the case at bar, no such proclamation, executive order, administrative action, report, statute, or certification
was presented to the Court. The records are bereft of evidence showing that, prior to 2006, the portions of
Boracay occupied by private claimants were subject of a government proclamation that the land is alienable and
disposable. Matters of land classification or reclassification cannot be assumed. They call for proof.
Proc. No. 1801 cannot be deemed the positive act needed to classify Boracay Island as alienable and disposable
land. If President Marcos intended to classify the island as alienable and disposable or forest, or both, he would
have identified the specific limits of each, as President Arroyo did in Proclamation No. 1064. This was not
done in Proclamation No. 1801.

Notes:
1. Private claimants’ reliance on Ankron and De Aldecoa is misplaced. Ankron and De Aldecoa were
decided at a time when the President of the Philippines had no power to classify lands of the public
domain into mineral, timber, and agricultural. At that time, the courts were free to make corresponding
classifications in justiciable cases, or were vested with implicit power to do so, depending upon the
preponderance of the evidence. Act No. 2874, promulgated in 1919 and reproduced in Section 6 of
Public Land Act, gave the Executive Department, through the President, the exclusive prerogative to
classify or reclassify public lands into alienable or disposable, mineral or forest. Since then, courts no
longer had the authority, whether express or implied, to determine the classification of lands of the
public domain.
2. Each case must be decided upon the proof in that particular case, having regard for its present or
future value for one or the other purposes. We believe, however, considering the fact that it is a
matter of public knowledge that a majority of the lands in the Philippine Islands are agricultural lands
that the courts have a right to presume, in the absence of evidence to the contrary, that in each case the
lands are agricultural lands until the contrary is shown. Whatever the land involved in a particular
land registration case is forestry or mineral land must, therefore, be a matter of proof. Its
superior value for one purpose or the other is a question of fact to be settled by the proof in each
particular case
Page 10

Forests, in the context of both the Public Land Act and the Constitution classifying lands of the public
domain into “agricultural, forest or timber, mineral lands, and national parks,” do not necessarily refer
to large tracts of wooded land or expanses covered by dense growths of trees and underbrushes. The
discussion in Heirs of Amunategui v. Director of Forestry is particularly instructive:

A forested area classified as forest land of the public domain does not lose such classification simply
because loggers or settlers may have stripped it of its forest cover. Parcels of land classified as forest
land may actually be covered with grass or planted to crops by kaingin cultivators or other farmers.
“Forest lands” do not have to be on mountains or in out of the way places. Swampy areas covered by
mangrove trees, nipa palms, and other trees growing in brackish or sea water may also be classified as
forest land. The classification is descriptive of its legal nature or status and does not have to be
descriptive of what the land actually looks like. Unless and until the land classified as “forest” is
released in an official proclamation to that effect so that it may form part of the disposable agricultural
lands of the public domain, the rules on confirmation of imperfect title do not apply.

There is a big difference between “forest” as defined in a dictionary and “forest or timber land” as a
classification of lands of the public domain as appearing in our statutes. One is descriptive of what
appears on the land while the other is a legal status, a classification for legal purposes. At any rate, the
Court is tasked to determine the legal status of Boracay Island, and not look into its physical layout.
Hence, even if its forest cover has been replaced by beach resorts, restaurants and other commercial
establishments, it has not been automatically converted from public forest to alienable agricultural land.

3. All is not lost, however, for private claimants. While they may not be eligible to apply for judicial
confirmation of imperfect title under Section 48(b) of CA No. 141, as amended, this does not denote
their automatic ouster from the residential, commercial, and other areas they possess now classified as
agricultural. Neither will this mean the loss of their substantial investments on their occupied alienable
lands. Lack of title does not necessarily mean lack of right to possess.

For one thing, those with lawful possession may claim good faith as builders of improvements. They
can take steps to preserve or protect their possession. For another, they may look into other modes of
applying for original registration of title, such as by homestead or sales patent, subject to the conditions
imposed by law.

More realistically, Congress may enact a law to entitle private claimants to acquire title to their occupied
lots or to exempt them from certain requirements under the present land laws. There is one such
bill now pending in the House of Representatives.
Cruz v. Sec. of DENR. GR No. 135385, Dec. 6, 2000
ISAGANI CRUZ and CESAR EUROPA, petitioners, vs. SECRETARY OF ENVIRONMENT AND
NATURAL RESOURCES, SECRETARY OF BUDGET AND MANAGEMENT and CHAIRMAN and
COMMISSIONERS OF THE NATIONAL COMMISSION ON INDIGENOUS PEOPLES, respondents.
Respondents: Secretary of Environment and Natural Resources Topic: Territory, People, and Government –
Territory

Summary: Petitioners assail the constitutionality of certain provisions of the Indigenous Peoples Rights Act of
1997 (IPRA) or RA 8371 and its Implementing Rules and Regulations (IRR). However, due to a tie vote (7-7)
after due deliberation and re-deliberation, the petition was dismissed. (Thus, the separate opinions are more
important in this case).
Page 11

Facts: Petitioners Isagani Cruz and Cesar Europa filed a suit for prohibition and mandamus as citizens and
taxpayers, assailing the constitutionality of certain provisions of Republic Act No. 8371, otherwise known as the
Indigenous People’s Rights Act of 1997 (IPRA) and its implementing rules and regulations (IRR). The
petitioners assail certain provisions of the IPRA and its IRR on the ground that these amount to an unlawful
deprivation of the State’s ownership over lands of the public domain as well as minerals and other natural
resources therein, in violation of the regalian doctrine embodied in section 2, Article XII of the Constitution.

Issue/s: Do the provisions of IPRA contravene the Constitution?

Held: No, the provisions of IPRA do not contravene the Constitution. Examining the IPRA, there is nothing in
the law that grants to the ICCs/IPs ownership over the natural resources within their ancestral domain.
Ownership over the natural resources in the ancestral domains remains with the State and the rights granted by
the IPRA to the ICCs/IPs over the natural resources in their ancestral domains merely gives them, as owners and
occupants of the land on which the resources are found, the right to the small scale utilization of these
resources, and at the same time, a priority in their large scale development and exploitation.
Additionally, ancestral lands and ancestral domains are not part of the lands of the public domain. They are
private lands and belong to the ICCs/IPs by native title, which is a concept of private land title that existed
irrespective of any royal grant from the State. However, the right of ownership and possession by the ICCs/IPs
of their ancestral domains is a limited form of ownership and does not include the right to alienate the same.
As the votes were equally divided (7 to 7) and the necessary majority was not obtained, the case was
redeliberated upon. However, after redeliberation, the voting remained the same. Accordingly, pursuant to Rule
56, Section 7 of the Rules of Civil Procedure, the petition is DISMISSED.

Notes: WoN Sections 3(a) and (b), 5, 6, 7, 8, and 57, and 58 of RA 8371/IPRA and its IRR are unconstitutional
for unlawfully depriving the State of its ownership over lands of the public domain, minerals, and other natural
resources therein, violating the regalian doctrine enshrined in Section 2, Article XII of the Constitution
● JusticeKapunan:NO.
Said provisions affirming the ownership by indigenous peoples of their ancestral lands and domains by
virtue of native title (definition: lands held in private ownership since time immemorial) do not diminish
the State’s ownership of lands within the public domain, because said ancestral lands and domains are
considered as private land, and never to have been part of the public domain, following the doctrine laid
down in Cariño v. Insular Government.
Section 3(a) does not confer or recognize any right of ownership over the natural resources to the
ICCs/IPs. Its purpose is definitional and not declarative of a right or title.
Section 57 only grants “priority rights” to ICCs/IPs in the utilization of natural resources and not
absolute ownership thereof. The State retains full control over the exploration, development and
utilization of natural resources through the imposition of requirements and conditions for the utilization
of natural resources under existing laws, such as the Small- Scale Mining Act of 1991 and the Philippine
Mining Act of 1995. Neither does the grant of said rights exclude non-indigenous peoples from
undertaking the same activities within the ancestral domains upon authority granted by the proper
governmental agency.
● JusticePuno:NO.
Ancestral lands and ancestral domains are not part of the lands of the public domain. They are private
and belong to the ICCs/IPs. The classification of lands in the public domain under Section 3, Article XII
of the Constitution does not include ancestral lands nor ancestral domains.
The rights of ICCs/IPs to their ancestral domains and ancestral lands may be acquired in two modes: (1)
by native title over both ancestral lands and domains; or (2) by torrens title under the Public Land Act
and the Land Registration Act with respect to ancestral lands only. Both modes presume or recognize the
land as private and not public.
Page 12

The right of ownership to ancestral domain under Section 7(a) involves “lands, bodies of water
traditionally and actually occupied by ICCs/IPs, sacred places, traditional hunting and fishing grounds,
and all improvements made by them at any time within the domains”, not “waters, minerals, coal,
petroleum, and other mineral oils, all forces of potential energy, fisheries, forests or timber, wildlife,
flora and fauna, and other natural resources” enumerated in Section 2, Article XII of the Constitution.
Ownership therefore of natural resources remain with the State.
Small-scale utilization of resources in Section 7(b) is also allowed under paragraph 3, section 2, Article
XII of the Constitution.
Finally, the large-scale utilization of natural resources in Section 57 of RA 8371/IPRA is allowed under
paragraphs 1 and 4, section 2, Article XII of the Constitution since only “priority rights” (which does not
necessarily mean ownership rights) are given to ICCs/IPs.
However, by including “natural resources”, Section 1, Part II, Rule III of the Implementing Rules goes
beyond Section 7(a) and therefore unconstitutional.
● JusticePanganiban:YES.
Section 3(a) [whose definition of ancestral domain encompasses natural resources found therein], and
3(b) [defines ancestral lands as those possessed by ICCs/IPs since time immemorial] contravene Section
2, Article XII of the Constitution, which declares that the State owns all lands of the public domain,
minerals, and natural resources – none of which, except agricultural lands, can be alienated. In addition,
mere possession or utilization of land, however long, does not automatically convert them into private
properties.
IPRA/RA 8371 does not specify limits to ancestral lands and domains.
IPRA/RA 8371 relinquishes the State’s power under Section 2, Article XII of the Constitution of full
control of natural resources in ancestral lands and ancestral domains in favor of ICCs/IPs, who may
exercise these rights without any time limit. In addition, they are also given the right to negotiate
directly the terms and conditions for the exploration of natural resources under Section 7(b), a right
vested by the Constitution only to the State.
● Justice Vitug: YES. Sections 7 and 57 go beyond the context of thefundamental law and virtually
amount to an undue delegation, if not an unacceptable abdication, of State authority over a significant
area of the country and its patrimony
WoN Sections 51 to 53, 59, 52(i), 63, 65, and 66 of RA 8371/IPRA, defining the powers and
jurisdiction of the NCIP and making customary law applicable to the settlement of disputes involving
ancestral domains and ancestral lands, violate the due process clause of the Constitution
● Justice Kapunan: NO. The fact the NCIP is composed wholly of indigenous peoples does not mean
that it is incapable of being impartial. Moreover, the use of customary laws is sanctioned by paragraph 2,
Section 5 of Article XII of the Constitution.
● JusticePuno: Notdiscussed.
● Justice Panganiban: Not discussed. It is best to await specific cases filed by
those whose rights may have been injured by these provisions.
● Justice Vitug: YES, but only on making customary law applicable to the
settlement of disputes involving ancestral domains and ancestral lands. The second paragraph of Section
5 of Article XII of the Constitution allows Congress to provide for the applicability of customary laws
governing property rights or relations in determining the ownership and extent of ancestral domains. I do
not see this statement as saying that Congress may enact a law that would simply express that customary
laws shall govern and end it there. No discussion on the powers and jurisdiction of the NCIP.
WoN Rule VII, Part II, Section 1 of the NCIP Administrative Order No. 1, series of 1998, which
provides that the administrative relationship of the NCIP to the Office of the President is characterized
as a lateral but autonomous relationship for purposes of policy and program coordination, infringes upon
the Presidents power of control over executive departments under Section 17, Article VII of the
Constitution
Page 13

● Justice Kapunan: NO, since said provision as well as Section 40 of the IPRA expressly places the
NCIP under the Office of the President, and therefore under the President’s control and supervision with
respect to its administrative functions. However, insofar as the decisions of the NCIP in the exercise of
its quasi-judicial powers are concerned, the same are reviewable by the Court of Appeals, like those of
the NLRC and the SEC.
● JusticePuno:Notdiscussed.
● Justice Panganiban: Not discussed. It is best to await specific cases filed by those whose rights may
have been injured by these provisions.
● JusticeVitug: Notdiscussed.

Republic v. Naguiat, GR No. 134209, Jan. 24, 2006


REPUBLIC OF THE PHILIPPINES, Petitioner, vs. CELESTINA NAGUIAT, Respondent.

Facts: petition for review under Rule 45 of the Rules of Court seeking the reversal of the Decision[1] dated
May 29, 1998 of the Court of Appeals (CA) in CA-G.R. CV No. 37001 which affirmed an earlier... decision[2]
of the Regional Trial Cour... in Land Registration Case No. N-25-1. factual backdrop
This is an application for registration of title to four (4) parcels of land located in Panan, Botolan, Zambales,
more particularly described in the amended application filed by Celestina Naguiat on 29 December 1989 with
the Regional Trial Court of Zambales, Branch.
Applicant [herein respondent] alleges, inter alia, that she is the owner of the said parcels of land having acquired
them by purchase from the LID Corporation which likewise acquired the same from Demetria Calderon,
Josefina Moraga and Fausto Monje and their... predecessors-in-interest who have been in possession... hereof
for more than thirty (30) years; and that to the best of her knowledge, said lots suffer no mortgage or
encumbrance of whatever kind nor is there any person having any interest, legal or equitable, or in possession...
thereof.

On 29 June 1990, the Republic of the Philippines [herein petitioner]. . . filed an opposition to the application on
the ground that neither the applicant nor her predecessors-in interest have been in open, continuous, exclusive
and notorious possession and occupation of the... lands in question since 12 June 1945 or prior thereto; that the
muniments of title and tax payment receipts of applicant do not constitute competent and sufficient evidence of
a bona-fide acquisition of the lands... applied for or of his open, continuous, exclusive and notorious...
possession and occupation thereof in the concept of (an) owner; that the applicant's claim of ownership in fee
simple on the basis of Spanish title or grant can no longer be availed of . . .; and that the parcels of land applied
for are part of the public domain belonging to the Republic of the Philippines not subject to private
appropriation.

On 15 October 1990, the lower court issued an order of... general default as against the whole world... and
proceeded with the hearing of this registration case.

In a decision[4] dated September 30, 1991, the trial court rendered judgment for herein respondent Celestina
Naguiat, adjudicating unto her the parcels of land in question and decreeing the registration thereof in her name
With its motion for reconsideration having been denied by the trial court, petitioner Republic went on appeal to
the CA... CA, in the herein assailed decision of May 29, 1998, affirmed that of the trial court,

Issues: Whether or not the areas in question have ceased to have the status of forest or other inalienable
lands of the... public domain.

Ruling: No, the said areas are still classified as forest land. Forests, in the context of both the Public Land
Act[7] and the Constitution[8] classifying lands of the public domain into "agricultural, forest or timber, mineral
Page 14

lands and national parks," do not necessarily refer to a large tract of... wooded land or an expanse covered by
dense growth of trees and underbrush.
A forested area classified as forest land of the public domain does not lose such classification simply because
loggers or settlers have stripped it of its forest cover. Parcels of land classified as forest land may actually be
covered with grass or planted to crops... by kaingin cultivators or other farmers. "Forest lands" do not have to be
on mountains or in out of the way places. xxx. The classification is merely descriptive of its legal nature or
status and does not have to be descriptive of what the land actually looks like.
xxx
Under Section 2, Article XII of the Constitution,[10] which embodies the Regalian doctrine, all lands of the
public domain belong to the State - the source of any asserted right to ownership of land.[11] All lands not
appearing to be... clearly of private dominion presumptively belong to the State.[12] Accordingly, public lands
not shown to have been reclassified or released as alienable agricultural land or alienated to a private person by
the State remain part of the inalienable public... domain.[13] Under Section 6 of the Public Land Act, the
prerogative of classifying or reclassifying lands of the public domain, i.e., from forest or mineral to agricultural
and vice versa, belongs to the Executive Branch of the government and not the... court.[14] Needless to stress,
the onus to overturn, by incontrovertible evidence, the presumption that the land subject of an application for
registration is alienable or disposable rests with the applicant.

In the present case, the CA assumed that the lands in question are already alienable and disposable.
The principal reason for the appellate court's disposition, finding a registerable title for respondent, is her and
her predecessor-in-interest's open, continuous and exclusive occupation of the subject property for more than 30
years. Prescinding from its above assumption and... finding, the appellate court went on to conclude,... hat, upon
the completion of the requisite period of... possession, the lands in question cease to be public land and become
private property.

Respondent never presented the required certification from the proper government agency or official
proclamation reclassifying the land applied for as alienable and disposable. Matters of land classification or
reclassification cannot be assumed. It calls for... proof.[18] Aside from tax receipts, respondent submitted in
evidence the survey map and technical descriptions of the lands, which, needless to state, provided no
information respecting the classification of the property. As the Court has held, however,... these documents are
not sufficient to overcome the presumption that the land sought to be registered forms part of the public domain
It cannot be overemphasized that unwarranted appropriation of public lands has been a notorious practice
resorted to in land registration cases.[20] For this reason, the Court has made it a point to stress, when
appropriate,... that declassification of forest... and mineral lands, as the case may be, and their conversion into
alienable and disposable lands need an express and positive act from the governmen... unclassified land...
cannot be acquired by adverse... occupation or possession; occupation thereof in the concept of owner, however
long, cannot ripen into private ownership and be registered as title.
Ligon v. Court of Appeals, GR No. 1077 51, June l, 1995
LETICIA P. LIGON, petitioner, vs. COURT OF APPEALS, JUDGE CELIA LIPANA-REYES, Presiding Judge,
Branch 81, Regional Trial Court of Quezon City, Iglesia ni Kristo and the Register of Deeds of Quezon
City, respondent.

Facts:
 The Islamic Directorate of the Philippines (IDP), by virtue of an absolute deed, sold to Iglesia ni Kristo
(INK) 2 parcels of land in Tandang Sora, Barrio Culiat, QC.
 It was stipulated therein that IDP shall undertake to evict all squatters in the property within 45 days
from the execution of the contract.
 IDP failed to do such, hence, INK sued for specific performance with damages.
Page 15

 IDP, on the other hand, alleged that it was INK which violated the contract by delaying the payment of
the purchase price and sought to have the contract of sale rescinded.
 Thereafter, INK filed a motion for partial summary judgment on the ground that there was actually no
genuine issue as to any material fact; the TC granted.
A year after, INK filed a motion in the same case seeking to compel Leticia Ligon (petitioner), who was in
possession of the certificates of title over the properties as mortgagee of IDP, to surrender said certificates to the
RD of QC for the registration of the absolute deed of sale in its name. Ligon allegedly refused and/or failed to
deliver the certificates despite repeated requests.
To this, Ligon opposed saying that
a) IDP was not served copy of the motion,
b) (b) ownership of INK over the property was still in issue,
c) (c) and that the trial court had no jurisdiction as the motion involved the registrability of the document
of sale, and she was not made a party in the main case.
The TC granted INK’s motion and ordered petitioner to surrender the certificates of title in open court for the
registration of the absolute deed of sale in the latter’s name and the annotation of the mortgage executed in
favor of petitioner on the new certificates (to be issued to INK).
Upon Ligon’s motion, the TC redirected her to deliver the documents to the RD of QC.

Issue: W/N INK has a superior right to the possession of the owner’s copies of the certificate of title.

Held: YES. Under our land registration law, no voluntary instrument shall be registered by the Register of
Deeds unless the owner’s duplicate certificate is presented together with such instrument, except in some cases
or upon the order of the court for cause shown. In case the person in possession refuses or fails to surrender the
same to the RD so that a voluntary document may be registered and a new certificate issued, Sec. 107 of P.D.
No. 1529 states:
“Where a voluntary instrument cannot be registered by reason of the refusal or failure of the holder to
surrender the owner’s duplicate, the party in interest may file a petition in court to compel surrender of the
same to the RD. The court, after hearing, may order the registered owner or any person withholding the
duplicate certificate to surrender the same and direct the entry of a new certificate or memorandum upon such
surrender. If the person withholding the duplicate certificate is not amenable to the process of the court, of if for
any reason the outstanding owner’s duplicate cannot be delivered, the court may order the annulment of the
same as well as the issuance of a new certificate of title in lieu thereof. “

Pursuant to Sec. 2 of P.D. No. 1529, the distinction between the RTC’s general and the limited jurisdiction when
acting merely as a cadastral court has been eliminated.
Aimed at avoiding multiplicity of suits, the change has simplified registration proceedings by conferring upon
the RTCs the authority to act not only on applications for original registration but also over petitions filed after
original registration of title, with power to hear and determine all questions arising upon such applications or
petitions.

Even while Sec. 107 of PD 1529 speaks of a petition which can be filed by one who wants to compel another to
surrender the certificates of title to the RD, this does not preclude a party to a pending case to include as
incident therein the relief stated under said section, especially if the subject certificates of title to be surrendered
are intimately connected with the subject matter of the principal action. The principal action is based on
expediency and in accordance with the policy against multiplicity of suits.
The order directing the surrender of the certificates to the RD in order that the deed be registered in favor of
INK cannot in any way prejudice her rights and interests as mortagee, since any lien annotated on the
previous certificates which subsists shall be incorporated or carried over to the new certificates of title.
It is inseparable from the property mortgaged as it is a right in rem — a lien on the property whoever its
owner may be. It subsists notwithstanding a change in ownership; in short, the personality of the owner is
Page 16

disregarded. Thus, all subsequent purchasers must respect the mortgage whether the transfer to them be with or
without the consent of the mortgagee, for such mortgage until discharged follows the property.9 It is clear
therefore that the surrender by petitioner of the certificates of title to the Register of Deeds as ordered by
the trial court will not create any substantial injustice to her.

To grant the petition and compel INK to file a new action in order to obtain the same reliefs it asked in the
motion before the trial court is to encourage litigations where no substantial rights are prejudiced. This end
should be avoided. Courts should not be so strict about procedural lapses that do not really impair the proper
administration of justice. The rules are intended to insure the orderly conduct of litigations because of the higher
objective they seek, which is, to protect the parties' substantive rights.
Intestate Estate of Don Mariano San Pedro v. Court of Appeals, GR No. 103727, Dec. 1, 1996
INTESTATE ESTATE OF THE LATE DON MARIANO SAN PEDRO Y ESTEBAN, represented by its HEIR-
JUDICIAL ADMINISTRATOR, ENGRACIO F. SAN PEDRO, petitioner-appellant, vs. COURT OF APPEALS
(Second Division) AURELIO OCAMPO, DOMINADOR D. BUHAIN, TERESA C. DELA
CRUZ, respondents-appellees.

Facts
The most fantastic land claim in the history of the Philippines is the subject of controversy in these two
consolidated cases. The heirs of the late Mariano San Pedro y Esteban laid claim and have been laying claim to
the ownership of, against third persons and the Government itself, a total land area of approximately 173,000
hectares or 214,047 quiniones,[ on the basis of a Spanish title, entitled Titulo de Propriedad Numero 4136 dated
April 25, 1894.

The claim, according to the San Pedro heirs, appears to cover lands in the provinces of Nueva Ecija, Bulacan,
Rizal, Laguna and Quezon; and such Metro Manila cities as Quezon City, Caloocan City, Pasay City, City of
Pasig and City of Manila, thus affecting in general lands extending from Malolos, Bulacan to the City Hall of
Quezon City and the land area between Dingalan Bay in the north and Tayabas Bay in the south.
Considering the vastness of the land claim, innumerable disputes cropped up and land swindles and
rackets proliferated resulting in tedious litigation in various trial courts, in the appellate court and in the
Supreme Court, in connection therewith. G.R No. 103727, an appeal by certiorari, arose out of a complaint
for recovery of possession and/or damages with a prayer for a writ of preliminary injunction. This was
dismissed by the Regional Trial Court, National Capital Judicial Region, Branch 104, Quezon City in its
decision dated July 7, 1989, the dispositive portion of which reads:
WHEREFORE, judgment is hereby rendered, dismissing the complaint against the defendants Aurelio Ocampo,
Dominador Buhain and Teresa dela Cruz and ordering plaintiff to pay each of the herein defendants, the sum of
FIVE THOUSAND PESOS (P5,000.00) as and for attorney’s fees, and to pay the costs of suit.
The said complaint for recovery of possession of real property and/or reconveyance with damages and with a
prayer for preliminary injunction was filed on August 15, 1988 by Engracio San Pedro as heir-judicial
administrator of the Intestate Estate of Don Mariano San Pedro y Esteban against Jose G. De Ocampo, Aurelio
Ocampo, MARECO, Inc., Rey Antonio Noguera, Teresa C. dela Cruz, Gaudencio R. Soliven, Diomedes Millan,
Carmen Rayasco, Dominador D. Buhain, Mario D. Buhain, Jose D. Buhain, Arestedes S. Cauntay, Manuel
Chung and Victoria Chung Tiu (El Mavic Investment & Development Corporation), Capitol Hills Realty
Corporation and Jose F. Castro. The complaint was docketed as Civil Case No. Q-88-447 in Branch 104,
Regional Trial Court of Quezon City.
In the complaint, it was alleged, among others:
1. that Engracio San Pedro discovered that the aforenamed defendants were able to secure from the
Registry of Deeds of Quezon City titles to portions of the subject estate, particularly Transfer
Certificates of Title Nos. 1386, 8982, 951975-951977, 313624, 279067, 1412, 353054, 372592, 149120,
Page 17

86404, 17874-17875, all emanating from Original Certificate of Title No. 614 and Transfer Certificates
of Title Nos. 255544 and 264124, both derivatives of Original Certificate of Title No. 333;
2. that the aforesaid defendants were able to acquire exclusive ownership and possession of certain
portions of the subject estate in their names through deceit, fraud, bad faith and misrepresentation;
3. that Original Certificates of Title Nos. 614 and 333 had been cancelled by and through a final and
executory decision dated March 21, 1988 in relation to letter recommendations by the Bureau of Lands,
Bureau of Forest Development and the Office of the Solicitor General and also in relation to Central
Bank Circulars dated April 7, 1971, April 23, 1971, September 12, 1972 and June 10, 1980; and
4. that the issue of the existence, validity and genuineness of Titulo Propriedad No. 4136 dated April 25,
1894 which covers the subject estate had been resolved in favor of the petitioner estate in a decision
dated April 25, 1978 by the defunct Court of First Instance, Branch 1 of Baliwag, Bulacan pertaining to
a case docketed as Special Proceeding No. 312-B.
Summons were served on only five of the aforementioned defendants, namely, Aurelio Ocampo, MARECO,
Inc., Teresita G. dela Cruz, Dominador Buhain and Manuel Chung and Victoria Chung Tiu.
On February 7, 1989, the lower court ordered the dismissal of the complaint against Mareco, Inc. for improper
service of summons and against Manuel Chung and Victoria Chung Tiu for lack of cause of action considering
that the registered owner of the parcel of land covered by TCT No. 86404 is El Mavic Investment and
Development Co., Inc., not Manuel Chung and Victoria Chung Tiu.
Trial on the merits proceeded against the private respondents Ocampo, Buhain and Dela Cruz.
a) On July 7, 1989, the lower court rendered judgment dismissing the complaint based on the following
grounds: Ocampo, Buhain and Dela Cruz are already the registered owners of the parcels of land
covered by Torrens titles which cannot be defeated by the alleged Spanish title, Titulo Propriedad No.
4136, covering the subject estate; and
b) the decision of the Court of First Instance of Bulacan entitled In the Matter of the Intestate Estate of the
late Don Mariano San Pedro y Esteban specifically stated in its dispositive portion that all lands which
have already been legally and validly titled under the Torrens system by private persons shall be
excluded from the coverage of Titulo Propriedad No. 4136.
Issue: Whether the claim of 173 hectares land is valid

Held: NO.
(1) Titulo de Propriedad No. 4136 is declared null and void and, therefore, no rights could be derived therefrom;
(2) All lands covered by Titulo de Propriedad No. 4136 are excluded from the inventory of the estate of the late
Mariano San Pedro y Esteban;
(3) The petition for letters of administration, docketed as Special Proceedings No. 312-B, should be, as it is,
hereby closed and terminated.
(4) The heirs, agents, privies and/or anyone acting for and in behalf of the estate of the late Mariano San Pedro y
Esteban are hereby disallowed to exercise any act of possession or ownership or to otherwise, dispose of in any
manner the whole or any portion of the estate covered by Titulo de Propriedad No. 4136; and they are hereby
ordered to immediately vacate the same, if they or any of them are in possession thereof.

RATIO: The Title embraces land `located in the Provinces of Bulacan, Rizal, Quezon, and Quezon City.
Second. The title was signed only by the provincial officials of Bulacan, and inscribed only in the Land Registry
of Bulacan. Why? The situation, indeed, cries desperately for a plausible answer.
To be underscored at this point is the well-embedded principle that private ownership of land must be proved
not only through the genuineness of title but also with a clear identity of the land claimed. (Oligan v. Mejia, 17
Phil. 494, 496; Villa Abrille v. Banuelos, 20 Phil. 1, 8, citing Sison v. Ramos, 13 Phil. 54 and Belen v. Belen, 13
Phil. 202; Licad v. Bacani, 51 Phil 51, 54-56; Lasam v. Director, 65 Phil. 367, 371. This Court ruled in a case
involving a Spanish title acquired by purchase that the land must be concretely measured per hectare or per
quinon, not in mass (cuerpos ciertos), (Valdez v. Director, 62 Phil. 362, 373, 375). The fact that the Royal
Page 18

Decree of August 31, 1888 used 30 hectares as a basis for classifying lands strongly suggests that the land
applied for must be measured per hectare.
Here, no definite area seems to have been mentioned in the title. In Piadecos Rejoinder to Opposition dated
April 28, 1964 filed in Civil Case 3035-M, it specified that area covered by its Titulo de Propiedad as 74,000
hectares (Rollo in L-24796, p. 36). In its Opposition of May 13, 1964 in the same case, it described the land as
containing 72,000 hectares (Id., p. 48). Which is which? This but accentuates the nebulous identity of Piadecos
land. Piadecos ownership thereof then equally suffers from vagueness, fatal at least in these proceedings.
Piadeco asserts that Don Mariano San Pedro y Esteban, the original owner appearing on the title, acquired his
rights over the property by prescription under Articles 4 and 5 of the Royal Decree of June 25, 1880, (Rollo of
L-24796, p. 184) the basic decree that authorized adjustment of lands. By this decree, applications for
adjustment -- showing the location, boundaries and area of land applied for -- were to be filed with the
Direccion General de Administracion Civil, which then ordered the classification and survey of the land with
the assistance of the interested party or his legal representative (Ponce, op. cit., p. 22).
The Royal Decree of June 5, 1880 also fixed the period for filing applications for adjustment at one year from
the date of publication of the decree in the Gaceta de Manila on September 10, 1880, extended for another year
by the Royal Order of July 15, 1881 (Ibid.). If Don Mariano sought adjustment within the time prescribed, as he
should have, then, seriously to be considered here are the Royal Orders of November 25, 1880 and of October
26, 1881, which limited adjustment to 1,000 hectares of arid lands, 500 hectares of land with trees and 100
hectares of irrigable lands (See: Government v. Avila, 46 Phil. 146, 154; Bayot v. Director of Lands, 98 Phil.
935, 941. Article 15 of the Royal Decree of January 26, 1889 limited the area that may be acquired by purchase
to 2,500 hectares, with allowable error up to 5%. Ponce, op. cit., p. 19). And, at the risk of repetition, it should
be stated again that Piadecos Titulo is held out to embrace 72,000 or 74,000 hectares of land.

CHAPTER 2
The Land registration Commission & Its registration of Deeds
Abrigo v. De Vera, GRT No. 154409, June 21, 2004
Spouses NOEL and JULIE ABRIGO, petitioners, vs. ROMANA DE VERA, respondent.
Rule on Double Sales: Between two buyers of the same immovable property registered under the Torrens
system, the law gives ownership priority to (1) the first registrant in good faith; (2) then, the first possessor in
good faith; and (3) finally, the buyer who in good faith presents the oldest title. This provision, however, does
not apply if the property is not registered under the Torrens system.

Facts: On May 27, 1993, Gloria Villafania sold a house and lot covered by a Tax Declaration to Rosenda Tigno-
Salazar and Rosita Cave-Go. The said sale became a subject of a suit for annulment of documents between the
vendor and the vendees.
The RTC rendered judgment approving the Compromise Agreement submitted by the parties. Villafania was
given one year to buy back the house and lot. Gloria Villafania failed to buy back the house and lot. Unknown,
however to Rosenda Tigno-Salazar and Rosita Cave-Go, Gloria Villafania obtained a free patent over the parcel
of land involved.
On October 16, 1997, Rosenda Tigno-Salazar and Rosita Cave-Go, sold the house and lot to the herein
[Petitioner-Spouses Noel and Julie Abrigo].
On October 23, 1997, Gloria Villafania sold the same house and lot to Romana de Vera x x x. Romana de Vera
registered the sale and as a consequence, TCT No. 22515 was issued in her name.
On November 21, 1997, petitioners filed a case for the annulment of documents, injunction, preliminary
injunction, restraining order and damages against respondent and Gloria Villafania. The lower court rendered
the assailed Decision dated January 4, 1999, awarding the properties to [petitioners] as well as damages. Both
parties appealed to the CA. The CA issued its March 21, 2002 Amended Decision, finding Respondent De Vera
Page 19

to be a purchaser in good faith and for value. The appellate court ruled that she had relied in good faith on the
Torrens title of her vendor and must thus be protected.
Hence, this Petition.

Issue: Who between petitioner-spouses and respondent has a better right to the property.

Held: Respondent Romana De Vera.


Law on Double Sale
Petitioners contend that Gloria Villafania could not have transferred the property to Respondent De Vera
because it no longer belonged to her. They further claim that the sale could not be validated, since respondent
was not a purchaser in good faith and for value.
Article 1544 of the Civil Code states the law on double sale. Otherwise stated, the law provides that a double
sale of immovables transfers ownership to (1) the first registrant in good faith; (2) then, the first possessor in
good faith; and (3) finally, the buyer who in good faith presents the oldest title. There is no ambiguity in the
application of this law with respect to lands registered under the Torrens system.
In the instant case, both Petitioners Abrigo and respondent registered the sale of the property. Since Neither
Petitioners nor their predecessors (Tigno-Salazar and Cave-Go) knew that the property was covered by the
Torrens system, they registered their respective sales under Act 3344. For her part, respondent registered the
transaction under the Torrens systembecause, during the sale, Villafania had presented the transfer certificate of
title (TCT) covering the property.
Respondent De Vera contends that her registration under the Torrens system should prevail over that of
petitioners who recorded theirs under Act 3344. De Vera relies on the following insight of Justice Edgardo L.
Paras:

“x x x If the land is registered under the Land Registration Act (and has therefore a Torrens Title), and it is sold
but the subsequent sale is registered not under the Land Registration Act but under Act 3344, as amended, such
sale is not considered REGISTERED, as the term is used under Art. 1544 x x x.”
Soriano v. Heirs of Magali held that registration must be done in the proper registry in order to bind the land.
Since the property in dispute in the present case was already registered under the Torrens system, petitioners’
registration of the sale under Act 3344 was not effective for purposes of Article 1544 of the Civil Code.
Under Act No. 3344, registration of instruments affecting unregistered lands is ‘without prejudice to a third
party with a better right.’ The aforequoted phrase has been held by this Court to mean that the mere registration
of a sale in one’s favor does not give him any right over the land if the vendor was not anymore the owner of the
land having previously sold the same to somebody else even if the earlier sale was unrecorded.
Petitioners cannot validly argue that they were fraudulently misled into believing that the property was
unregistered.
A Torrens title, once registered, serves as a notice to the whole world. All persons must take notice, and no one
can plead ignorance of the registration.
Good-Faith Requirement
Knowledge gained by the second buyer of the first sale defeats his rights even if he is first to register the second
sale, since such knowledge taints his prior registration with bad faith. This is the price exacted by Article 1544
of the Civil Code for the second buyer being able to displace the first buyer; that before the second buyer can
obtain priority over the first, he must show that he acted in good faith throughout (i.e. in ignorance of the first
sale and of the first buyer’s rights) ---- from the time of acquisition until the title is transferred to him by
registration, or failing registration, by delivery of possession.
As can be gathered from the foregoing, constructive notice to the second buyer through registration under Act
3344 does not apply if the property is registered under the Torrens system, as in this case.
"The registration contemplated under Art. 1544 has been held to refer to registration under Act 496 Land
Registration Act (now PD 1529) which considers the act of registration as the operative act that binds the land
(see Mediante vs. Rosabal, 1 O.G. [12] 900, Garcia vs. Rosabal, 73 Phil 694). On lands covered by the Torrens
Page 20

System, the purchaser acquires such rights and interest as they appear in the certificate of title, unaffected by
any prior lien or encumbrance not noted therein. The purchaser is not required to explore farther than what the
Torrens title, upon its face, indicates. The only exception is where the purchaser has actual knowledge of a flaw
or defect in the title of the seller or of such liens or encumbrances which, as to him, is equivalent to registration
(see Sec. 39, Act 496; Bernales vs. IAC, G.R. 75336, 18 October
1988; Hernandez vs. Sales, 69 Phil 744; Tajonera vs. Court of Appeals, L-26677, 27 March 1981),"
Respondent in Good Faith
The Court of Appeals examined the facts to determine whether respondent was an innocent purchaser for value.
After its factual findings revealed that Respondent De Vera was in good faith.
WHEREFORE, the Petition is DENIED and the assailed Decision AFFIRMED.
Guaranteed Homes v. Valdez, GR No. l7l53l, Jan. 30, 2009
GUARANTEED HOMES, INC., Petitioner, vs. HEIRS OF MARIA P. VALDEZ, (EMILIA V. YUMUL and
VICTORIA V. MOLINO), HEIRS OF SEVERINA P. TUGADE (ILUMINADA and LEONORA P. TUGADE,
HEIRS OF ETANG P. GATMIN (LUDIVINA G. DELA CRUZ (by and through ALFONSO G. DELA CRUZ),
HILARIA G. COBERO and ALFREDO G. COBERO) and SIONY G. TEPOL (by and through ELENA T.
RIVAS and ELESIO TEPOL, JR.), AS HEIRS OF DECEDENT PABLO PASCUA, Respondents.
Petition for review on certiorari of CA decision and resolution. Justice Tinga

Facts:
● CA reversed the order of RTC-Olongapo, which granted the motion to dismiss filed by petitioner
Guaranteed Homes.
● Respondents, descendants of Pablo Pascua, filed a complaint seeking reconveyance of a parcel of land in
Cabitaugan, Subic Zambales and covered by OCT No. 404 in the name of Pablo.
● They are praying for damages.
● From the annexes attached in the complaint, it was ascertained that the OCT contained several
annotations which showed that the property had been sold by Pablo to Alejandria Marquinez and
Resituto Morales.
● Also attached in the complaint, and averred by the respondents:
○ Extrajudicial Settlement of a Sole Heir and Confirmation of Sales executed by Cipriano Pascua,
Sr., declaring himself as the only heir of Pablo and confirming the sales made by Pablo to
spouses Rodolfo;
○ TCT Nos. T-8241 – issued in the name of Cipriano without cancellation of OCT No. 404 & not
signed by Register of Deeds;
○ TCT No. T-8242 – issued in the name of spouses Rodolfo, which canceled T-8241;
○ TCT No. T-10863 – issued in the name of petitioner when spouses Rodolfo sold the property to
petitioner, which canceled T-8242;
○ Deed of Sale with Mortgage between spouses Albino Rodolfo and Fabia Rodolfo and
Guaranteed Homes;
● Jorge Pascua, Sr., son of Cipriano filed a petition with RTC-Olongapo for the issuance of a new owner’s
duplicate of OCT No. 404, which was denied. The trial court held that petitioner was already the owner
of the subject property, noting that the failure to annotate the subsequent property to it at the back of said
OCT did not affect its title to the property.
● Petitioner filed a motion to dismiss the complaint on the following grounds:
○ Action is barred by the Statute of Limitations (since more than 28 years have passed since the
issuance from T-10863 to the filing of the complaint);
○ No cause of action, since petitioner is an innocent purchaser for value relying on the clean title of
spouses Rodolfo.
● RTC granted the motion.
● CA reversed and held that the respondents’ complaint for quieting of title had not yet prescribed.
Page 21

● Hence, this petition.

Issue: Whether petitioner is an innocent purchaser for value, i.e. there is no need to go beyond the
registered title of spouses Rodolfo.

Held: Yes (there is no need for petitioner to go beyond the clean title presented to them). Petition granted. CA
decision reversed and set aside.

Ratio:
● The Court held that it is basic that a person dealing with registered property need not go beyond, but
only has to rely on, the title of his predecessor-in-interest. Since “the act of registration is the operative
act to convey or affect the land insofar as third persons are concerned,” it follows that where there is
nothing in the certificate of title to indicate any cloud or vice in the ownership of the property, or any
encumbrance thereon, the purchaser is not required to explore farther than what the Torrens title upon its
face indicates in quest for any hidden defect or inchoate right that may subsequently defeat his right
thereto.
● In the present case, it is enough that petitioner examined the latest certificate of title issued in the name
of spouses Rodolfo. The petitioner as purchaser is not bound by the original certificate but only by the
certificate of title of the person from whom he had purchased the property.
● Furthermore, registration in the public registry is considered a notice to the whole world. Every
conveyance, mortgage, lease, lien, attachment, order, judgment, instrument or entry affecting registered
land shall be, if registered, filed or entered in the Office of the Register of Deeds of the province or city
where the land to which it relates lies, be constructive notice to all persons from the time of such
registering, filing or entering.
Baranda. Gustilo, GR No. 81153, Sept. 26, 1988
EDUARDO S. BARANDA and ALFONSO HITALIA, Petitioners, v. HONORABLE JUDGE TITO GUSTILO,
ACTING REGISTER OF DEEDS AVITO SACLAUSO, HONORABLE COURT OF APPEALS, and ATTY.
HECTOR P. TEODOSIO, Respondents.

Facts:
 A petition for reconstitution of title was filed with the CFI (now RTC) of Iloilo involving a parcel of
land known as Lot No. 4517 of the Sta. Barbara Cadastre covered by OCT No. 6406 in the name of
Romana Hitalia.
 The OCT was cancelled and TCT No. 106098 was issued in the names of petitioners Baranda and
Hitalia.
 The Court issued a writ of possession which Gregorio Perez, Maria P. Gotera and Susana Silao refused
to honor on the ground that they also have TCT No. 25772 over the same Lot No. 4517.
 The Court found out that TCT No. 257772 was fraudulently acquired by Perez, Gotera and Susana.
 Thereafter, the court issued a writ of demolition which was questioned by Perez and others so a motion
for reconsideration was filed.
 Another case was filed by Baranda and Hitalia (GR. NO. 62042) for the execution of judgement in the
resolutions issued by the courts.
 In the meantime, the CA dismissed a civil case (GR. NO. 00827) involving the same properties. (NOTE:
This time three cases na ang involve excluding the case at bar.)
 The petitioners prayed that an order be released to cancel No.T-25772. Likewise to cancel No.T-106098
and once cancelled to issue new certificates of title to each of Eduardo S. Baranda and Alfonso Hitalia
To cancel No.T-25772. Likewise to cancel No.T-106098 and once cancelled to issue new certificates of
title to each of Eduardo S. Baranda and Alfonso Hitalia.
Page 22

 In compliance with the order or the RTC, the Acting Register of Deeds Avito Saclauso annotated the
order declaring TCT T-25772 null and void, cancelled the same and issued new certificate of titles in the
name of petitioners.
 However, by reason of a separate case pending in the Court of Appeals, a notice of lis pendens was
annotated in the new certificate of title.
 This prompted the petitioners to move for the cancellation of the notice of lis pendens in the new
certificates.
 Judge Tito Gustilo then ordered the Acting Register of Deeds for the cancellation of the notice of lis
pendens but the Acting Register of Deeds filed a motion for reconsideration invoking Sec 77 of PD
1529.

Issue: What is the nature of the duty of a Register of Deeds to annotate or annul a notice of lis pendens in
a torrens certificate of title.

Held:
Section 10, Presidential Decree No. 1529 states that "It shall be the duty of the Register of Deeds to
immediately register an instrument presented for registration dealing with real or personal property which
complies with all the requisites for registration. ... If the instrument is not registrable, he shall forthwith deny
registration thereof and inform the presentor of such denial in writing, stating the ground or reasons therefore,
and advising him of his right to appeal by consulta in accordance with Section 117 of this Decree."
Section 117 provides that "When the Register of Deeds is in doubt with regard to the proper step to be taken or
memoranda to be made in pursuance of any deed, mortgage or other instrument presented to him for registration
or where any party in interest does not agree with the action taken by the Register of Deeds with reference to
any such instrument, the question shall be submitted to the Commission of Land Registration by the Register of
Deeds, or by the party in interest thru the Register of Deeds. ... ."
The function of ROD is ministerial in nature
The function of a Register of Deeds with reference to the registration of deeds encumbrances, instruments and
the like is ministerial in nature. The respondent Acting Register of Deeds did not have any legal standing to file
a motion for reconsideration of the respondent Judge's Order directing him to cancel the notice of lis pendens
annotated in the certificates of titles of the petitioners over the subject parcel of land.
In case of doubt as to the proper step to be taken in pursuance of any deed ... or other instrument presented to
him, he should have asked the opinion of the Commissioner of Land Registration now, the Administrator of the
National Land Title and Deeds Registration Administration in accordance with Section 117 of Presidential
Decree No. 1529.

No room for construction for the laws on functions of ROD


The elementary rule in statutory construction is that when the words and phrases of the statute are clear and
unequivocal, their meaning must be determined from the language employed and the statute must be taken to
mean exactly what it says. The statute concerning the function of the Register of Deeds to register instruments
in a torrens certificate of title is clear and leaves no room for construction.
De Leon v. De Leon, GR No. 185063, July 23, 2009
SPS. LITA DE LEON and FELIX RIO TARROSA, Petitioners, vs. ANITA B. DE LEON, DANILO B. DE
LEON, and VILMA B. DE LEON, Respondents.

Facts: On July 20, 1965, Bonifacio De Leon, then single, and the People’s Homesite and Housing Corporation
(PHHC) entered into a Conditional Contract to Sell for the purchase on installment of a lot situated in Quezon
City. On April 24, 1968, Bonifacio married Anita de Leon. They had two children, Danilo and Vilma. On June
22, 1970, PHHC executed a Final Deed of Sale in favor of Bonifacio upon full payment of the price of the lot.
TCT was issued on February 24, 1972 in the name of Bonifacio, “single.” On January 12, 1974, Bonifacio sold
Page 23

the lot to his sister, Lita, and her husband, Felix Tarrosa. The Deed of Sale did not bear the written consent and
signature of Anita. On February 29, 1996, Bonifacio died.
Three months later, Tarrosa spouses registered the Deed of Sale. Anita, Danilo, and Vilma filed a reconveyance
suit allegeing that Bonifacio was still the owner of the lands. Tarrosa spouses averred that the lot Bonifacio sold
to them was his exclusive property because he was still single when he acquired it from PHHC. They further
alleged that they were not aware of the marriage between Bonifacio and Anita at the time of the execution of the
Deed of Sale.
The RTC ruled in favor of Anita De Leon et al stating that the lot in question was the conjugal property of
Bonifacio and Anita. The CA affirmed the decision of the RTC. Hence, this petition.

Issue: W/N the property that Bonifacio has purchased on installment before the marriage although some
installments were paid during the marriage would be considered conjugal property

Held: Yes. The subject lot which was once owned by PHHC and covered by the Conditional Contract to Sell
was only transferred during the marriage of Bonifacio and Anita. The title to the property was only passed to
Bonifacio after he had fully paid the purchase price on June 22, 1970. This full payment was made more than 2
years after his marriage to Anita on April 24, 1968. In effect, the property was acquired during the existence of
the marriage. Hence, ownership to the property is presumed to belong to the conjugal partnership.
Alfonso v. Office of the President, GR No. 150091, April2, 2007
YOLANDA O. ALFONSO, Petitioner, vs. OFFICE OF THE PRESIDENT and PHIL-VILLE DEVELOPMENT
AND HOUSING CORPORATION, Respondents.

Fact: Petitioner Yolanda O. Alfonso (Alfonso), then the register of deeds of Caloocan City, was found
administratively liable for allegedly ―acquiescing to the change of the date of the registration of OCT No. 994
from May 3, 1917 to April 19, 1917, and for making it appear that there were two OCT Nos. 994. Consequently,
she was dismissed from government service for grave misconduct and dishonesty. Alfonso was investigated by
the Land Registration Authority (LRA) upon the request of Phil- Ville Development Corporation (Phil-Ville)
who purchased some parts of the land. Phil-Ville‘s letter- complaint led to the conduct of an inquiry by the
Senate Committees on Justice and Human Rights, and on Urban Planning, Housing and Resettlement which
finds that Alfonso acted maliciously, fraudulently and in bad faith recommending the filing of administrative
cases against her and her conspirators. On the other hand, LRA finds her guilty of Grave Misconduct and
recommended her dismissal. The Office of the President subsequently dismissed Alfonso. The Court of Appeals
affirmed the dismissal of Alfonso. Hence, this petition contending that her right to due process was violated.

Issue: Whether or not the Court of Appeals erred in upholding decision of the Office of the President
because Alfonso‘s right to due process was violated.

Held: In the landmark case of Ang Tibay v. Court of Industrial Relations, this Court laid down the cardinal
primary requirements of due process in administrative proceedings. Foremost of these requisites is the right to a
hearing, including the right to present one‘s case and submit evidence in support thereof. The essence of due
process in administrative proceedings is the opportunity to explain one‘s side or to seek a reconsideration of the
action or ruling complained of.
As aptly observed by the CA, Alfonso was given every opportunity to explain her side and to present evidence
in her defense during the administrative investigation conducted by the LRA. Records sufficiently show that in
compliance with the ―show-cause letter of the LRA Administrator, she submitted her written explanation, and
that during the pre-trial conferences, she presented documentary evidence.
Likewise, the quantum of proof required in an administrative proceeding is only substantial evidence or that
amount of relevant evidence that a reasonable mind might accept as adequate to support a conclusion. The
standard of substantial evidence is satisfied when there is reasonable ground to believe that the person indicted
Page 24

was responsible for the alleged wrongdoing or misconduct. In the case at bar, Alfonso stood charged not for
changing the date of registration of OCT No. 994 but rather, she was indicted for acquiescing to the change by
(1) issuing conflicting ―certifications‖ on the date of issuance of OCT No. 994; and (2) for making it appear
that there were two OCT Nos. 994. Thus, her protestations that she had no hand in the alteration are unavailing.
―Serious misconduct, as a valid cause for the dismissal of an employee, is improper or wrong conduct; the
transgression of some established and definite rule of action; a forbidden act or dereliction of duty, which is
willful and intentional neglect and not mere error in judgment. It must be grave and aggravated in character and
not merely trivial or unimportant.. In addition, it must be directly related and/or connected to the performance of
official duties. Without question, all of these requisites are present in this case. Alfonso is thus administratively
liable for serious misconduct.
Naawan Community Rural Bank v. Court of Appeals, GR No. 128573, Jan. 13, 2003
NAAWAN COMMUNITY RURAL BANK INC., Petitioner, vs. THE COURT OF APPEALS and SPOUSES
ALFREDO AND ANNABELLE LUMO, Respondents.
Under the established principles of land registration, a person dealing with registered land may generally rely on
the correctness of a certificate of title and the law will in no way oblige him to go beyond it to determine the
legal status of the property.

Facts:
1. On April 30, 1988, a certain Guillermo Comayas offered to sell to private respondent-spouses Alfredo
and Annabelle Lumo, a house and lot measuring located at Pinikitan, Camaman-an, Cagayan de Oro
City.
2. Wanting to buy said house and lot, private respondents made inquiries at the Office of the Register of
Deeds of Cagayan de Oro City where the property is located and the Bureau of Lands on the legal status
of the vendor’s title. They found out that the property was mortgaged for P8,000 to a certain Mrs.
Galupo and that the owner’s copy of the Certificate of Title to said property was in her possession.
3. Private respondents directed Guillermo Comayas to redeem the property from Galupo at their expense,
giving the amount of P10,000 to Comayas for that purpose.
4. On May 30, 1988, a release of the adverse claim of Galupo was annotated on TCT No. T-41499 which
covered the subject property.
5. In the meantime, on May 17, 1988, even before the release of Galupo’s adverse claim, private
respondents and Guillermo Comayas, executed a deed of absolute sale. The subject property was
allegedly sold for P125,000 but the deed of sale reflected the amount of only P30,000 which was the
amount private respondents were ready to pay at the time of the execution of said deed, the balance
payable by installment.
6. On June 9, 1988, the deed of absolute sale was registered and inscribed on TCT No. T-41499 and, on
even date, TCT No. T-50134 was issued in favor of private respondents
7. After obtaining their TCT, private respondents requested the issuance of a new tax declaration certificate
in their names. However, they were surprised to learn from the City Assessor’s Office that the property
was also declared for tax purposes in the name of petitioner Naawan Community Rural Bank Inc.
Records in the City Assessor’s Office revealed that, for the lot covered by TCT No. T-50134, Alfredo
Lumo’s T/D # 83324 bore the note: “This lot is also declared in the name of Naawan Community Rural
Bank Inc. under T/D # 71210”. Apparently, on February 7, 1983, Guillermo Comayas obtained a
P15,000 loan from petitioner Bank using the subject property as security. At the time said contract of
mortgage was entered into, the subject property was then an unregistered parcel of residential land, tax-
declared in the name of a certain Sergio A. Balibay while the residential one-storey house was tax-
declared in the name of Comayas. Balibay executed a special power of attorney authorizing Comayas to
borrow money and use the subject lot as security. But the Deed of Real Estate Mortgage and the Special
Power of Attorney were recorded in the registration book of the Province of Misamis Oriental, not in the
registration book of Cagayan de Oro City. It appears that, when the registration was made, there was
Page 25

only one Register of Deeds for the entire province of Misamis Oriental, including Cagayan de Oro City.
It was only in 1985 when the Office of the Register of Deeds for Cagayan de Oro City was established
separately from the Office of the Register of Deeds for the Province of Misamis Oriental.
For failure of Comayas to pay, the real estate mortgage was foreclosed and the subject property sold at a
public auction to the mortgagee Naawan Community Rural Bank as the highest bidder in the amount of
P16,031.35. Meanwhile, on September 5, 1986, the period for redemption of the foreclosed subject
property lapsed and the MTCC Deputy Sheriff of Cagayan de Oro City issued and delivered to petitioner
bank the sheriff’s deed of final conveyance. This time, the deed was registered under Act 3344 and
recorded in the registration book of the Register of Deeds of Cagayan de Oro City. By virtue of said
deed, petitioner Bank obtained a tax declaration for the subject house and lot.
8. Thereafter, petitioner Bank instituted an action for ejectment against Comayas before the MTCC
which decided in its favor. On appeal, the Regional Trial Court affirmed the decision of the MTCC in a
decision dated April 13, 1988.
9. On January 27, 1989, the Regional Trial Court issued an order for the issuance of a writ of execution
of its judgment. The MTCC, being the court of origin, promptly issued said writ.
10. However, when the writ was served, the property was no longer occupied by Comayas but herein
private respondents, the spouses Lumo who had, as earlier mentioned, bought it from Comayas on May
17, 1988.
11. Alarmed by the prospect of being ejected from their home, private respondents filed an action for
quieting of title. After trial, the Regional Trial Court rendered a decision declaring private respondents as
purchasers for value and in good faith, and consequently declaring them as the absolute owners and
possessors of the subject house and lot.
Issue:
1. WHETHER OR NOT REGISTRATION OF SHERIFF’S DEED OF FINAL CONVEYANCE IN THE
PROPER REGISTRY OF DEEDS IS MORE SUPERIOR THAN THE TORRENS TITLE? NO.
2. WHETHER OR NOT PRIVATE RESPONDENTS COULD BE CONSIDERED AS BUYERS IN GOOD
FAITH? YES.

HELD: Petitioner bank contends that the earlier registration of the sheriff’s deed of final conveyance in the day
book under Act 3344 should prevail over the later registration of private respondents’ deed of absolute sale
under Act 496, as amended by the Property Registration Decree, PD 1529.
1. This contention has no leg to stand on.
2. It has been held that, where a person claims to have superior proprietary rights over another on the
ground that he derived his title from a sheriff’s sale registered in the Registry of Property, Article 1473
(now Article 1544) of the Civil Code will apply only if said execution sale of real estate is registered
under Act 496.
3. Unfortunately, the subject property was still untitled when it was acquired by petitioner bank by virtue
of a final deed of conveyance. On the other hand, when private respondents purchased the same
property, it was already covered by the Torrens System. Petitioner also relies on the case of Bautista vs.
Fule where the Court ruled that the registration of an instrument involving unregistered land in the
Registry of Deeds creates constructive notice and binds third person who may subsequently deal with
the same property.
4. However, a close scrutiny of the records reveals that, at the time of the execution and delivery of the
sheriff’s deed of final conveyance on September 5, 1986, the disputed property was already covered by
the Land Registration Act and Original Certificate of Title No. 0-820 pursuant to Decree No. N189413
was likewise already entered in the registration book of the Register of Deeds of Cagayan De Oro City
as of April 17, 1984.
5. Thus, from April 17, 1984, the subject property was already under the operation of the Torrens System.
Under the said system, registration is the operative act that gives validity to the transfer or creates a lien
upon the land.
Page 26

6. Moreover, the issuance of a certificate of title had the effect of relieving the land of all claims except
those noted thereon.
7. Accordingly, private respondents, in dealing with the subject registered land, were not required by law to
go beyond the register to determine the legal condition of the property. They were only charged with
notice of such burdens on the property as were noted on the register or the certificate of title. To have
required them to do more would have been to defeat the primary object of the Torrens System which is
to make the Torrens Title indefeasible and valid against the whole world.
8. Mere registration of title in case of double sale is not enough; good faith must concur with the
registration.
Petitioner contends that the due and proper registration of the sheriff’s deed of final conveyance on December 2,
1986 amounted to constructive notice to private respondents. Thus, when private respondents bought the subject
property on May 17, 1988, they were deemed to have purchased the said property with the knowledge that it
was already registered in the name of petitioner bank.
1. The “priority in time” principle being invoked by petitioner bank is misplaced because its registration
referred to land not within the Torrens System but under Act 3344.
2. On the other hand, when private respondents bought the subject property, the same was already
registered under the Torrens System. It is a well-known rule in this jurisdiction that persons dealing with
registered land have the legal right to rely on the face of the Torrens Certificate of Title and to dispense
with the need to inquire further, except when the party concerned has actual knowledge of facts and
circumstances that would impel a reasonably cautious man to make such inquiry.
3. Private respondents exercise the required diligence in ascertaining the legal condition of the title to the
subject property so as to be considered as innocent purchasers for value and in good faith
Before private respondents bought the subject property from Guillermo Comayas, inquiries were made with the
Registry of Deeds and the Bureau of Lands regarding the status of the vendor’s title. No liens or encumbrances
were found to have been annotated on the certificate of title. Neither were private respondents aware of any
adverse claim or lien on the property other than the adverse claim of a certain Geneva Galupo to whom
Guillermo Comayas had mortgaged the subject property. But, as already mentioned, the claim of Galupo was
eventually settled and the adverse claim previously annotated on the title cancelled. Thus, having made the
necessary inquiries, private respondents did not have to go beyond the certificate of title. Otherwise, the efficacy
and conclusiveness of the Torrens Certificate of Title would be rendered futile and nugatory.
Considering therefore that private respondents exercised the diligence required by law in ascertaining the legal
status of the Torrens title of Guillermo Comayas over the subject property and found no flaws therein, they
should be considered as innocent purchasers for value and in good faith.
Accordingly, the appealed judgment of the appellate court upholding private respondents Alfredo and Annabelle
Lumo as the true and rightful owners of the disputed property is affirmed.

CHAPTER 3: Original Registration (Ordinary Registration Proceedings)

A1. Applications: registration under PD No. 1529 (Property Registration Decree)


Malabanan v. Republic, GR No. 179987, April 29, 2009; Resolution on motion for reconsideration, Sept.
3, 2013
HEIRS OF MARIO MALABANAN, (Represented by Sally A. Malabanan), Petitioners, vs. REPUBLIC OF
THE PHILIPPINES, Respondent.

Facts:This case is a resolution of the Motions for Reconsideration, filed by the parties who both assail the
decision of the Court promulgated on April 29, 2009. In the decision, the Court upheld the ruling of the Court of
Appeals which denied the application of the petitioners for the registration of a parcel of land situated in
Page 27

Barangay Tibig, Silang, Cavite, on the ground that they had not established by sufficient evidence their
right to the registration in accordance with either section 14 (1) or 14 (2) of the Property Registration
Decree (P.D. 1529).
On February 20, 1998, applicant Mario Malabanan, who had purchased the property from Eduardo Velazco,
filed an application for land registration covering the property in the RTC in Tagaytay City, Cavite, claiming
that the property formed part of the alienable and disposable land of the public domain, and that he and his
predecessors-in-interest had been in open, continuous, uninterrupted, public and adverse possession and
occupation of the land for more than 30 years, thereby entitling him to the judicial confirmation of his title. To
prove such, Malabanan presented during trial a certification issued by the Community Environment and Natural
Resources Office (CENRO) of the DENR. The RTC rendered judgment granting Malabanan’s application for
land registration. The Office of the Solicitor General (OSG) appealed the judgment to the CA, arguing that
Malabanan had failed to prove that the property belonged to the alienable and disposable land of the public
domain, and that the RTC erred in finding that he had been in possession of the property in the manner and for
the length of time required by law for confirmation of imperfect title. The CA promulgated its decision
reversing the RTC and dismissing the application for registration of Malabanan. Citing the ruling in Republic v.
Herbieto (Herbieto), the CA declared that under Section 14(1) of the Property Registration Decree, any period
of possession prior to the classification of the land as alienable and disposable was inconsequential and should
be excluded from the computation of the period of possession. Noting that the CENRO-DENR certification
stated that the property had been declared alienable and disposable only on March 15, 1982, Velazco’s
possession prior to March 15, 1982 could not be tacked for purposes of computing Malabanan’s period of
possession.

Issue:
1) What are the classifications of public lands?
2) Whether or not petitioners were able to prove that the property was an alienable and disposable
land of the public domain.

Ruling:
1) Classifications of land according to ownership.
Land, which is an immovable property, may be classified as either of public dominion or of private
ownership. Land is considered of public dominion if it either:
a) is intended for public use; or
b) belongs to the State, without being for public use, and is intended for some public service or for the
development of the national wealth.
Land belonging to the State that is not of such character, or although of such character but no longer intended
for public use or for public service forms part of the patrimonial property of the State. Land that is other than
part of the patrimonial property of the State, provinces, cities and municipalities is of private ownership if it
belongs to a private individual.
Pursuant to the Regalian Doctrine (Jura Regalia), a legal concept first introduced into the country from the West
by Spain through the Laws of the Indies and the Royal Cedulas, all lands of the public domain belong to the
State. This means that the State is the source of any asserted right to ownership of land, and is charged with the
conservation of such patrimony.
All lands not appearing to be clearly under private ownership are presumed to belong to the State. Also, public
lands remain part of the inalienable land of the public domain unless the State is shown to have reclassified or
alienated them to private persons.
A positive act of the Government is necessary to enable such reclassification, and the exclusive prerogative to
classify public lands under existing laws is vested in the Executive Department, not in the courts. If, however,
public land will be classified as neither agricultural, forest or timber, mineral or national park, or when public
land is no longer intended for public service or for the development of the national wealth, thereby effectively
removing the land from the ambit of public dominion, a declaration of such conversion must be made in the
Page 28

form of a law duly enacted by Congress or by a Presidential proclamation in cases where the President is duly
authorized by law to that effect. Thus, until the Executive Department exercises its prerogative to classify or
reclassify lands, or until Congress or the President declares that the State no longer intends the land to be used
for public service or for the development of national wealth, the Regalian Doctrine is applicable.
2) Petitioners failed to present sufficient evidence to establish that they and their predecessors-in-interest had
been in possession of the land since June 12, 1945. Without satisfying the requisite character and period of
possession – possession and occupation that is open, continuous, exclusive, and notorious since June 12,
1945, or earlier – the land cannot be considered ipso jure converted to private property even upon the
subsequent declaration of it as alienable and disposable.

Prescription never began to run against the State, such that the land has remained ineligible for registration
under Section 14(1) of the Property Registration Decree. Likewise, the land continues to be ineligible for
land registration under Section 14(2) of the Property Registration Decree unless Congress enacts a law or the
President issues a proclamation declaring the land as no longer intended for public service or for the
development of the national wealth.
Dumo v. Republic, GR No. 218269, June 6, 2018
IN RE: APPLICATION FOR LAND REGISTRATION
SUPREMA T. DUMO, Petitioner, v. REPUBLIC OF THE PHILIPPINES, Respondent.

Facts:
Petitioner Suprema T. Dumo filed an application for registration of two parcels of land, covered by Advance
Plan of Lot Nos. 400398 and 400399 with a total area of 1,273 square meters (LRC Case No. 270-Bg). Dumo
alleged that the lots belonged to her mother Bernarda M. Trinidad, and that she and her siblings inherited them
upon their mother's death. She further alleged that through a Deed of Partition with Absolute Sale dated 6
February 1987, she acquired the subject lots from her siblings. Dumo traces her title from her mother, Trinidad,
who purchased the lots from Florencio Mabalay in August 1951. Mabalay was Dumo's maternal grandfather.
Mabalay, on the other hand, purchased the properties from Carlos Calica.
The heirs of Marcelino Espinas opposed Dumo's application for land registration on the ground that the
properties sought to be registered by Dumo are involved in the accion reivindicatoria case. Thus, the RTC
consolidated the land registration case with the Complaint for Recovery of Ownership, Possession and
Damages.
On 2 July 2010, the RTC rendered its Joint Decision, finding that the subject property was owned by the heirs of
Espinas. The RTC ordered the dismissal of Dumo's land registration application on the ground of lack of
registerable title, and ordered Dumo to restore ownership and possession of the lots to the heirs of Espinas.
The CA rendered its Decision dated 28 January 2014, affirming the RTC's decision dismissing the application
for land registration of Dumo, and finding that she failed to demonstrate that she and her predecessors-in¬
interest possessed the property in the manner required by law to merit the grant of her application for land
registration. The CA, however, modified the decision of the RTC insofar as it found that the Subject Property
belonged to the heirs of Espinas. The CA found that since the property still belonged to the public domain, and
the heirs of Espinas were not able to establish their open, continuous, exclusive and notorious possession and
occupation of the land under a bona fide claim of ownership since 12 June 1945 or earlier, it was erroneous for
the RTC to declare the heirs of Espinas as the owners of the Subject Property; hence, this petition.

Issues:
1. Whether Dumo is able to prove that the subject property forms part of the alienable and disposable land
of public domain
2. Whether the requirement that documents to prove the status of land shall be based on the land
classification approved by the DENR
Page 29

Secretary is not a mere superfluity.

Ruling:
1. NO, Dumo failed to submit any of the documents required to prove that the land she seeks to register is
alienable and disposable land of the public domain.

The applicant bears the burden of proving the status of the land. In this connection, the Court held that
there are two (2) documents which must be presented: first, a copy of the original classification
approved by the Secretary of the DENR and certified as a true copy by the legal custodian of the official
records, and second, a certificate of land classification status issued by the CENRO or the PENRO based
on the land classification approved by the DENR Secretary.

In this case, none of the documents submitted by respondent to the trial court indicated that the subject
property was agricultural or part of the alienable and disposable lands of the public domain. At most, the
CENRO Report and Certification stated that the land was not covered by any kind of public land
application. This was far from an adequate proof of the classification of the land.

Unfortunately for respondent, the evidence submitted clearly falls short of the requirements for original
registration in order to show the alienable character of the lands subject herein

2. YES, the requirement that documents to prove the status of land shall be based on the land classification
approved by the DENR Secretary is not a mere superfluity.

This requirement stems from the fact that the alienable and disposable classification of agricultural land may be
made by the President or DENR Secretary. And while the DENR Secretary may perform this act in the regular
course of business, this does not extend to the CENRO or PENRO – the DENR Secretary may no longer
delegate the power to issue such certification as the power to classify lands of the public domain as alienable
and disposable lands is in itself a delegated power under CA No. 141 and PD No. 705.
Republic v. Alejandre, GRNo. 217336, Oct. 17, 2018
REPUBLIC OF THE PHILIPPINES, PETITIONER, VS. SPS. ILDEFONSO ALEJANDRE AND ZENAIDA
FERRER ALEJANDRE, RESPONDENTS.

DECISION
CAGUIOA, J:

Before the Court is a petition for review on certiorari[1] (Petition) under Rule 45 of the Rules of Court (Rules)
assailing the Decision[2] dated February 27, 2015 (Decision) of the Court of Appeals[3] (CA) in CA-G.R. CV
No. 101259, which sustained the Amended Decision[4] dated June 12, 2008 of the Regional Trial Court of
Bangued, Abra, Branch 2 (RTC) in LRC Case No. N-20, which granted the respondents' application for
registration of Lot 6487, Cad. 536, Ap-CAR-000007, with an area of 256 square meters located at Barrio
Poblacion, Municipality of Bangued, Province of Abra.

The Facts
The CA Decision narrates the antecedents as follows:
On July 18, 1991, Spouses Alejandre (applicants-spouses, for brevity) filed an application for the registration of
Lot No. 6487 under P.D. No. 1529, described in plan Ap-CAR-000007, Cad-536, with an area of 256 square
meters. They alleged that they are the owners of the subject property by virtue of a deed of sale or conveyance;
that the subject property was sold to them by its former owner Angustia Lizardo Taleon by way of a Deed of
Absolute Sale executed on June 20, 1990; that the said land is presently occupied by the applicants-spouses.
Page 30

On September 16, 1991, the Office of the Solicitor General, as counsel for the Republic, entered its appearance.
On November 12, 1991, the Land Registration Authority (LRA, for brevity) submitted a Report noting that
there were discrepancies in the plan submitted by the applicant spouses, which discrepancies were referred to
the Lands Management Sector for verification and correction.

On January 30, 1992, the trial court issued an order of general default and allowed the applicants-spouses to
present their evidence.

On July 20, 1992, the trial court granted the applicant spouses' motion to submit original tracing cloth plan and
technical description for purposes of facilitating the approval of the re-surveyed plans as well as the submission
of the new plan for the scrutiny and approval of the LRA.

On August 10, 1992, the applicants-spouses filed their Formal Offer of Evidence. On April 26, 1993, they
submitted the corrected advance plan and technical description to the trial court.

On August 20, 1993, the LRA submitted its Supplementary Report stating that the "polygon does not close"
even after the corrections effected on the bearings and distances of the technical description were made. Hence,
the LRA requested for reverification and correction.

In an Order dated December 10, 1997, the trial court deemed the case submitted for decision.
Subsequently, or on April 15, 1998, the LRA submitted its Final Report stating that it applied the corrected
technical description of the subject lot and no more discrepancy exists, however, the area was increased by six
(6) meters. As such, on August 24, 1998, the trial court ordered the submission of publication of the amended or
new technical description. On May 6, 2000, the trial court issued another Notice setting the case for Initial
Hearing on July 25, 2000.

On June 1, 2000, the Republic filed its Opposition to the application based on the following grounds: (1) that
neither the applicants nor their predecessors-in-interest have been in open, continuous, exclusive and notorious
possession and occupation of the land in question since June 12, 1945 or earlier as required by Section 48 (b) of
Commonwealth Act No. 141 (CA 141), x x x as amended by Presidential Decree No. 1073 (PD 1073); (2) that
applicants failed to adduce any muniment of title and/or the tax declarations with the petition to evidence bona
fide acquisition of the land applied for or of its open, continuous, exclusive and notorious possession and
occupation thereof in the concept of an owner since 12 June 1945 or earlier; that the tax declaration adverted to
in the petition does not appear to be genuine and the tax declaration indicates pretended possession of applicants
to be of recent vintage[;] and (3) that the subject property applied for is a portion of the public domain
belonging to the Republic of the Philippines which is not subject to private appropriation.
After trial, the trial court rendered its Decision dated March 31, 2006 granting the application for registration of
title, the dispositive portion of which reads:

"WHEREFORE, premises considered, the Court finds the application to be well-taken and the same is hereby
granted.

Let a copy of this decision be furnished the Land Registration Authority, Office of the Solicitor General and
Bureau of Lands.
SO ORDERED."

On June 12, 2008, the trial court issued the Amended Decision which increased the area subject for land
registration to two hundred sixty-two square meters (262 sqm) from two hundred fifty-six square meters (256
sqm) from the original decision.
Page 31

Disagreeing with the trial court's grant of the application for land registration, the Republic interposed [an]
appeal [to the CA].[5]
Ruling of the CA

The CA in its Decision[6] dated February 27, 2015 denied the appeal of the Republic. The dispositive portion
thereof states:
WHEREFORE, premises considered, the present appeal is DENIED. Accordingly, the Amended Decision of the
Regional Trial Court of Bangued, Abra, Branch 2, is SUSTAINED.
SO ORDERED.[7]

The CA justified that based on the allegations of the applicants spouses Ildefonso Alejandre and Zenaida Ferrer
Alejandre (respondents) in their application for land registration and subsequent pleadings, they come under
paragraph 4 of Section 14, Presidential Decree No. (PD) 1529[8] - those who have acquired ownership of lands
in any manner provided for by law - because they acquired the land in question by virtue of a Deed of Absolute
Sale executed on June 20, 1990[9] from Angustia Alejandre Taleon who acquired the land from her mother by
inheritance.[10]

The Republic filed the instant Petition without filing a motion for reconsideration with the CA on the ground
that the CA decided the Republic's appeal in gross disregard of the law and in a manner not in accordance with
the applicable decisions of the Court.[11]
Respondents filed their "Comment and Compliance"[12] dated July 18, 2016. The Republic filed a Reply[13]
dated March 3, 2017.

The Issue
The Petition raises this sole issue: whether the CA seriously misappreciated the facts as well as made findings
which are inconsistent with, or not supported by, the evidence on record; and gravely misapplied the applicable
laws and jurisprudence.[14]

The Court's Ruling


The Petition is impressed with merit.
The RTC Amended Decision justified the granting of the application for land registration under the Property
Registration Decree (PD 1529) on these factual findings:
It appears from the evidence presented that the applicants acquired the property sought to be registered by
means of a Deed of Absolute Sale [dated June 20, 1990 (Exh. "K" to "K5")] executed by Angustia Alejandre
Taleon as vendor in favor of the petitioners spouses Ildefonso Alejandre and Zenaida F. Alejandre as vendees.
Said property was previously inherited by the vendor from her late mother Angustia Alejandre who inherited the
same property from Don Santiago Alejandre, the grandfather of the applicant Dr. Ildefonso Alejandre.[15]
The CA sustained the RTC Amended Decision in this wise:
Under Section 14 of PD No. 1529, there are four (4) types of applicants who may apply for registration of title
to land[,] viz[.]:
Section 14. Who may apply. The following persons may file in the proper Court of First Instance an application
for registration of title to land, whether personally or through their duly authorized representatives:
(1) Those who by themselves or through their predecessors-in-interest have been in open, continuous, exclusive
and notorious possession and occupation of alienable and disposable lands of the public domain under a bona
fide claim of ownership since June 12, 1945, or earlier.
(2) Those who have acquired ownership of private lands by prescription under the provision of existing laws.
(3) Those who have acquired ownership of private lands or abandoned river beds by right of accession or
accretion under the existing laws.
(4) Those who have acquired ownership of land in any other manner provided for by law. (Italics and Emphasis
Ours)
Page 32

In the case at bar, basing from the allegations of the applicants spouses in their application for land registration
and subsequent pleadings, clearly, they come under Paragraph 4 of the quoted section and not under Paragraph
1 of the same section. It is undisputed that they acquired the land in question by virtue of a Deed of Absolute
Sale executed on June 20, 1990 from Angustia Alejandre Taleon who acquired the land from her mother by
inheritance. In other words, the applicant spouses acquired ownership over Lot 6487 through a contract of sale,
which is well within the purview of Paragraph 4 of Section 14 of P.D. No. 1529.
As a consequence, the requirement of open, continuous, exclusive and notorious possession and/or occupation
in the concept of an owner has no application in the case at bar. Not even the requirement that the land applied
for should have been declared disposable and alienable applies considering that this is just one of the requisites
to be proven when applicants for land registration fall under Paragraph 1 of Section 14 of P.D. No. 1529, which
is not the case at bar.[16]
The Republic argues that under the law, citing Section 24 of PD 1529 and Section 48(b) of Commonwealth Act
No. 141,[17] as amended by Section 4 of PD 1073,[18] before an applicant can register his title over a particular
parcel of land, he must show that: (a) he, by himself or through his predecessors-in-interest, has been in open,
continuous, exclusive and notorious possession and occupation of the subject land under a bona fide claim of
ownership since June 12, 1945, or earlier; and (b) the subject land falls within the alienable and disposable
portion of the public domain.[19]
The Republic also argues, citing Republic v. Sayo,[20] Director of Lands v. IAC[21] and Director of Lands v.
Aquino,[22] that in land registration proceedings, the applicant has the burden of overcoming the presumption
that the land sought to be registered belongs to the public domain or the presumption of State ownership of the
lands of the public domain.[23]
Citing Bracewell v. Court of Appeals,[24] the Republic further posits that to prove that the subject land is
alienable, the applicant must establish the existence of a positive act of the government, such as a presidential
proclamation or an executive order, an administrative action, investigation reports of Bureau of Land
investigators, and a legislative act or a statute, declaring the land as already alienable and disposable.[25]

Pursuant to Article 419 of the Civil Code, property, in relation to the person to whom it belongs, is either of
public dominion or of private ownership. As such, properties are owned either in a public capacity (dominio
publico) or in a private capacity (propiedad privado).[26]
There are three kinds of property of public dominion: (1) those intended for public use; (2) those intended for
some public service; and (3) those intended for the development of national wealth. This is provided in Article
420 of the Civil Code, to wit:
ART. 420. The following things are property of public dominion:
(1) Those intended for public use, such as roads, canals, rivers, torrents, ports and bridges constructed by the
State, banks, shores, roadsteads, and others of similar character;
(2) Those which belong to the State, without being for public use, and are intended for some public service or
for the development of the national wealth.
With respect to provinces, cities and municipalities or local government units (LGUs), property for public use
"consist of the provincial roads, city streets, municipal streets, the squares, fountains, public waters,
promenades, and public works for public service paid for by said provinces, cities, or municipalities."[27]
In turn, the Civil Code classifies property of private ownership into three categories: (1) patrimonial property of
the State under Articles 421 and 422; (2) patrimonial property of LGUs under Article 424; and (3) property
belonging to private individuals under Article 425, hence:
ART. 421. All other property of the State, which is not of the character stated in the preceding article, is
patrimonial property.
ART. 422. Property of public dominion, when no longer intended for public use or for public service, shall form
part of the patrimonial property of the State.
xxxx
Page 33

ART. 424. Property for public use, in the provinces, cities, and municipalities, consist of the provincial roads,
city streets, municipal streets, the squares, fountains, public waters, promenades, and public works for public
service paid for by said provinces, cities, or municipalities.
All other property possessed by any of them is patrimonial and shall be governed by this Code, without
prejudice to the provisions of special laws.
ART. 425. Property of private ownership, besides the patrimonial property of the State, provinces, cities, and
municipalities, consists of all property belonging to private persons, either individually or collectively.
From the foregoing, property of private ownership or patrimonial property of the State may be sub-classified
into:

(1) "By nature or use" or those covered by Article 421, which are not property of public dominion or imbued
with public purpose based on the State's current or intended use; and
(2) "By conversion" or those covered by Article 422, which previously assumed the nature of property of public
dominion by virtue of the State's use, but which are no longer being used or intended for said purpose. Since
those properties could only come from property of public dominion as defined under Article 420, "converted"
patrimonial property of the State are separate from and not a subset of patrimonial property "by nature or use"
under Article 421.
With respect to lands, which are immovable property pursuant to Article 415(1) of the Civil Code, they can
either be lands of public dominion or of private ownership following the general classification of property under
Article 419.
Section 3, Article XII of the 1987 Constitution, which embodies the Regalian doctrine, classifies lands of the
public domain into five categories - agricultural lands, forest lands, timber lands, mineral lands, and national
parks. The provision states:
SEC. 3. Lands of the public domain are classified into agricultural, forest or timber, mineral lands, and national
parks. Agricultural lands of the public domain may be further classified by law according to the uses to which
they may be devoted. Alienable lands of the public domain shall be limited to agricultural lands. x x x
(Emphasis supplied)
Section 3 mandates that only lands classified as agricultural may be declared alienable, and thus susceptible of
private ownership. As the connotative term suggests, the conversion of land of the public domain into alienable
and disposable opens the latter to private ownership.[28] At that point (i.e., upon the declaration of alienability
and disposability), the land ceases to possess the characteristics inherent in properties of public dominion that
they are outside the commerce of man, cannot be acquired by prescription, and cannot be registered under the
land registration law,[29] and accordingly assume the nature of patrimonial property of the State that is property
owned by the State in its private capacity.
As noted by Justice Edgardo L. Paras:
It is believed that forest and mining lands are properties of public dominion of the third class, i.e., properties for
the development of the national wealth. Upon the other hand, the public agricultural lands before being made
available to the general public should also be properties of public dominion for the development of the national
wealth (and as such may not be acquired by prescription); but after being made so available, they become
patrimonial property of the State, and therefore subject to prescription. Moreover, once already acquired by
private individuals, they become private property. x x x[30] (Emphasis and underscoring supplied)
Thus, it can be gathered from the foregoing that the subject of the land registration application under Section 14
of PD 1529 is either alienable and disposable land of public domain or private land. While Section 14(4) does
not describe or identify the kind of land unlike in (1), which refer to "alienable and disposable lands of the
public domain;" (2), which refer to "private lands"; and (3) "private lands or abandoned river beds," the land
covered by (4) cannot be other than alienable and disposable land of public domain, i.e., public agricultural
lands[31] and private lands or lands of private ownership in the context of Article 435.

This premise proceeds from the well-entrenched rule that all lands not appearing to be clearly of private
dominion or ownership presumptively belong to the State.[32] Accordingly, public lands not shown to have
Page 34

been classified, reclassified or released as alienable agricultural land or alienated to a private person by the State
remain part of the inalienable lands of public domain.[33] Therefore, the onus to overturn, by incontrovertible
evidence, the presumption that the land subject of an application for registration is alienable and disposable
rests with the applicant.[34]
Respondents, based on the evidence that they adduced, are apparently claiming ownership over the land subject
of their application for registration by virtue of tradition, as a consequence of the contract of sale, and by
succession in so far as their predecessors-in-interest are concerned. Both modes are derivative modes of
acquiring ownership. Yet, they failed to prove the nature or classification of the land. The fact that they acquired
the same by sale and their transferor by succession is not incontrovertible proof that it is of private dominion or
ownership. In the absence of such incontrovertible proof of private ownership, the well-entrenched presumption
arising from the Regalian doctrine that the subject land is of public domain or dominion must be overcome.
Respondents failed to do this.
The real property tax declarations (Exhibits "L" and "M"), the Deed of Absolute Sale dated June 20, 1990
(Exhibit "K" to "K5"), and the technical descriptions of the subject property (Exhibit "J") are insufficient
evidence to overcome the presumption that the land subject of the registration is inalienable land of public
domain or dominion. Thus, respondents' application for land registration should not have been granted.
WHEREFORE, the Petition is hereby GRANTED. The Decision dated February 27, 2015 of the Court of
Appeals in CA-G.R. CV No. 101259 and the Amended Decision dated June 12, 2008 of the Regional Trial
Court of Bangued, Abra, Branch 2 in LRC Case No. N-20 are REVERSED and SET ASIDE. Respondents'
application for registration in LRC Case No. N-20 is DISMISSED without prejudice.
SO ORDERED.
Republic v. Fabio, GRNo. 159589, Dec.23, 2008
REPUBLIC OF THE PHILIPPINES, Petitioner, v. HEIRS OF JUAN FABIO, namely: DOMINGA C. FABIO,
SOCORRO D. FABIO, LYDIA D. FABIO, ROLANDO D. FABIO, NORMA D. FABIO, NORMA L. FABIO,
ANGELITA FABIO, ROSALIE FABIO, DANILO FABIO, RENATO FABIO, LEVITA FABIO, IRENE
FABIO, TERESITA MOLERA, ROSEMARIE C. PAKAY, LIGAYA C. MASANGKAY, ALFREDO F.
CASTILLO, MELINDA F. CASTILLO, MERCEDITA F. CASTILLO, ESTELA DE JESUS AQUINO,
FELECITO FABIO, and ALEXANDER FABIO, represented herein by ANGELITA F. ESTEIBAR as their
Attorney-in-Fact, Respondent.

Facts: petition for review on certiorari


The respondents sought the registration of title under the provisions of Act No. 496 or the Land Registration
Act, as amended by Presidential
Decree No. 1529 (PD 1529)

In the application, respondents alleged that they are the owners of the Lot, including all the improvements,
having acquired the same through a bona fide claim of ownership. They declared that they and their
predecessors-in-interest were in open, continuous, exclusive and... notorious possession of the Lot in the
concept of an owner for more than 100 years
Esteibar, the duly appointed representative of the heirs of Juan Fabio, testified that her grandfather, Juan, died in
1959 when she was only 13 years old. She attested that she was born on the Lot and knows that her grandfather
owned, possessed and occupied the Lot until his... death
Further, he confirmed that there is a notation at the left hand footnote of the approved survey plan which reads
"this survey falls within the Calumpang Point Naval Reservation and disposition hereof shall be subject to the
final... delimitation thereof as per Proc. No. 1582-A dated September 6, 1976."
Dominga Fabio Lozano, the only living and youngest child of Juan Fabio and who was then 63 years of age,
testified that she was born in 1934 in Calumpang, Ternate, Cavite
Mariano Huerto, a helper of the late Juan Fabio, testified that since 1935, when he was only 12 years old, he had
helped cultivate the Lot until he left the place in 1955
Page 35

Raymundo Pakay, 70 years of age at the time and a resident of Ternate, Cavite, testified that he knew Juan Fabio
as the owner of the Lot,... The Republic of the Philippines (petitioner), through the Office of the Solicitor
General, filed an appeal with the Court of Appeals
To prove that the Lot is alienable and disposable land of the public domain, respondents presented in evidence a
letter[33] dated 22 April 1991 of Conlu, a Land Management Inspector of the DENR-Region IV. The relevant
portion of the letter states:
That the land is within alienable and disposable zone under Project No. 22-B, L.C. Map No. 3091;

Issues: whether the respondents have acquired a right over the Lot

Ruling:
On 29 September 1997, the trial court rendered a Decision ordering the registration of the Lot in the name of
Juan Fabio
On 29 August 2003, the Court of Appeals affirmed the ruling of the trial court
Public lands not shown to have been classified as alienable and disposable land remain part of the inalienable
public domain.[41] In view of the lack of sufficient evidence showing that the Lot was already classified as
alienable and disposable, the Lot... applied for by respondents is inalienable land of the public domain, not
subject to registration under Section 14(1) of PD 1529 and Section 48(b) of CA 141, as amended by PD 1073.
Hence, there is no need to discuss the other requisites dealing with respondents' occupation and... possession of
the Lot in the concept of an owner.

While it is an acknowledged policy of the State to promote the distribution of alienable public lands to spur
economic growth and in line with the ideal of social justice, the law imposes stringent safeguards upon the grant
of such resources lest they fall into the wrong hands... to the prejudice of the national patrimony.[42] We must
not, therefore, relax the stringent safeguards relative to the registration of imperfect titles.[43]
In Republic v. Estonilo,[44] we ruled that persons claiming the protection of "private rights" in order to exclude
their lands from military reservations must show by clear and convincing evidence that the properties in
question have been acquired by... a legal method of acquiring public lands. Here, respondents failed to do so,
and are thus not entitled to have the Lot registered in their names. Clearly, both the trial and appellate courts
gravely erred in granting respondents' application for registration of title.

Principles:
At the time the application for registration of title was filed, the Lot was no... longer open to private ownership
as it had been classified as a military reservation for public service.
Thus, respondents are not entitled to have the Lot registered under the Torrens system.
The proclamations established that as early as 1904 a certain parcel of land was placed under the exclusive use
of the government for military purposes by the then colonial American government.
There is no question that the Lot is situated within a military reservation.
The only issue to be resolved is whether the respondents are entitled to have the Lot registered under the
Torrens systems based on the limitation clause cited in the proclamations
No public land can be acquired by private... persons through any other means, and it is indispensable that the
person claiming title to public land should show that his title was acquired through purchase or grant from the
State, or through any other mode of acquisition recognized by law
This letter-certification is insufficient. Conlu is merely a land investigator of the DENR. It is not enough that he
alone should certify that the Lot is within the alienable and disposable zone. Under Section 6 of the Public Land
Act, the prerogative of classifying or... reclassifying lands of the public domain belongs to the President
The President, through a presidential proclamation or executive order, can classify or reclassify a land to be
included or excluded from the public domain. The DENR Secretary is the only... other public official
empowered by law to approve a land classification and declare such land as alienable and disposable... this
letter was the only evidence presented by respondents to prove that the Lot is alienable and disposable... not
Page 36

even the Community Environment and Natural Resources Office (CENRO) certified as correct the investigation
report of the Land Management
Inspector.
In Republic v. T.A.N. Properties, Inc.,[37] we ruled that it is not enough for the Provincial

Environment and Natural Resources Office (PENRO) or CENRO to certify that a land is alienable and
disposable. The applicant for land registration must prove that the DENR Secretary had approved the land
classification and released the land of the public domain as alienable and... disposable, and that the land subject
of the application for registration falls within the approved area per verification through survey by the PENRO
or CENRO. In addition, the applicant must present a copy of the original classification of the land into alienable
and... disposable, as declared by the DENR Secretary, or as proclaimed by the President. Such copy of the
DENR Secretary's declaration or the President's proclamation must be certified as a true copy by the legal
custodian of such official record. These facts must be established to... prove that the land is alienable and
disposable.

Further, the burden is on respondents to prove that the Lot ceased to have the status of a military reservation or
other inalienable land of the public domain. No proof was ever submitted by respondents that the Calumpang
Point Naval Reservation, or the Lot, ceased as a military... reservation. Even if its ownership and control had
been transferred by the Americans to the Philippine government, the Calumpang Point Naval Reservation
remained as an official military reservation.

Thus, being a military reservation at the time, the Calumpang Point Naval
Reservation, to which the Lot is a part of, can not be subject to occupation, entry or settlement.[39] This is clear
from Sections 83 and 88 of CA 141, which provide:

SECTION 83. Upon the recommendation of the Secretary of Agriculture and Commerce, the President may
designate by proclamation any tract or tracts of land of the public domain as reservations for the use of the
Commonwealth of the Philippines or of any of its... branches, or of the inhabitants thereof, in accordance with
regulations prescribed for this purpose, or for quasi-public uses or purposes when the public interest requires it,
including reservations for highways, rights of way for railroads, hydraulic power sites, irrigation... systems,
communal pastures or leguas comunales, public parks, public quarries, public fishponds, working-men's village
and other improvements for the public benefit.

SECTION 88. The tract or tracts of land reserved under the provisions of section eighty-three shall be non-
alienable and shall not be subject to occupation, entry, sale, lease, or other disposition until again declared
alienable under the provision of this Act or by... proclamation of the President.

Well-entrenched is the rule that unless a land is reclassified and declared alienable and disposable, occupation in
the concept of an owner, no matter how long, cannot ripen into ownership and be registered as a title.[40]
Consequently, respondents could... not have occupied the Lot in the concept of an owner in 1947 and
subsequent years when respondents declared the Lot for taxation purposes, or even earlier when respondents'
predecessors-in-interest possessed the Lot, because the Lot was considered inalienable from the time of... its
declaration as a military reservation in 1904. Therefore, respondents failed to prove, by clear and convincing
evidence, that the Lot is alienable and disposable.
Republic v. T.A.N. Properties, Inc., GR No. 154953, June 26, 2008
REPUBLIC OF THE PHILIPPINES, Petitioner, v. T.A.N. PROPERTIES, INC., Respondent.

Facts
Page 37

1. T.A.N. Properties, Inc. (TAN) filed for an application for Original Registration of Title of a portion (Lot
10705-B) of Lot 10705. When the trial court called the case for initial hearing, after notice of such hearing
was published in the Official Gazette and People’s Journal Taliba, and was posted on the bulletin board of
the Municipal Building and in the land, there was no oppositor other than the Republic of the Philippines
represented by the Director of Lands (DIR).
2. During the hearings, TAN presented three witnesses whose testimonies showed that Prospero Dimayuga
(Kabesang Puroy) had peaceful, adverse, open, and continuous possession of the land in the concept of an
owner since 1942. Upon his death, Antonio Dimayuga (Antonio), his son, succeeded him. On 27 Sept 1960,
Antonio executed a Deed of Donation covering the land in favor of one of his children, Fortunato Dimayuga
(Fortunato).
3. Later, however, Antonio gave Fortunato another piece of land. The land in question was adjudicated to one
of Antonio’s children, Prospero Dimayuga (PORTING). On 8 Aug. 1997, Porting sold the land to TAN.
Summary: Kabesang Puroy -> Antonio -> Porting -> TAN
4. RTC: In favor of TAN.
Republic: Appealed. It alleged that RTC was wrong in granting the application for registration absent clear
evidence that the applicant (TAN) and its predecessors-in-interest have complied with the period of
possession and occupation as required by law.
CA: Affirmed RTC.

Issues:
1. Whether the land is alienable and disposable – NO.
2. Whether TAN or its predecessors-in-interest had open, continuous, exclusive, and notorios possession and
occupation of the land in the concept of an owner since June 1945 or earlier (for at least 30 yrs) – NONE.
3. Whether TAN (a private corporation) is qualified to apply for registration of a public land under the Public
Land Act – NO.

Held:
First Issue: No.
 The rule is that all lands not appearing to be clearly of private dominion presumably belong to the State.
Anyone who applies for registration has the burden of overcoming otherwise.
 In this case, TAN submitted two certifications issued by DENR.
a) Certification by the CENRO (Community Environment and Natural Resources Offices) which certified
the subject land as within the Alienable and Disposable Zone under Project 30.
b) The second certification in the form of a memorandum to the trial court issued by FMS-DENR (Forest
Management Services of the DENR) which stated that the subect area falls within an alienable and
disposable land.
 The certifications, however, are not sufficient. Why?
a) CENRO – issues certificates of land classification status for areas below 50 hectares. (It is the PENRO
that issues certificates for lands covering 50 hectares). Here, the area of the subject land covers over 50
hectares (564,007 square meters). The CENRO certificate covered the entire Lot 10705 with an area of
596,116 square meters which is beyond the authority of the CENRO to certify as alienable and
disposable.
b) FMS-DENR – has no authority under DAO (DENR-Admin Order) to issue certificates of land
classification.
 Moreover, it is not enough for the PENRO or CENRO to certify that a land is alienable and disposable. The
applicant for land registration must prove that (1) the DENR Secretary had approved the land classification
and (2) released the land of the public domain as alienable and disposable, and that (3) the land subject of
the application for registration falls within the approved area per verification through survey by the
PENRO or CENRO.
Page 38

In addition, the applicant for land registration must (4) present a copy of the original classification
approved by the DENR Secretary and certified as a true copy by the legal custodian of the official records.
These facts must be established to prove that the land is alienable and disposable. TAN failed to do so.
 Furthermore, CENRO and FMS-DENR certifications are not public documents. The certifications are not
the certified copies or authenticated reproductions of original official records in the legal custody of a
government office. They are conclusions unsupported by adequate proof, hence, no probative value.
Second issue: None.
 The trial court relied on the testimoies of Evangelista (72-year old resident near the subject land) and
Torres (TAN’s Operations Manager).
a) Evangelista – testified that Kabesang Puroy had been in possession of the land before 1945. Yet,
Evangelista only worked on the land for 3 yrs.
Also, Evangelista testified that Kabesang Puroy was succeeded by Fortunato. Take note, they are
neighbors. However, he admitted that he did not know the exact relationship between Kabesang
Puroy and Fortunato, which is rather unusual for neighbors in a small community.
He did not also know the relationship between Fortunato and Porting. In fact, Evangelista’s
testimony is contrary to the factual finding of the trial court that Kabesang Puroy was succeeded
by Antonio, and not Fortunato.
The Court find’s Evangelista’s uncorroborated testimony insufficient to prove that TAN’s
predecessors-in-interest had been in possession of the land in the concept of an owner for more
than 30 years.
b) Torres – while he claimed to be related to the Dimayugas, his knowledge of their possession of
the land was hearsay. He did not even tell the trial court where he obtained his information.
 Moreover, the tax declarations presented were only for the years starting 1955. TAN did not present any
credible explanation why the realty taxes were only paid starting 1955 considering the claim that the
Dimayugas were allegedly in possession of the land before 1945. The payment of the realty taxes
starting 1955 gives rise to the presumption that the Dimayugas claimed ownership or possession of the
land only in that year.
Third issue: No.
 The 1987 Constitution (Art. XII, Sec. 3) absolutely prohibits private corporations from acquiring any
kind of alienable land of the public domain. Private corporations are allowed to hold alienable lands of
the public domain only through lease. Why?
 In actual practice, the constitutional ban strengthens the constitutional limitation on individuals from
acquiring more than the allowed area of alienable lands of the public domain. Without the constitutional
ban, individuals who already acquired the maximum area of alienable lands of the public domain could
easily set up corporations to acquire more alienable public lands. The corporation is a convenient vehicle
to circumvent the constitutional limitation on acquisition by individuals of alienable lands of the public
domain.
 To enable a corporation to file for registration of alienable and disposable land (and only land not more
than 12 hectares), the corporation must have acquired the land when its transferor had already a vested
right to a judicial confirmation of title to the land by virtue of his open, continuous and adverse
possession of the land in the concept of an owner for at least 30 years since 12 June 1945. (In short, the
land was already a private property at the time it was acquired by the corporation).
 In this case, TAN acquired the land on 8 August 1997 from Porting, who, along with his predecessors-
in-interest, has not shown to have been, as of that date, in open, continuous, and adverse possession of
the land for 30 years since 12 June 1945. Hence, TAN failed to prove that any portion of the land was
already private land when it acquired it from Porting in 1997.

Republic v. Jabson, GRNo. 200223, June 6, 2018


Page 39

REPUBLIC OF THE PHILIPPINES, Petitioner, v. LAKAMBINI C. JABSON, PARALUMAN C. JABSON,


MAGPURI C. JABSON, MANUEL C. JABSON III, EDGARDO C. JABSON, RENATO C. JABSON, NOEL
C. JABSON, AND NESTOR C. JABSON, REPRESENTED BY LAKAMBINI C. JABSON, ATTORNEY-IN-
FACT, Respondents.

Doctrine
That land has been removed from the scope of the Regalian Doctrine and reclassified as part of the public
domain's alienable and disposable portion cannot be assumed or implied. The prevailing rule is that the
applicant must clearly establish the existence of a positive act of the government, such as a presidential
proclamation or an executive order; an administrative action; investigation reports of Bureau of Lands
investigators; and a legislative act or a statute to prove the alienable and disposable nature of the subject land.

Facts:
● There are two parcels of land being applied for registration - one is located at Barrio San Jose, Pasig
City, and the other is situated in Barangay Bagong Katipunan, Pasig City.
● Both used to form part of seven parcels of land owned and possessed by the Jabson family as early as
1909. Each and every applicant herein claims undivided share and participation as follows: Lakambini
C. Jabson-1/5; Paraluman Jabson1/5; Magpuri Jabson-1/5 & Tala J. Olega-1/5; Manuel III, Edgardo,
Renato, Noel & Nestor Jabson as legal heirs of their father Manuel Jabson, Jr.-1/5.
● In 1978, applicants had already applied for registration of the same parcels of land. However this was
dismissed for failure of the applicants to comply with the recommendation of the then Land Registration
Commission to include in their application the complete names and postal addresses of all the lessees
occupying the lands sought to be registered.

RTC:
● Found that respondents Jabson acquired the properties from their predecessors-in-interest who, in tum,
have possessed the same since time immemorial. Upon acquisition, respondents Jabson possessed the
parcels of land for more than 30 years in an open, continuous, exclusive, and notorious manner, and in
the concept of an owner.
● Ruled that respondents Jabson satisfactorily proved and established their rights over the subject
properties, in compliance with Section 14(1) and (2) of Presidential Decree No. 1529.

CA:
● CA reversed the decision of the RTC.
● It held that the applicant has the burden of showing that he is the real and absolute owner in fee simple
of the land applied for. To have his imperfect title confirmed, the applicant must present evidence to
prove that his possession has been adverse, continuous, open, public, peaceful, and in the concept of an
owner
● HOWEVER, the appellate court noted that the rule on confirmation of an imperfect title grounded on
adverse possession does not apply unless and until the subject land has been released in an official
proclamation to that effect so that it may form part of the disposable lands of the public domain. To this
end, the applicant must secure a certification from the Government that the land applied for is in fact
alienable and disposable.
● It found that respondents Jabson did not present any evidence showing that the San Jose property had
already been classified as alienable and disposable land of the public domain.
● A plain photocopy of a purported Community Environment and Natural Resources Office (CENRO)
Certification dated May 14, 1998, which tended to show that the Bagong Katipunan property is "within
the alienable and disposable zone," was submitted to the trial court. 15 However, the Court of Appeals
noted that no party identified, testified to, nor offered the certification in evidence. Thus, the Court of
Appeals held that it cannot be admitted in evidence.
Page 40

RESPONDENTS FILED MR: CA GRANTED


● The Court of Appeals found that respondents Jabson sufficiently established that: (a) they have had
open, continuous, exclusive, and notorious possession of the subject properties; and (b) such properties
formed part of the alienable and disposable lands of the public domain.
● The Court of Appeals pointed out that based on Llanes v. Republic,20 in the interest of substantial
justice and to resolve a material issue in a land registration case, the court is allowed to admit a CENRO
Certification in evidence despite its belated submission and lack of formal offer:
● ruled that respondents Jabson sufficiently established their adverse possession of the subject properties
through the following: (a) by exercising specific acts of ownership such as constructing residential
houses on the subject properties and leasing the same to third parties, and (b) as admitted by petitioner
Republic, by possessing and occupying the San Jose property since 1944.
● Hence, the present petition by the Republic.

Issues:
I. W/N the grant of respondent Jabson’s application for registration of title to the subject property was
proper under the law and current jurisprudence? (NO)

1. It is a general rule that prevailing over claims of land is the Regalian Doctrine, which, as enshrined in
the 1987 Constitution, declares that the State owns all lands of the public domain. Land that has not been
acquired from the government, either by purchase, grant, or any other mode recognized by law, belongs
to the State as part of the public domain.
2. The Public Land Act1 governs the classification and disposition of lands of the public domain, except
for timber and mineral lands. The law also entitles possessors of public lands to judicial confirmation of
their imperfect titles.
3. Section 14 of Presidential Decree No. 15292 provides that any applicant for registration of title to land
derived through a public grant must sufficiently establish three things:
a. the subject land's alienable and disposable nature;
b. his or her predecessors' adverse possession thereof, and
c. the reckoning date from which such adverse possession was under a bona fide claim of
ownership, that is, since June 12, 1945 or earlier.
4. That land has been removed from the scope of the Regalian Doctrine and reclassified as part of the
public domain's alienable and disposable portion cannot be assumed or implied
5. The prevailing rule is that the applicant must clearly establish the existence of a positive act of the
government, such as a presidential proclamation or an executive order; an administrative action;
investigation reports of Bureau of Lands investigators; and a legislative act or a statute to prove the
alienable and disposable nature of the subject land. Thus the DENR Certification dated February 19,
2009 was NOT sufficient evidence to establish the subject properties' alienable and disposable character.
6. FIRST: The respondents CANNOT use the Llanes case as a defense. The CENRO that was belatedly
filed in Llanes was merely a corrected or amended certification, the unedited version of which had been
earlier presented in the trial court as evidence of the alienable and disposable nature of the land. And the
correction or amendment pertained merely to the statement of the reckoning date of adverse possession.
HOWEVER, respondents Jabson failed to present during trial any evidence establishing the subject
properties' alienable and disposable nature. The DENR Certification dated February 19, 2009 was
submitted for the first time by respondents Jabson in their Motion for Reconsideration of the Court of
Appeals' original Decision dated January 30, 2009. This document also cannot be given probative value
- it was not presented and identified during trial, much less formally offered in evidence.
7. SECOND: Carlito P. Castaneda, a DENR Sr. Forest Management Specialist, was not authorized to issue
certifications as to land classification, much less order for the release of lands of the public domain as
alienable and disposable. The Public Land Act vested the President the authority to classify lands of the
Page 41

public domain into alienable and disposable. Subsequently, the Revised Forestry Code of the Philippines
also empowered the DENR Secretary to determine and approve land classification as well as declare the
same as alienable and disposable. Only the DENR Secretary is empowered to declare that a certain
parcel of land forms part of the alienable and disposable portion of the public domain.
8. THIRD: a certification alone is not sufficient in proving the subject land's alienable and disposable
nature. We have already ruled that a PENRO and/or CENRO certification must be accompanied by a
copy of the original classification.

Held
Decision REVERSED, and the Petition is GRANTED.
Republic v. Remman Enterprises, Inc., GR No. 1993l0,Feb. 19, 2014
REPUBLIC OF THE PHILIPPINES, Petitioner, vs. REMMAN ENTERPRISES, INC., represented by
RONNIE P. INOCENCIO, Respondent.

Facts:
An application for land registration was filed in the CFI in Bulacan by herein Respondent. The subject property
was a riceland with an area of 12,342 sq.m. known as Lot 2633, Cad-297, Paombong, Bulacan. It was originally
owned and possessed by one MamertoDionision since 1907 and was, thereafter, sold to Romualda Jacinto in
1926. Upon the death of Romualda Jacinto, her sister Maria Jacinto (mother of the respondent) inherited the
land. Thereafter, upon the death of Maria Jacinto in 1963, respondent had herself inherited the land, owning and
possessing it openly, publicly, uninterruptedly, adversely against the whole world, and in the concept of owner
since then. Taxes due thereon had been paid as well.

The CFI ordered the registration of the land in favor of respondent on the ground that she had sufficiently
established her open, public, continuous and adverse possession in the concept of an owner for more than 30
years. The OSG appealed to the CA and alleged that subject land is a part of the unclassified region
denominated as forest land of Paombong, Bulacan. The CA affirmed the decision of the trial court.

Issues: Whether or not the land subject of the application for registration is susceptible of private
acquisition?
Held: The Court of Appealsdecision is overruled.
CIVIL LAW: land belonging to public domain

Section 14 (1) and (2) of the Property Registration Decree provides for those who may apply for registration of
title to land.

The Court in Republic v. Dela Paz, G.R. No. 171631 held that under Section 14(1), respondent had to prove
that: (1) the land formed part of the alienable and disposable land of the public domain and (2) she, by herself of
through her predecessors-in-interest, had been in open, continuous, exclusive, and notorious possession and
occupation of the subject land under a bona fide claim of ownership from June 12, 1945 or earlier.Thus, the
burden of proof is on the applicant and failure to do so warrants the dismissal of the application.
It is without question that respondent complied with the second requisite. However, the same cannot be said
with regard to the first requisite. No evidence was presented that the subject land had been declared alienable
and disposable by the State.

Realizing that the burden to prove the second requisite belongs to her, respondent attached to her appellee brief
the certification dated March 8, 2000 issued by the Department of Environment and Natural Resources
Community Environment and Natural Resources Office declaring that Lot 2633 falls within the alienable or
disposable land of Paombong, Bulacan. The CA, however, expunged the appellee brief. The Court in Menguito
Page 42

v. Republic G.R. No. 134308 declared that a survey conducted by a geodetic engineer that included a
certification on the classification of the land as alienable and disposable was not sufficient to overcome the
presumption that the land still formed part of the inalienable public domain.

It is a standing doctrine that land of the public domain, in order to be the subject of appropriation, must be
declared alienable and disposable either by the President or the Secretary of the DENR.
Granting for the sake of argument that the certification alone would have sufficed, respondent application would
still be denied considering that the reclassification of the land as alienable or disposable came only after the
filing of the application in court in 1976. The certification indicated that the land was reclassified as alienable or
disposable only on October 15, 1980.

Section 14(2) of the Property Registration Decree provides that ownership of private lands acquired through
prescription may be registered in the owner name. However, respondent did not acquire the land through
prescription notwithstanding the fact that possession of the same by her and her predecessors-in- interest could
be traced back as early as in 1926.

The Court in Heirs of Mario Malabanan v. Republic, G.R. No. 179987 ruled that, roperty of public domain,
which generally includes property belonging to the State, cannot be the object of prescription or, indeed, be
subject of the commerce of man. Lands of the public domain, whether declared alienable and disposable or not,
are property of public dominion and thus insusceptible to acquisition by prescription. xxx It is only when such
alienable and disposable lands are expressly declared by the State to be no longer intended for public service or
for the development of the national wealth that the period of acquisitive prescription can begin to run.

Application for land registration of respondent Rosario de Guzman Vda. De Joson respecting Lot 2633, Cad-
297 is DISMISSED.
Republic v. Joson, GR No. 163767, March 10, 2014
REPUBLIC OF THE PHILIPPINES, represented by THE DIRECTOR OF LANDS, Petitioner, vs. ROSARIO
DE GUZMAN VDA. DE JOSON, Respondent.
BERSAMIN, J.:
Facts: An application for land registration was filed in the CFI in Bulacan by herein Respondent. The subject
property was a riceland with an area of 12,342 sq.m. known as Lot 2633, Cad-297, Paombong, Bulacan. It was
originally owned and possessed by one MamertoDionision since 1907 and was, thereafter, sold to Romualda
Jacinto in 1926. Upon the death of Romualda Jacinto, her sister Maria Jacinto (mother of the respondent)
inherited the land. Thereafter, upon the death of Maria Jacinto in 1963, respondent had herself inherited the
land, owning and possessing it openly, publicly, uninterruptedly, adversely against the whole world, and in the
concept of owner since then. Taxes due thereon had been paid as well. The CFI ordered the registration of the
land in favor of respondent on the ground that she had sufficiently established her open, public, continuous and
adverse possession in the concept of an owner for more than 30 years. The OSG appealed to the CA and alleged
that subject land is a part of the unclassified region denominated as forest land of Paombong, Bulacan. The CA
affirmed the decision of the trial court.

Issues: Whether or not the land subject of the application for registration is susceptible of private
acquisition?

Held: The Court of Appeals decision is overruled.


CIVIL LAW: land belonging to public domain
Section 14 (1) and (2) of the Property Registration Decree provides for those who may apply for registration of
title to land.
Page 43

The Court in Republic v. Dela Paz, G.R. No. 171631 held that nder Section 14(1), respondent had to prove that:
(1) the land formed part of the alienable and disposable land of the public domain and (2) she, by herself of
through her predecessors-in-interest, had been in open, continuous, exclusive, and notorious possession and
occupation of the subject land under a bona fide claim of ownership from June 12, 1945 or earlier. Thus, the
burden of proof is on the applicant and failure to do so warrants the dismissal of the application.
It is without question that respondent complied with the second requisite. However, the same cannot be said
with regard to the first requisite. No evidence was presented that the subject land had been declared alienable
and disposable by the State.

Realizing that the burden to prove the second requisite belongs to her, respondent attached to her appellee brief
the certification dated March 8, 2000 issued by the Department of Environment and Natural Resources
Community Environment and Natural Resources Office declaring that Lot 2633 falls within the alienable or
disposable land of Paombong, Bulacan. The CA, however, expunged the appellee brief. The Court in Menguito
v. Republic G.R. No. 134308 declared that a survey conducted by a geodetic engineer that included a
certification on the classification of the land as alienable and disposable was not sufficient to overcome the
presumption that the land still formed part of the inalienable public domain.
It is a standing doctrine that land of the public domain, in order to be the subject of appropriation, must be
declared alienable and disposable either by the President or the Secretary of the DENR. Granting for the sake of
argument that the certification alone would have sufficed, respondent application would still be denied
considering that the reclassification of the land as alienable or disposable came only after the filing of the
application in court in 1976. The certification indicated that the land was reclassified as alienable or disposable
only on October 15, 1980.

Section 14(2) of the Property Registration Decree provides that ownership of private lands acquired through
prescription may be registered in the owner name. However, respondent did not acquire the land through
prescription notwithstanding the fact that possession of the same by her and her predecessors-ininterest could be
traced back as early as in 1926.

The Court in Heirs of Mario Malabanan v. Republic, G.R. No. 179987 ruled that, roperty of public domain,
which generally includes property belonging to the State, cannot be the object of prescription or, indeed, be
subject of the commerce of man. Lands of the public domain, whether declared alienable and disposable or not,
are property of public dominion and thus insusceptible to acquisition by prescription. xxx It is only when such
alienable and disposable lands are expressly declared by the State to be no longer intended for public service or
for the development of the national wealth that the period of acquisitive prescription can begin to run.

Application for land registration of respondent Rosario de Guzman Vda. De Joson respecting Lot 2633, Cad-
297 is DISMISSED.
Republic v. Rovency Realty, GR No. 190817, Jan. 10, 2018
REPUBLIC OF THE PHILIPPINES, Petitioner vs. ROVENCY REALTY AND DEVELOPMENT
CORPORATION, Respondent

Facts:
MARTIRES, J.:
● On 22 March 2001, RRDC filed before the RTC an Amended Application for Registration covering a
parcel of land - Lot No. 3009 (subject land) situated in Barangay Balulang, Cagayan de Oro City, a
parcel of land (Lot No. 3009, Cad-237, Cagayan Cadastre) situated in the Barrio of Carmen, City of
Cagayan de Oro, Island of Mindanao with an area of 318,345 sq m more or less.
● RRDC alleged that it is a domestic corporation; the absolute owner in fee simple of the subject land
having acquired the same from its previous owner, P.N. Roa Enterprises, Inc., by virtue of a notarized
Page 44

deed of absolute sale executed on 05 March 1997; that the subject land was assessed at ₱2.228M as
shown in the Tax Declaration (TD) No. 141011; that it has registered the subject land for taxation
purposes and paid the realty taxes, to the filing of the application; that immediately after acquiring the
subject land, it took actual physical possession of the same and continuously occupying the subject land;
and that it and its predecessors-in- interest have been in open, continuous, adverse, and peaceful
possession in concept of owner of the subject land since time immemorial, or for more than thirty (30)
years.
● Attached to the application are: original copy of the technical description of the subject land6; the
Tracing Cloth Plan of the survey plan7; Certification in Lieu of Surveyor's/Geodetic Engineer's
Certificate8 issued by the Chief of the Land Surveys Assistance Section, DENR Region X; T.D. No.
141011 in the name of RRDC; and the Deed of Absolute Sale between RRDC and P.N. Roa Enterprises,
Inc., dated 5 March 1997.
● On 16 July 2001, an opposition to the application was filed by the Heirs of Paulino Avancena. They
alleged the following:
○ The subject land was already claimed and owned by the late Atty. Paulino Avancena (Paulino),
their father and predecessor-in-interest, as early as 1926;
○ That Paulino had been in open, continuous, notorious, adverse, and exclusive possession and
occupation of the subject land; that Paulino registered the subject land for taxation purposes and
has paid the taxes due thereon in 1948;
○ That their parents, Paulino and Rizalina Neri (Rizalina) merely allowed and tolerated Pedro N.
Roa's (Pedro) possession of the subject land after the latter approached them and requested that
he be allowed to use the subject land for his businesses;
○ That Pedro is one of RRDC's predecessors-in-interest; that sometime in 1994, Rizalina
demanded the return of the subject land from the heirs of Pedro, but to no avail;
○ That in 1996, Rizalina died leaving the private oppositors as the rightful heirs of the subject land;
that their parents never sold the subject land to Pedro nor to RRDC, and as such, no right or title
over the subject land was passed on to RRDC. Thus, they prayed that RRDC's application be
dismissed, and that their opposition be treated as their own application for registration.
● On 3 August 2001, the petitioner Republic, through the OSG, filed its opposition to the application on
the following grounds: that neither RRDC nor its predecessors-in-interest have been in open,
continuous, exclusive, and notorious possession and occupation of the land in question since 12 June
1945 or prior thereto; that the subject land exceeds the twelve (12)- hectare limit for confirmation of
imperfect title set by Section 47 of Commonwealth Act (CA.) No. 141, as amended by Republic Act
(R.A.) No. 6940; and that the subject land forms part of the public domain belonging to the Republic
and, thus, not subject to private appropriation.
● During trial, RRDC presented deeds of sale with different dates from 1937 to 1996 covering the subject
property. RRDC also presented a certification CENRO,Cagayan de Oro City, certifying that the subject
land is alienable and disposable and not covered by any public land application patent and hence, no
patent has been issued thereon. RRDC presented several tax declarations, the earliest of which is T.D.
No. 91264 (1947).
● On the other hand, to support their claim that a patent over the subject land had been issued in the name
of their father, the private oppositors presented a certification issued by the Records Management
Division of the Lands Management Bureau of the DENR which merely states that " It is unfortunate
however that as of this moment, this office (Records Management Division) cannot locate said records
despite diligent search made thereon."
● The RTC Ruling - granted RRDC's application for registration of the subject land.
● Unconvinced, the Republic and private oppositors heirs filed an appeal to the CA.
● The Republic moved for reconsideration; while the Heirs of Paulino Avanceña adopted the Republic's
motion for reconsideration as their own - denied.
Hence, this petition.
Page 45

Issues:
1. WHETHER OR NOT THE LAND APPLIED FOR REGISTRATION OF TITLE BY
RESPONDENT IS IN EXCESS OF WHAT IS ALLOWED BY LAW AND THAT
RESPONDENT'S RIGHT TO ACQUIRE THE SUBJECT PARCEL OF LAND IS FURTHER
LIMITED BY THE CORPORATION CODE.
2. WHETHER RESPONDENT'S EVIDENCE IS INSUFFICIENT TO PROVE THAT IT OR ITS
PREDECESSORS-IN-INTEREST HAVE BEEN IN OPEN, CONTINUOUS, EXCLUSIVE AND
NOTORIOUS POSSESSION UNDER A BONA FIDE CLAIM OF OWNERSHIP SINCE JUNE
12, 1945 OR EARLIER AND THE SUBJECT PROPERTY IS NO LONGER INTENDED FOR
PUBLIC USE OR FOR THE DEVELOPMENT OF THE NATIONAL WEALTH.27

HELD: The petition is meritorious.


12-hectare limit under Section 3, Article XII of the 1987 Constitution
As can be clearly gleaned from its language, Section 3, Article XII applies only to lands of the public domain.
Private lands are, therefore, outside of the prohibitions and limitations stated therein. Thus, the appellate court
correctly declared that the 12-hectare limitation on the acquisition of lands under Section 3, Article XII of the
1987 Constitution has no application to private lands.
A case in point is the absolute prohibition on private corporations from acquiring any kind of alienable land of
the public domain. This prohibition could be traced to the 1973 Constitution which limited the alienation of
lands of the public domain to individuals who were citizens of the Philippines. This constitutional prohibition,
however, does not necessarily mean that corporations may not apply for original registration of title to lands. In
fact, the Court, in several instances, affirmed the grant of applications for original registration filed by
corporations, for as long as the lands were already converted to private ownership by operation of law as a
result of satisfying the requisite possession required by the Public Land Act.
The pronouncements in Director of Lands and TA.N. Properties apply with equal force to the 12- hectare
limitation, considering that both the limitation and the prohibition on corporations to acquire lands, do not cover
ownership of private lands. Stated differently, whether RRDC can acquire the subject land and to what extent,
depends on whether the pieces of evidence it presented before the trial court sufficiently established that the
subject land is alienable and disposable land of the public domain; and that the nature and duration of the
possession of its individual predecessors-in-interest converted the subject land to private land by operation of
law.

Requirements for original registration of title to land


In Republic of the Philippines vs. Cortez,32 the Court explained that applicants for original registration of title
to land must first establish compliance with the provisions of either Section 14(1) or Section 14(2) of P.D. No.
1529, which state:
Sec. 14. Who may apply. The following persons may file in the proper Court of First Instance an application for
registration of title to land, whether personally or through their duly authorized representatives:
(1) Those who by themselves or through their predecessors-in interest have been in open, continuous, exclusive
and notorious possession and occupation of alienable and disposable lands of the public domain under a bona
fide claim of ownership since June 12, 1945, or earlier.
(2) Those who have acquired ownership of private lands by prescription under the provision of existing laws.

It must be emphasized that the requirements and bases for registration under these two provisions of law differ
from one another. Section 14 (1) mandates registration on the basis of possession, while Section14(2) entitles
registration on the basis of prescription. 33 Thus,it is important to ascertain under what provision of Section 14
the registration is sought.
Page 46

A reading of the application, however, is unavailing. In its application, RRDC alleged that it and its
predecessors-in-interest "had been in open, continuous, adverse, and peaceful possession in concept of owner of
the subject property since time immemorial or for more than thirty years." This allegation made it unclear
whether registration is sought under Section 14(1) - possession since 12 June 1945 or earlier; or under Section
14(2) - possession for more than thirty years.

An examination of the 7 November 2003 RTC decision also proved futile considering that, and as previously
pointed out, aside from enumerating the exhibits offered by the applicant, the trial court did not discuss how
these pieces of evidence established the requisites for registration. Thus, for the proper resolution of the issues
and arguments raised herein, it becomes necessary for the present application to be scrutinized based on the
requirements of the provisions of Sections 14 (1) and (2) of P.D. No. 1529.

Registration under Section 14(1) of P.D. No. 1529


Under Section 14(1), applicants for registration of title must sufficiently establish the following requisites: first,
that the subject land forms part of the disposable and alienable lands of the public domain; second, that the
applicant and his predecessors-in-interest have been in open, continuous, exclusive, and notorious possession
and occupation of the same; and third, that the possession is under a bona fide claim of ownership since 12 June
1945, or earlier.34

The first requisite of Section 14(1) entails only that the property sought to be registered be alienable and
disposable at the time of the filing of the application for registration. To prove that the land sought to be
registered is alienable and disposable, the present rule is that the application for original registration must be
accompanied by (1) a CENRO or PENRO Certification; and (2) a copy of the original classification approved
by the DENR Secretary, and certified as true copy by the legal custodian of the official records. This strict
requirement for the registration of lands enunciated in TA.N Properties had been consistently applied and
affirmed by the Court in a plethora of cases.

In the present case, to prove that the subject land is alienable and disposable, RRDC presented a CENRO
certification stating that the subject land is "alienable and disposable and not covered by any public land
application." RRDC, however, failed to present a certified true copy of the original classification approved by
the DENR Secretary declaring the subject land alienable and disposable. Clearly, the evidence presented by
RRDC falls short of the requirements in TA.N. Properties. Thus, the trial and appellate courts erred when they
ruled that the subject land is alienable and disposable part of the public domain and susceptible to original
registration.

Furthermore, RRDC also failed to prove that it and its individual predecessors-in-interest sufficiently complied
with the required period and nature of possession.
An applicant for land registration must exhibit that it and its predecessors-in-interest had been in open,
continuous, exclusive, and notorious possession and occupation of the land under a bona fide claim of
ownership since 12 June 1945 or earlier. It has been held that possession is open when it is patent, visible,
apparent, notorious, and not clandestine; it is continuous when uninterrupted, unbroken, and not intermittent or
occasional; it is exclusive when the adverse possessor can show exclusive dominion over the land and an
appropriation of it to his own use and benefit; and notorious when it is so conspicuous, that it is generally
known and talked of by the public or the people in the neighborhood.38

In Republic vs. Remman Enterprises, Inc., the Court held that for purposes of land registration under Section
14(1) of P.D. No. 1529, proof of specific acts of ownership must be presented to substantiate the claim of open,
continuous, exclusive, and notorious possession and occupation of the land subject of the application.
Applicants for land registration cannot just offer general statements which are mere conclusions of law rather
Page 47

than factual evidence of possession. Actual possession is in the manifestation of acts of dominion over it of such
nature as a party would actually exercise over his own property.

In Republic v. Gielczyk, the Court explained that "possession" and "occupation" are not synonymous to each
other. Possession is broader than occupation because it includes constructive possession; whereas occupation
delimits the all-encompassing effect of constructive possession. Thus, taken together with the words open,
continuous, exclusive, and notorious, the word occupation means that for one's title to land to be judicially
recognized, his possession of the land must not be mere fiction.40

In this case, aside from the deeds of absolute sale covering the subject land which were executed prior to 12
June 1945, RRDC did not present any evidence which would show that its predecessors- in-interest actually
exercised acts of dominion over the subject land even before the cut-off period. As such, RRDC failed to prove
that its possession of the land, or at the very least, its individual predecessors-in-interest's possession over the
same was not mere fiction.

Neither would the tax declarations presented by RRDC suffice to prove the required possession. To recall, the
earliest of these tax declarations dates back only to 1948. Clearly, the required possession and occupation since
12 June 1945 or earlier, was not demonstrated.

From the foregoing, it is clear that RRDC failed to prove that its individual predecessors-in-interest had been in
open, continuous, exclusive and notorious possession and occupation of the subject land under a bona fide claim
of ownership since 12 June 1945 or earlier; and that said possession and occupation converted the subject land
into a private property by operation of law. Consequently, the subject land cannot be registered in the name of
RRDC under Section 14(1) of P.D. No. 1529.
Requirements under Section 14(2) of P.D. No. 1529
RRDC also failed to establish compliance with the requirements for registration under Section 14(2).

In Heirs of Mario Malabanan vs. Republic(Malabanan),41 the Courtexplained that whenSection14(2) of P.D.
No. 1529 provides that persons "who have acquired ownership over private lands by prescription under the
provisions of existing laws," it unmistakably refers to the Civil Code as a valid basis for the registration of
lands. The Civil Code is the only existing law that specifically allows the acquisition by prescription of private
lands, including patrimonial property belonging to the State.
The Civil Code makes it clear that patrimonial property of the State may be acquired by private persons through
prescription.1âwphi1 This is brought about by Article 1113, which states that all things which are within the
commerce of man are susceptible to prescription, and that property of the State or any of its subdivisions not
patrimonial in character shall not be the object of prescription.42

Nonetheless, this does not necessarily mean that when a piece of land is declared alienable and disposable part
of the public domain, it can already be acquired by prescription. In Malabanan, this Court ruled that declaration
of alienability and disposability is not enough - there must be an express declaration that the public dominion
property is no longer intended for public service or the development of the national wealth or that the property
has been converted into patrimonial, thus:
"(2) In complying with Section 14(2) of the Property Registration Decree, consider that under the Civil Code,
prescription is recognized as a mode of acquiring ownership of patrimonial property. However, public domain
lands become only patrimonial property not only with a declaration that these are alienable or disposable. There
must also be an express government manifestation that the property is already patrimonial or no longer retained
for public service or the development of national wealth, under Article 422 of the Civil Code. And only when
the property has become patrimonial can the prescriptive period for the acquisition of property of the public
dominion begin to run. "43 [emphasis supplied]
Page 48

The classification of the land as alienable and disposable land of the public domain does not change its status as
property of the public dominion under Article 420(2) of the Civil Code. As such, said land, although classified
as alienable and disposable, is insusceptible to acquisition by prescription.44 In this case, RRDC did not present
any evidence which would show that the subject land was expressly declared as no longer intended for public
service or the development of the national wealth, or that the property has been converted into patrimonial.
Hence, it failed to prove that acquisitive prescription has begun to run against the State, and that it has acquired
title to the subject land by virtue thereof.

In fine, RRDC failed to satisfy all the requisites for registration of title to land under either Sections 14(1) or (2)
of P.D. No. 1529. RRDC also failed to establish that when it or P.N. Roa Enterprises, Inc., also a corporation
and its direct predecessor-in-interest, acquired the subject land, it had already been converted to private
property, thus, the prohibition on the corporation's acquisition of agricultural lands of the public domain under
Section 3, Article XII of the 1987 Constitution applies. RRDC's application for original registration of imperfect
title over Lot No. 3009 must perforce be denied.

WHEREFORE, the instant petition is GRANTED. The Application for Registration of Lot No. 3009 filed by
Rovency Realty and Development Corporation is DENIED. SO ORDERED.
Republic v. Cabrera, GR No. 218418, Nov. 8, 2017
REPUBLIC OF THE PIDLIPPINES, represented by the REGIONAL EXECUTIVE DIRECTOR, DENR,
REGION IV, MANILA, Petitioner vs.THE HEIRS OF MEYNARDO CABRERA, as herein represented by
MEYNARDO CABRERA, JR. and ALMA RODRIGUEZ CABRERA, THE HEIRS OF CONSOLACION
DIMACULANGAN CABRERA, as herein represented by ALEXANDER CABRERA, MANIBI CABRERA,
MILAGROS CABRERA GARA, AND RAUL CABRERA, JACKSON CINCO DY, LORETA AGBAYANI,
GLORIA SORIANO, CRIS CALMA, NORA LIWANAG and the REGISTER OF DEEDS OF ORIENTAL
MINDORO, Respondent
DECISION
CAGUIOA, J:

The Case
This is a Petition for Review on Certiorari[1] (Petition) filed under Rule 45 of the Rules of Court against the
Decision[2] dated July 18, 2014 (Assailed Decision) and Resolution[3] dated May 20, 2015 (Assailed
Resolution) in CA-G.R. CV No. 98120 rendered by the Court of Appeals (CA) Eleventh Division and Special
Former Eleventh Division, respectively.
The Assailed Decision and Resolution stem from an appeal from the Decision[4] dated December 5, 2005
rendered by the Regional Trial Court of Roxas, Oriental Mindoro, Branch 43 (RTC) in Civil Case No. C-358,
dismissing the complaint for cancellation of free patent and reversion filed by the Republic of the Philippines
(Republic) against the Heirs of Meynardo Cabrera (Heirs of Meynardo), the Heirs of Consolacion
Dimaculangan Cabrera (Heirs of Consolacion), Jackson Cinco Dy (Dy), Loreta Agbayani (Agbayani), Gloria
Soriano (Soriano), Cris Calma (Calma), Nora Liwanag (Liwanag), and the Register of Deeds of Oriental
Mindoro (ROD) (collectively, Respondents).[5]

The Facts
Sometime in 1971, Meynardo filed an Application for Free Patent concerning an 8,072[6] square-meter parcel
of land situated in Pining, Roxas, Oriental Mindoro.[7] In said application, Meynardo alleged that he had been
in possession of such parcel of land since 1936, through his predecessor-in interest Marcelo Cabrera.[8]
In the same year, the Bureau of Lands (BOL) issued Free Patent No. 516197 in favor of Meynardo, covering
two (2) lots denominated as: (i) Lot 1 with an area of 3,591 square meters, and (ii) Lot 2, with an area of 4,481
square meters.[9] On the basis of said patent, the ROD issued Original Certificate of Title (OCT) No. RP-132
(P-9193) covering both lots in Meynardo's name.[10]
Page 49

Thereafter, a 2,445-square-meter portion of Lot 1 (Lot 1-A[11] was transferred to Consolacion.[12] Thus, on
April 6, 1982, Transfer Certificate of Title (TCT) No. 16580 covering Lot 1-A was issued in Consolacion's
name.[13] Later still, Consolacion sold portions of Lot 1-A to several purchasers namely: Dy, Agbayani,
Soriano, Calma, and Liwanag.[14]
Learning of the issuance of TCT No. 16580, Jose and Leticia De Castro (De Castros), claiming to be the actual
possessors of Lot 1-A, filed before the Department of Environment and Natural Resources (DENR) a petition
urging DENR to conduct an investigation to determine Lot 1-A's land classification status.[15]

Consequently, in the DENR Final Investigation Report[16] (DENR Final Report) dated November 9, 1994
issued by Erwin D. Talento of the DENR Land Management Office (LMO), Free Patent No. 516197, covering
Lots 1, 1-A, and 2 (collectively, Roxas Properties), was declared null and void for having been issued over land
forming part of the public domain. The pertinent portions of the DENR Final Report read:
Sensing that they don't have any chance in the court to prove their better right to occupy and possess [Lot 1-A]
x x x the [De Castros] addressed their petitions to the DENR basing their claim on the weight of a certification
of [the National Mapping and Resource Information Authority (NAMRIA)] x x x. The [De Castros] are now
seeking administrative remedies for the issue which they have already brought to the attention of the court and
wherein they have failed to prove their priority right to occupy and possess [Lot 1-A]. Granting that [the Roxas
Properties constitute] forest land and [Free Patent No. 516197] issued in favor of [Meynardo] be (sic) rendered
null and void [ab] initio, it (sic) doesn't warrant that they have better right to possess and occupy [Lot 1-A]
because [Meynardo, through his predecessors-in-interest] have entered [Lot 1-A] since the year 1943 and have
exercised their ownership over the same x x x.

In view of the foregoing, it is respectfully recommended that the petition of [the De Castros] be dismissed x x x
and appropriate legal action be instituted for the cancellation of Free Patent No. 516197 issued in favor of
Meynardo x x x for the same covers land of the public domain which is certified by the proper authority as
public forest.[17] (Emphasis supplied.)

Thereafter, Antonio G. Principe, the DENR Regional Executive Director of Region IV, issued an Order[18]
dated August 8, 1997 declaring Free Patent No. 516197 null and void.
Later, on November 15, 1999, the Republic filed against the Respondents a complaint (Complaint) for the
annulment and/or cancellation of Free Patent No. 516197, OCT No. RP-132 (P-9193), and TCT No. 16580. The
Complaint also prayed for the reversion of the Roxas Properties in the State's favor.[19]
The Republic based its claim on the (i) DENR Final Report; and (ii) NAMRIA certifications dated January 31,
1994, February 1, 1994, and October 3, 1994, all stating that the Roxas Properties (including Lot 1-A) had been
reclassified as forest land as early as November 24, 1949. The statements in these documents were, in turn,
based on the inscriptions appearing on Land Classification Map No. 209 (LC Map 209) dated March 6, 1924
covering the Roxas Properties. The Republic reasoned that while LC Map 209 indicates that the parcels of land
thereunder were classified as alienable and disposable at the time it was prepared, a subsequent annotation made
thereon indicates that they were reclassified as forest land sometime thereafter, and had thus become
inalienable.[20]

In their respective answers, the Respondents averred, among others, that: (i) Lot 1-A forms part of the alienable
and disposable land of the public domain, as evidenced by the original statements appearing on LC Map 209;
(ii) the annotations appearing on LC Map 209 do not serve as sufficient proof of reversion; and (iii) the land
area which had been purportedly reclassified as forest land was not properly identified since the Republic failed
to present the technical description corresponding thereto.[21] In addition to these common assertions,
respondents Dy, Agbayani, Soriano, and Liwanag further averred that they acquired portions of Lot 1-A from
Consolacion in good faith, and have, since then, been in actual, exclusive, open, and continuous possession of
their respective portions as owners.[22]
Page 50

On December 5, 2005, the RTC rendered a Decision, the dispositive portion of which states:
ACCORDINGLY, judgment is hereby rendered DISMISSING the instant complaint for lack of merit.
SO ORDERED.[23]

The RTC found that the Republic failed to present proof that the Roxas Properties (including Lot 1-A) have
been reclassified as forest land. Citing Republic v. Animas,[24] (Animas) the RTC held that in order to prove
reversion of alienable and disposable land to forest land, a positive government act evincing the same is
necessary.[25]

The Republic filed a motion for reconsideration (MR), which was denied in the RTC's Order dated October 18,
2011.[26]

CA Proceedings
Aggrieved, the Republic elevated the case to the CA via petition for review under Rule 42, docketed as CA-
G.R. CV No. 98120 (Appeal).

In the Appeal, the Republic argued that the Court's ruling in Animas cannot be applied to the present case, since,
in the former, the fact sought to be established was the classification of forest land to alienable and disposable
land, and not the other way around, as in this case.[27] Further, the Republic averred that fraud must have
necessarily attended the issuance of Free Patent No. 516197, OCT No. RP-132 and TCT No. 16580, owing to
the status of the Roxas Properties as forest land.[28]
On July 18, 2014, the CA rendered the Assailed Decision dismissing the Appeal. The dispositive portion of said
decision reads:

WHEREFORE, premises considered, the Appeal is DISMISSED. The Decision dated December 5, 2005 of the
[RTC] x x x is AFFIRMED.
SO ORDERED.[29]

According to the CA, the Public Land Act vests the power to classify (and reclassify) lands of the public domain
with the President. On this score, the CA held that the annotations appearing on LC Map 209 anent the alleged
reversion of the Roxas Properties deserve scant consideration, as they do not appear to be based on any
executive directive. Consequently, the NAMRIA certifications and DENR Final Report relied upon by the
Republic are insufficient to sustain its cause, as they are, in turn, based solely on said annotations.[30]
The Republic filed an MR, which was denied by the CA in its Assailed Resolution dated May 20, 2015. The
Republic received a copy of the Assailed Resolution on June 8, 2015.[31]

On June 19, 2015, the Republic filed a Motion for Extension of Time to File Petition for Review, praying for an
additional period of twenty-five (25) days from June 23, 2015, or until July 18, 2015 within which to file a
petition for review on certiorari. Subsequently, the Republic filed a Second Motion for Extension, praying for a
five (5)-day extension.[32]

Finally, on July 22, 2015, the Republic filed the present Petition, to which Respondents filed their Compliance
and Comment dated December 16, 2016.[33]
Thereafter, the Republic filed a Manifestation and Motion dated May 28, 2017, adopting the Petition as its reply
to Respondents' Compliance and Comment.[34]

The Issue
The Petition calls on the Court to determine whether the CA erred when it held that a positive act of
government is necessary to evince the reclassification of land from alienable and disposable to forest.
Page 51

The Court's Ruling


In this Petition, the Republic maintains that the Court's ruling in Animas did not have the effect of making a
positive executive act a necessary requirement for the purpose of proving the reclassification of alienable and
disposable land.[35] Instead, the Republic posits that Animas affirms its right to institute reversion proceedings
in instances where portions of forest land are erroneously included within the scope of land patents.[36]
Moreover, the Republic argues that in reversion proceedings, the State should not be made to bear the burden of
proving that the land in question constitutes public domain (i.e., forest land).[37] In any case, the Republic
posits that the documentary and testimonial evidence it had presented sufficiently proved such fact.[38]
The Petition should be denied for lack of merit. The CA did not err when it affirmed the RTC Decision, as the
Republic failed to establish that the Roxas Properties were classified as forest land at the time Free Patent No.
516197 was issued.

The Republic's Petition and Respondents' Compliance and Comment should be admitted in the interest of
substantial justice.

At the outset, the Court notes that the parties herein, albeit at different stages of the proceedings, have both
prayed for the relaxation of the Rules of Court (Rules).
For its part, the Republic filed two (2) motions which sought for an aggregate period of thirty (30) days from
the expiration of the initial thirty (30)-day period prescribed by the Rules for the filing of a petition for review
on certiorari. The Respondents, on the other hand, sought the admission of their Compliance and Comment,
filed more than seven (7) months after the filing of the Petition.[39]
Considering the nature of the issues involved in the present Petition, and the lack of evidence showing that
neither the Republic's nor the Respondents' requests for accommodation had been impelled by any ill-motive,
the Court resolves to admit in the interest of substantial justice the Republic's Petition and the Respondents'
Comment with Compliance.

The Court's ruling in Animas does not apply to the present case.
The Republic's Petition primarily proceeds from the supposition that in ruling in favor of Respondents, the RTC
and the CA erroneously relied on Animas.

In Animas, the Republic filed an action for reversion against respondent therein, claiming that the Free Patent
issued in the latter's favor covered forest land. The Court of First Instance dismissed the Republic's action on the
ground that the original certificate of title covering said land had become indefeasible, the same having been
issued more than one (1) year prior to the filing of the Republic's action. Hence, the issue brought before the
Court in Animas was whether the lapse of said one (1)-year period had the effect of precluding the State from
initiating reversion proceedings to recover land which had been unlawfully registered, either through fraud or
oversight. Resolving the issue, the Court held that public land fraudulently or erroneously included in the scope
of patents or certificates of title may be recovered by the State through reversion proceedings, in accordance
with the Public Land Act.

While the Animas ruling upholds the State's right to seek reversion with respect to fraudulently or erroneously
registered lands, it does not, in any manner, lay down the facts that must be established for an action for
reversion to prosper. Undoubtedly, the RTC and CA's reliance on the Animas ruling is misplaced.
Nevertheless, such erroneous reliance on Animas, as will be discussed below, does not advance the Republic's
cause, since the principle which serves as basis for the decisions of the RTC and CA remains correct, albeit
attributed to the wrong case.

The power to classify and reclassify land lies solely with the Executive Department.
The Regalian Doctrine has long been recognized as the basic foundation of the State's property regime,[40] and
has been consistently adopted under the 1935, 1973, and 1987 Constitutions;[41] it espouses that all lands of the
Page 52

public domain belong to the State, and that, as a consequence thereof, any asserted right of ownership over land
necessarily traces back to the State.[42]

At present, Section 3, Article XII of the 1987 Constitution classifies lands of the public domain into five (5)
categories — forest lands, agricultural lands, timber lands, mineral lands, and national parks. The Court's ruling
in Heirs of the Late Spouses Palanca v. Republic,[43] instructs that in the absence of any prior classification by
the State, unclassified lands of the public domain assume the category of forest lands not open to disposition.
[44]

In turn, the classification of unclassified lands of the public domain, and the reclassification of those previously
classified under any of the categories set forth in the 1987 Constitution (such as the Roxas Properties), are
governed by Commonwealth Act No. 141[45] dated November 7, 1936, otherwise known as the Public Land
Act. Sections 6 and 7 thereof provide:
SEC. 6. The President, upon the recommendation of the Secretary of Agriculture and Commerce, shall from
time to time classify the lands of the public domain into —
(a) Alienable or disposable,
(b) Timber, and
(c) Mineral lands,
and may at any time and in a like manner transfer such lands from one class to another, for the purposes of their
administration and disposition.

SEC. 7. For the purposes of the administration and disposition of alienable or disposable public lands, the
President, upon recommendation by the Secretary of Agriculture and Commerce, shall from time to time declare
what lands are open to disposition or concession under this Act. (Emphasis supplied)
These provisions are clear and leave no room for interpretation - the classification and reclassification of public
lands into alienable or disposable, mineral or forest land is the exclusive prerogative of the Executive
Department,[46] and is exercised by the latter through the President, or such other persons vested with authority
to exercise the same on his behalf.[47]
Since the power to classify and reclassify land are executive in nature, such acts, effected without executive
authority, are void, and essentially ultra vires.
In reversion proceedings, the State bears the burden of proving that the property in question was inalienable at
the time it was decreed or adjudicated in favor of the defendant.

A land registration proceeding is the manner through which an applicant confirms title to real property. In this
proceeding, the applicant bears the burden of overcoming the presumption of State ownership.[48] Accordingly,
the applicant is bound to establish, through incontrovertible evidence, that the land sought to be registered had
been declared alienable or disposable through a positive act of the State.[49]
Conversely, reversion proceeding is the manner through which the State seeks to revert land to the mass of the
public domain;[50] it is proper when public land is fraudulently awarded and disposed of in favor of private
individuals or corporations,[51] or when a person obtains a title under the Public Land Act which includes, by
oversight, lands which cannot be registered under the Torrens system as they form part of the public domain.
[52]

Owing to the nature of reversion proceedings and the outcome which a favorable decision therein entails, the
State bears the burden to prove that the land previously decreed or adjudicated in favor of the defendant
constitutes land which cannot be owned by private individuals. The Court's ruling in Republic v. Development
Resources Corporation[53] is instructive:
Page 53

Since a complaint for reversion can upset the stability of registered titles through the cancellation of the original
title and the others that emanate from it, the State bears a heavy burden of proving the ground for its action. x x
x[54] (Emphasis supplied)

Thus, in Republic v. Espinosa[55] (Espinosa), the Court held that the dismissal of the Republic's action for
reversion is proper since the Republic failed to establish that the land subject thereof was classified as forest
land at the time the cadastral decree in favor of the defendant was issued:
[I]t is undisputed that Espinosa was granted a cadastral decree and was subsequently issued OCT No. 191-N x x
x. Having been granted a decree in a cadastral proceeding, Espinosa can be presumed to have overcome the
presumption that the land sought to be registered forms part of the public domain. This means that Espinosa, as
the applicant, was able to prove by incontrovertible evidence that the property is alienable and disposable
property in the cadastral proceedings.
xxxx

In this case, the State, through the Solicitor General, alleges neither fraud nor misrepresentation in the cadastral
proceedings and in the issuance of the title in Espinosa's favor. The argument for the State is merely that the
property was unlawfully included in the certificate of title because it is of the public domain.
Since the case is one for reversion and not one for land registration, the burden is on the State to prove that the
property was classified as timberland or forest land at the time it was decreed to Espinosa. To reiterate, there is
no burden on [the present owner] to prove that the property in question is alienable and disposable land. At this
stage, it is reasonable to presume that Espinosa, from whom [the present owner] derive[s] her title, had already
established that the property is alienable and disposable land considering that she succeeded in obtaining the
OCT over it. In this reversion proceeding, the State must prove that there was an oversight or mistake in the
inclusion of the property in Espinosa's title because it was of public dominion. This is consistent with the rule
that the burden of proof rests on the party who, as determined by the pleadings or the nature of the case, asserts
the affirmative of an issue.[56] (Emphasis and underscoring supplied)

Hence, to resolve this Petition, the Court must determine whether the documentary and testimonial evidence
offered by the Republic are sufficient to sustain its cause.

The Complaint should be dismissed as the Republic failed to show that the Roxas Properties (including Lot 1-
A) were classified as forest land at the time Free Patent No. 516197 was issued in Meynardo's favor.
To recall, the Republic presented the following pieces of evidence to support its complaint for reversion: (i)
DENR Final Report; (ii) NAMRIA certifications; and (iii) LC Map 209. However, these documents, whether
taken individually or collectively, do not evince a positive act of reclassification by the Executive Department.
As aptly stated by the CA:

In this case, the Republic presented the [NAMRIA certifications], the [DENR Final Report] and [LC Map 209]
dated March 6, 1924, with an inscription that the [Roxas Properties] [were] reverted x x x to the category of
forest land on November 24, 1949. However, it appears that the findings of the CENRO and the NAMRIA are
based solely on such mapping [LC Map 209] where eighteen (18) hectares, including the location therein of the
[Roxas Properties], [were] reclassified as forest land. Engineer [Mariano] Mendez[57] testified that:
xxxx

Even Engineer Mendez of the NAMRIA agreed that a law or proclamation is required before a certain parcel of
land is reclassified from alienable and disposable to forest land. His insistence that because the land was
(originally) swamp land that reclassification was made (sic), is not supported by any presidential or legal
pronouncement or by practice and tradition x x x Unfortunately, the Republic failed to present any law,
presidential proclamation, order or act to prove that the subject property was indeed within the area which is
reclassified as forest land. Even an administrative order from the Bureau of Forestry was not presented to show
Page 54

that the subject property had been reclassified as forest land.[58] (Additional emphasis and underscoring
supplied)

The foregoing testimony, culled from the Assailed Decision, confirms that the alleged reclassification of the
Roxas Properties is bereft of basis, as it was done by Engineer Mendez on his sole account, without any prior
directive from the President, or a duly authorized officer from the Executive Department. In fact, the annotation
appearing on LC Map 209 upon which the Republic relies does not even state upon whose authority the alleged
reclassification had been made,[59] placing the annotation's validity, veracity and worth in serious doubt.
Ultimately, the Republic failed to prove that the Roxas Properties (including Lot 1-A) were classified as forest
land when they were decreed in Meynardo's favor in 1971. Thus, in accordance with the Court's ruling in
Development Resources Corporation and Espinosa, the present Petition must be, as it is hereby, denied.
WHEREFORE, premises considered, the Petition for Review on Certiorari is DENIED. The Assailed Decision
of the Court of Appeals dated July 18, 2014 and Resolution dated May 20, 2015 in CA-G.R. CV No. 98120 are
hereby AFFIRMED.
SO ORDERED.

Carpio, (Chairperson), Peralta, and Reyes, Jr., JJ., concur.


Perlas-Bernabe, J., on official leave.
Republic v. CA and De la Rosa, GR No. L-43938, April 15, 1980
G.R. No. L-43938 April 15, 1988 REPUBLIC OF THE PHILIPPINES (DIRECTOR OF FOREST
DEVELOPMENT), petitioner, vs. HON. COURT OF APPEALS (THIRD DIVISION) and JOSE Y. DE LA
ROSA, respondents.
G.R. No. L-44081 April 15, 1988 BENGUET CONSOLIDATED, INC., petitioner, vs. HON. COURT OF
APPEALS, JOSE Y. DE LA ROSA, VICTORIA, BENJAMIN and EDUARDO, all surnamed DE LA ROSA,
represented by their father JOSE Y. DE LA ROSA, respondents.
G.R. No. L-44092 April 15, 1988 ATOK-BIG WEDGE MINING COMPANY, petitioner, vs. HON. COURT OF
APPEALS, JOSE Y. DE LA ROSA, VICTORlA, BENJAMIN and EDUARDO, all surnamed DE LA ROSA,
represented by their father, JOSE Y. DE LA ROSA, respondents.

Republic of the Philippines, Benguet & Atok vs. Court of Appeals & De La Rosa G.R. No. L-43938, April 15,
1988
Cruz, J.:

Facts: These consolidated cases arose from the application for registration of a parcel of land filed on February
11, 1965, by Jose de la Rosa on his own behalf and on behalf of his three children, Victoria, Benjamin and
Eduardo. The land, situated in Tuding, Itogon, Benguet Province, was divided into 9 lots and covered by plan
Psu-225009. According to the application, Lots 1-5 were sold to Jose de la Rosa and Lots 6-9 to his children by
Mamaya Balbalio and Jaime Alberto, respectively, in 1964. The application was separately opposed by Benguet
Consolidated, Inc. as to Lots 1-5, Atok Big Wedge Corporation, as to Portions of Lots 1-5 and all of Lots 6-9,
and by the Republic of the Philippines, through the Bureau of Forestry Development, as to lots 1-9. In support
of the application, both Balbalio and Alberto testified that they had acquired the subject land by virtue of
prescription; Balbalio claimed to have received Lots 1-5 from her father shortly after the Liberation.
Benguet opposed on the ground that the “June Bug” mineral claim covering Lots 1-5 was sold to it on
September 22, 1934, by the successors-in-interest of James Kelly, who located the claim in September 1909 and
recorded it on October 14, 1909. From the date of its purchase, Benguet had been in actual, continuous and
exclusive possession of the land in concept of owner, as evidenced by its construction of adits, its affidavits of
annual assessment, its geological mappings, geological samplings and trench side cuts, and its payment of taxes
on the land.
Page 55

For its part, Atok alleged that a portion of Lots 1-5 and all of Lots 6-9 were covered by the Emma and Fredia
mineral claims located by Harrison and Reynolds on December 25, 1930, and recorded on January 2, 1931, in
the office of the mining recorder of Baguio. These claims were purchased from these locators on November 2,
1931, by Atok, which has since then been in open, continuous and exclusive possession of the said lots as
evidenced by its annual assessment work on the claims, such as the boring of tunnels, and its payment of annual
taxes thereon.

The Bureau of Forestry Development also interposed its objection, arguing that the land sought to be registered
was covered by the Central Cordillera Forest Reserve under Proclamation No. 217 dated February 16, 1929.
Moreover, by reason of its nature, it was not subject to alienation under the Constitutions of 1935 and 1973.
The trial court denied the application, holding that the applicants had failed to prove their claim of possession
and ownership of the land sought to be registered.

The applicants appealed to the respondent court, which reversed the trial court and recognized the claims of the
applicant, but subject to the rights of Benguet and Atok respecting their mining claims. In other words, the
Court of Appeals affirmed the surface rights of the de la Rosas over the land while at the same time reserving
the sub-surface rights of Benguet and Atok by virtue of their mining claims. Both Benguet and Atok have
appealed to this Court, invoking their superior right of ownership.

Issue: WHETHER OR NOT APPLICANTS ALL SURNAMED DELA ROSA HAVE SUPERIOR
RIGHTS OF OWNERSHIP OVER THE SURFACE RIGHTS OVER THE LAND IN QUESTION
WHILE OPPOSITORS BENGUET CONSOLIDATED, INC. AND ATOK BIG WEDGE MINING
COMPANY ARE RESERVED OF THEIR SUB- SURFACE RIGHTS BY VIRTUE OF THEIR MINING
CLAIM AS DECIDED BY THE RESPONDENT COURT.

Held: NO. Our holding is that Benguet and Atok have exclusive rights to the property in question by virtue of
their respective mining claims which they validly acquired before the Constitution of 1935 prohibited the
alienation of all lands of the public domain except agricultural lands, subject to vested rights existing at the time
of its adoption. The land was not and could not have been transferred to the private respondents by virtue of
acquisitive prescription, nor could its use be shared simultaneously by them and the mining companies for
agricultural and mineral purposes. It is true that the subject property was considered forest land and included in
the Central Cordillera Forest Reserve, but this did not impair the rights already vested in Benguet and Atok at
that time. Such rights were not affected either by the stricture in the Commonwealth Constitution against the
alienation of all lands of the public domain except those agricultural in nature for this was made subject to
existing rights. The perfection of the mining claim converted the property to mineral land and under the laws
then in force removed it from the public domain. By such act, the locators acquired exclusive rights over the
land, against even the government, without need of any further act such as the purchase of the land or the
obtention of a patent over it. As the land had become the private property of the locators, they had the right to
transfer the same, as they did, to Benguet and Atok. The Court of Appeals justified this by saying there is “no
conflict of interest” between the owners of the surface rights and the owners of the sub-surface rights. This is
rather doctrine, for it is a well-known principle that the owner of piece of land has rights not only to its surface
but also to everything underneath and the airspace above it up to a reasonable height. Under the aforesaid
ruling, the land is classified as mineral underneath and agricultural on the surface, subject to separate claims of
title. This is also difficult to understand, especially in its practical application. The Regalian doctrine which, as
its name implies, is intended for the benefit of the State, not of private persons. The rule simply reserves to the
State all minerals that may be found in public and even private land devoted to "agricultural, industrial,
commercial, residential or (for) any purpose other than mining." Thus, if a person is the owner of agricultural
land in which minerals are discovered, his ownership of such land does not give him the right to extract or
utilize the said minerals without the permission of the State to which such minerals belong.
Page 56

The flaw in the reasoning of the respondent court is in supposing that the rights over the land could be used for
both mining and non-mining purposes simultaneously. The correct interpretation is that once minerals are
discovered in the land, whatever the use to which it is being devoted at the time, such use may be discontinued
by the State to enable it to extract the minerals therein in the exercise of its sovereign prerogative. The land is
thus converted to mineral land and may not be used by any private party, including the registered owner thereof,
for any other purpose that will impede the mining operations to be undertaken therein. The Regalian doctrine
then extends not only to land but also to “all natural wealth that may be found in the bowels of the earth.”
Leonidas v. Vargas, 201031, Dec. 14, 2017
TOMAS R. LEONIDAS, Petitioner, v. TANCREDO VARGAS AND REPUBLIC OF THE
PHILIPPINES, Respondents.
DECISION

DEL CASTILLO, J.:

Assailed in this Petition for Review on Certiorari1 are the August 13, 2009 Decision 2 and February 22, 2012
Resolution3 of the Court of Appeals (CA) in CA-G.R. CV No. 02296, which affirmed with modification the
March 19, 2007 Decision4 of the Regional Trial Court (RTC) of Barotac Viejo, Iloilo, Branch 66, in LRC Case
No. 02-195.

Factual Antecedents

On February 2, 2002, Tomas R. Leonidas (herein petitioner) filed an application for land
registration5 (Application) covering Lot 566 and Lot 1677 which are both situated in Concepcion, Iloilo
(collectively, subject lots).

Petitioner alleged that he inherited the subject lots from his parents, Ponciano Leonidas, Jr. (Ponciano) and
Asuncion Roxas de Leonidas (Asuncion); that as evidenced by the May 17, 1937 Certificate of Sale issued by
the Provincial Treasurer of Iloilo, the subject lots, then covered by Tax Declaration (TD) No. 722, were
purchased by Asuncion when auctioned due to delinquency in the payment of real property taxes by the original
owners, the heirs of Inis Luching; that Asuncion immediately took possession of the subject lots and exercised
dominical rights thereover notoriously, continuously, and exclusively; that upon Asuncion’s death in 1986,
Ponciano succeeded to the ownership and possession of the subject lots; that after Ponciano's death in 1991, the
subject lots became his (petitioner's) own exclusive property; that he permitted and tolerated the occupation of
some portions of the subject lots by Juanito Tisolan, Pancing Guevarra, Carmencita Guevarra, Delia Aspera-
Ecleo, Victorino Mosqueda, Nora Biñas, Crisanto Amangas (Amangas), 6 Rosana Vasquez, Henry Asturias,
Ronnie Astorias, Antonio Asturias, and Jacob Narciso; that as far as known to him (petitioner), the following are
the owners of all adjoining properties, i.e. the owners of Lot 564, Lot 565, Lot 1578, and Lot 1677, Mansueto
Sicad, Francisco Aspero, Brigido Celestial, and Eugenio Bondoc, Jr. who are all from Poblacion, Concepcion,
Iloilo, and Carmen Paoli of unknown address; that Lot 566 is bounded on the west by the provincial road and he
(petitioner) does not claim any portion thereof; that the latest assessed value of the subject lots is P51,660.00 as
certified by the Provincial Treasurer of Iloilo; that to the best of his knowledge and belief, there is no mortgage
or encumbrance of any kind whatsoever affecting the subject lots except for taxes due thereon; that a certain
Tomas Varga(Tomas), however, had declared a portion of the subject lots in his name for taxation purposes but
that Tomas died shortly after the end of the Second World War, and the whereabouts of his heirs, if any, are
unknown, despite his diligent search to locate them in Concepcion, Iloilo, and elsewhere.
Page 57

Petitioner also alleged that he was 77 years old, Filipino, a resident of No. 55 Chestnut St., West Fairview,
Quezon City and married to Ofelia Gustilo Leonidas (Ofelia); that attached to his Application were the original
Survey Plans with two photographic copies each, the Tracing Cloth Plan (Sepia), a certificate of unavailability
issued by the Chief, Records Section, Land Management Services, Department of Environment and Natural
Resources (DENR), Region VI, Iloilo City, in lieu of the surveyor's certificate, Technical Descriptions with
photographic copies, the Certificate in quadruplicate of the Provincial Treasurer showing the latest assessed
value of the subject lots, and a copy of the muniment of title to prove ownership of the subject lots, with the
original to be presented at the trial.

Petitioner thus prayed that the subject lots be brought under the operation of the Property Registration
Decree7 (PD 1529) and that the titles thereto be registered and confirmed in his name.

The Republic of the Philippines (Republic), represented by the Office of the Solicitor General (OSG), opposed
the said Application. The Republic claimed that neither the petitioner nor his predecessors-in-interest had been
in continuous, exclusive, and notorious possession and occupation of the subject lots since June 12, 1945, or
prior thereto, as required by Section 48 of Commonwealth Act (CA) No. 141, as amended by PD 1073; that the
petitioner's muniment/s of title, tax declarations, and tax payment receipts did not constitute competent and
sufficient evidence of either a bona fide acquisition of the subject lots, and neither did the petitioner's bare claim
of open, continuous, exclusive, and notorious possession and occupation thereof in the concept of owner since
June 12, 1945, or prior thereto, amount to convincing proof of his claim of possession and ownership over the
subject lots; that, although the petitioner's muniments of title might appear genuine, the tax declarations and/or
tax payments showing the pretended possession were, in fact, of recent vintage; that the claim of ownership in
fee simple on the basis of a Spanish title or grant could no longer be availed of by petitioner who had failed to
file an appropriate application therefor within the period of six months from February 16, 1976, as required by
PD 892; and that the subject lots are portions of the public domain belonging to the Republic which are not
subject to private appropriation. Thus, the Republic prayed that the petitioner's Application be denied and that
the subject lots be declared part of the public domain.

On March 11, 2003, Tancredo Vargas (Tancredo) also filed an Opposition 8 to the Application. Tancredo averred
that he is Tomas' legitimate son and compulsory heir; that during Tomas's lifetime, the latter was the absolute
and exclusive owner of a certain parcel of land located at Loong, Concepcion, Iloilo, which parcel of land is
bounded on the north by the seashore, on the south by Severino Asturias (Asturias), 9 on the east by the seashore,
and on the west by Asturias and Braulio Celestial; that this parcel of land had an area of 36,237 square meters
and was covered by TD No. 3549 in Tomas's name; that the petitioner does not exclusively own Lot 1677 since
it had been split into two, viz. Lot 1677-A and Lot 1677-B; that he (Tancredo) is the owner of Lot 1677-A; that
Lot 566 was also not exclusively owned by the petitioner, as this Lot 566 had also been divided into two
lots, viz. Lot 566-A and Lot 566-B; that he (Tancredo) is the owner of Lot 566-A as shown in the RPTA Tax
Mapping project in the Municipality of Concepcion, Iloilo; that the petitioner's allegation that the owners of the
property covered by TD 772 became delinquent in the payment of the tax due thereon, for which reason the
Provincial Treasurer of Iloilo allegedly sold the same to Asuncion, was not at all true; that the property covered
by TD 772 was not sold at public auction because the forfeiture was lifted prior to the public auction sale; and
that the fact that the Office of the Provincial Treasurer of Iloilo did not have a copy of the Certificate of Sale
dated May 17, 1937 bolstered the argument that petitioner's allegation is questionable. Tancredo thus prayed
Page 58

that the petitioner's Application be denied insofar as the portions covered by the TDs in the name of Tomas
(disputed portions) are concerned.

On March 21, 2003, another Opposition 10 to the Application was filed by Moncerat A. Sicad-De Julian, Gil A.
Sicad, represented by his wife, Elizabeth Sicad, Teresita A. Sicad-Bayuran, Villaluz Sicad-Zarriz, Eden A.
Sicad, and Melchor Sicad, represented by his wife, Elena D. Sicad, (Elena; collectively, the Sicads) all
represented by their attorney-in-fact, Elena.11 These oppositors claimed that they are the heirs of the late
Mansueto Sicad (Mansueto) who was the owner of a portion of the subject lots (Sicads's contested portion); that
the Sicads's contested portion was bought by Mansueto from Asturias as evidenced by the Deed of Definite Sale
of a Parcel of Land described as Doc. No. 75, Page No. 35, Book No. 1, Series of 1950 of the notarial register of
notary public Crespo Celestial; that the Sicads's contested portion had been in the possession of Mansueto
during the latter's lifetime; that they had been in possession of the Sicads's contested portion since Mansueto's
death; that part of the Sicads's contested portion had already been registered under Original Certificate of Title
(OCT) No. F-36795; and that the petitioner had never been in possession of the lots subject of his Application.
The Sicads thus prayed that the petitioner's Application be dismissed, insofar as it concerned the Sicads's
contested portion as set forth in the aforesaid Deed of Definite Sale; and that the Sicads's contested portion be
registered instead in their names.

At the trial, the petitioner presented himself and Geronimo C. Peñaflorida (Peñaflorida), Land Management
Inspector, DENR, Community Environment and Natural Resources Office (CENRO), at Sara, Iloilo as
witnesses.12 On the other hand, Catalino Guinez, Emeliana Isturias Matulac, and Elena testified for the
Sicads.13 For his part, Tancredo presented himself and a former overseer or tenant of the Vargas familv, 14 Jose
Etchona (Etchona).15 Then on August 8, 2003, the petitioner filed his Formal Offer of Evidence 16 wherein he
submitted the Certificate of Sale dated May 17, 1937, TD 014134 tor the year 1976 in Asuncion's name and
covering Cadastral Lot Nos. 1, 2, and 3 PSU 216090, TD 0037 for the year 1994 in the names of Asuncion and
Ponciano and covering Cadastral Lot No. 1677, TD 0036 for the year 1994 in the names of Asuncion and
Ponciano and covering Cadastral Lot No. 566, TD 0114 for the year 2003 in the names of Asuncion and
Ponciano and covering Cadastral Lot No. 1677-A, TD 0118 for the year 2003 in the names of Asuncion and
Ponciano and covering Cadastral Lot No. 1 677-B, TD 0116 for the year 2003 in the names of Asuncion and
Ponciano and covering Cadastral Lot No. 566-A; and TD 0117 for the year 2003 in the names of Asuncion and
Ponciano and covering Cadastral Lot No. 566-B, 17 tax receipts for the years 1986, 1987, 1988, 1989, 1990,
1991, 1994, 2002 and 2003, statement of the assessed value issued by the Provincial Assessor of Iloilo on
March 26, 1996, Lot No. 566's Blue Print Survey Plan with technical description, Lot 1677's Blue Print Survey
Plan with technical description, Certificate of Unavailability of Surveyor's Certificate of Survey for Lots 566
and 1677, and Survey Inspection Report dated August 28, 1997 for Lot Nos. 566 and 1677 issued by
Peñaflorida,18 i.e. CENRO Report dated August 28, 1997, to the effect that the subject lots are free from liens
and encumbrances, and are moreover within the alienable and disposable area. Pursuant to the RTC's directive,
petitioner also offered as additional evidence the originally approved subdivision plan covering Lot No. 1677,
Csd-06-008798 to prove the identity and location of the easement for public use; 19 and a certification by Joel B.
Diaz, CENRO at Sara, Iloilo, to the effect that Lot No. 1677, Pls 1099, situated in Brgy. Loong, Concepcion,
Iloilo, with an area of 8,062 square meters was issued Patent No. 063015-92-846 dated May 28, 1992 in the
name of Flordeluz Sedigo, but that Lot No. 1677 has doubled with the lot situated at Poblacion, Concepcion,
Iloilo in the name of the Heirs of Ponciano and that this latter lot is not covered by any public land application
filed with the CENRO in Sara, Iloilo, which explained why no patent has been issued therefor, hence indicating
Page 59

that this other Lot No. 1677, Pls 1099, which is situated in Brgy. Aglusong, Concepcion, Iloilo is entirely
different from Lot No. 1677, which is situated in Sitio Loong, Poblacion, Concepcion, Iloilo.20

The petitioner likewise submitted in evidence an Ocular Inspection Report covering an ocular inspection earlier
ordered by the RTC.21

Ruling of the Regional Trial Court

In its Decision dated March 19, 2007, the RTC disposed of this case in this wise:

WHEREFORE, general default having been declared and the [A]pplication supported by evidence, the
adjudication and registration of portion of Lot No. 566 with an area of 3.1161 hectares and portion of Lot 1677
with an area of 3.7255 hectares, all of Concepcion Cadastre, together with all the improvements thereon are
hereby ordered in favor of applicant [petitioner], of legal age, married to [Ofelia], Filipino, and resident of
Fairview, Quezon City, Philippines. Portions of Lot [No.] 1677 with an area of 2.3642 hectares and portion of
Lot [No.] 566 with an area of 1.1782 hectares are hereby adjudicated in favor of [Tancredo], of legal age, single,
Filipino, and resident of Lawa-an Village, Balantang, Jaro, Iloilo City, Philippines which portions shall be
segregated in a proper subdivision survey and to follow the description of the plan of Municipal Assessor of
Concepcion, Iloilo commensurate to Lot 1677-A under [T.D.] No. 054822 and 566-A under [T.D.] No. 0550.

The easement of right of way of the lots, highways, streets, alleys, shorelines and other portion[s] of land not
specified as lots located within the borders of the land covered by this case are declared to be the properties of
the [Republic].

The Clerk of Court is directed to forward copies of this decision to all government agencies concerned.

And finally, the Administrator, Land Registration Authority, is hereby directed, after this decision shall have
become final for which he shall be duly advised by specific order of this Court, to issue [a] decree of
registration and title in accordance with the amended plan on file in the record.

SO ORDERED.23

The RTC held that petitioner had sufficiently established that his predecessors-in-interest had possessed and
owned a parcel of land in Barangay Loong, Concepcion, Iloilo to the extent not covered by Tancredo's
Opposition; that while petitioner and his predecessors-in-interest might not have been in actual possession of
the subject lots at all time, they nonetheless had been consistently visiting the same; and that petitioner's claim
of possession and ownership is supported by documents consisting of the Certificate of Sale issued by the
Provincial Treasurer of Iloilo on May 17, 1937, the tax declarations in Asuncion's name for the years 1976,
1994, and 2003, the official receipts showing payments of real estate taxes thereon, and the statement of the
assessed value issued by the Provincial Assessor of Iloilo on May 26, 1996. The RTC stressed that the period of
possession by petitioner and his predecessors-in-interest sufficed to confer a registrable title upon petitioner.

The RTC likewise ruled that Tancredo was also able to establish a superior claim with respect to his disputed
portions; that all of the tax declarations in Asuncion's name continuously bore the annotation acknowledging
Tomas's adverse claim relative to Tancredo's disputed portions; that Tomas's open and continuous possession for
more than the required number of years was sufficiently shown by a tax declaration issued as early as the year
1945; that the overseers and other persons authorized to manage Tancredo's disputed portions were never driven
Page 60

out by petitioner; and that Tancredo had visited the disputed portions more frequently than petitioner who, as
the evidence shows, has his permanent residence in Quezon City, Metro Manila.

With regard to the claim of the Sicads, the RTC held that Mansueto and his successors-in-interest had no more
interest in the Sicads' contested portion because what was shown to have been sold by Asturias to Mansueto
pertained to a lot measuring only two hectares, 52 acres, and 92 ares, a parcel of land at par with the land
covered by the aforementioned free patent issued to Mansueto.

The RTC emphasized that it is well-entrenched in jurisprudence that alienable public land openly, continuously,
and exclusively possessed by a person personally or through his predecessors-in-interest for at least 30 years
becomes ipso jure private property by mere lapse of time, or by completion of said period pursuant to Section
48(b) of CA 141, as amended by RA 1942 and RA 3872.

Ruling of the Court of Appeals

Only the petitioner and the Republic filed their respective Notices of Appeal 24 which were given due course by
the RTC in its Order of May 25, 2007.25 These notices of appeal were consolidated and docketed as CA-G.R.
CV No. 02296. In a Decision dated August 13, 2009, the CA disposed as follows:

WHEREFORE, the Decision dated March 19, 2007 is modified, as follows: 1.) the portion pertaining to the
award of [Lot No.] 566 with an area of 3.1161 hectares and [Lot No.] 1677 with an area of 3.7255 hectares to
[petitioner], is REVERSED and SET ASIDE; and 2.) the portion pertaining to the award of [Lot No.] 1677 with
an area of 2.3642 hectares and [Lot No.] 566 with an area of 1.1782 hectares in favor of [Tancredo] is
AFFIRMED.

SO ORDERED.26

The CA held that, contrary to the Republic's stance, the records showed that there had been compliance with the
jurisdictional requirements of publication, posting, and notice; that petitioner had properly identified the subject
lots; that the subject lots had already been classified as alienable and disposable at the time that petitioner filed
the Application in 2002, pursuant to the CENRO Report dated August 28, 1997 issued by Peñaflorida; that it has
been held that "[a] certification by the CENRO of the DENR stating that the subject lots are found to be within
the alienable and disposable site per land classification project map is sufficient evidence to show the real
character of the land subject of the application;" 27 that these notwithstanding, petitioner failed to prove with the
requisite evidence the kind of possession and the length of time required by law for the registration of the
subject lots in his name, because his lone testimony did not suffice to establish his and his predecessors-in-
interest's alleged open, continuous, exclusive, and notorious possession over the subject lots since June 12,
1945, or earlier; that petitioner's alleged acts of swimming in, and planting trees on the subject lots, his having
finished high school at the Victorino Salcedo High School in the neighboring town of Sara, Iloilo, and his
having left the subject lots when he attended college — all these neither added up nor supported his assertion of
dominion or ownership over the subject lots; that his allegation that his childhood memories regarding the
subject lots all came back to him after the death of his father Ponciano was indicative of the fact that he was
really unaware of the existence of the subject lots; that his Application was even opposed by Tancredo and by
the Sicads who claimed exclusive possession over certain portions of the subject lots; that petitioner's failure to
explain why he or his predecessors-in-interest declared the subject lots for taxation purposes only in 1976, was
inconsistent with his claim of possession thereover since 1937; and that it is an axiom of the law that the burden
Page 61

of proof in a land registration case rests upon the applicant who must present clear, positive, and convincing
evidence establishing the alleged possession and occupation in good faith, and for the period required by law.

On the other hand, the CA ruled that Tancredo had sufficiently proven his open, continuous, exclusive, and
notorious possession and occupation for the period required by law, over the portions of the subject lots he was
claiming in the concept of an owner; that Tomas's adverse claims were annotated on the TDs issued in
Asuncion's name covering the disputed portions, i.e. TD 014134, 0114, and 0117;28 that Tomas declared the
disputed portions for taxation purposes in his name as early as 1945; that Tancredo himself testified that Tomas
first used the disputed portions as rice land and converted the same into coconut land in the 1960s; that
Tancredo's witness, Etchona, likewise testified that Tomas employed him and Domingo Celestial not only to
cultivate, but also to guard the disputed portions, and that Tomas himself appropriated the harvest from the
disputed portions and introduced improvements thereon; and that even petitioner himself admitted in his
Application that Tomas had declared the disputed portions in his (Tomas') name for taxation purposes.

Petitioner moved for reconsideration29 but was denied by the CA in its Resolution of February 22, 2012.30

Issue

Before this Court, petitioner now raises the following issue:

[Whether] the [CA] gravely abused its discretion in denying the registration of [his] already vested title
[over] Lot [Nos.] 566 and 1677 of the Concepcion, Iloilo Cadastre as his private property, and in
awarding some portions thereof in favor of [Tancredo] in this land registration proceeding. 31

Petitioner's arguments

Petitioner insists in his Petition,32 Consolidated Reply,33 and Memorandum34 that the CA erred in finding that he
failed to prove that he and his predecessors-in-interest had been in open, continuous, exclusive, and notorious
possession and occupation of the subject lots since June 12, 1945, or earlier, and that there is indubitable
evidence that the subject lots were in fact sold in a tax sale on May 17, 1937 by the government through the
Provincial Treasurer of Iloilo; that he filed the present Application so that an OCT can be issued in his name as
evidence of his vested title over the subject lots; that assuming that the subject lots are still part of the public
domain, he is nevertheless still entitled to have the subject lots registered in his name by reason of his and his
predecessors-in-interest's exclusive possession and occupation thereof for more than 30 years, as compared to
Tancredo's possession which supposedly began only in 1945; that under the Land Registration Act, as amended,
the possessor is deemed to have acquired by operation of law the right to a government grant upon compliance
with the conditions therefor, which was just what he did in this case; that the confirmation proceeding is a mere
formality and the registration thereunder does not confer title but merely recognizes a title that is already vested;
that rejection of his vested title to the questioned lots will occasion loss of confidence in the government's sales
of forfeited property by reason of tax delinquency; that the CA erred in finding that the TDs in Asuncion's name
carried Tomas's adverse claim, as the attached copies thereof did not bear any such annotations; that the CA also
erred in stating that petitioner did not present any TDs to support his claim of ownership over the subject lots
for the reason that the CA Decision itself mentioned that he submitted a TD for the year 1976; that contrary to
the CA's findings, he did testify that he had visited the subject lots every so often to plant trees after he and his
parents left Concepcion in 1945, and that such improvements were reflected in his exhibits; that the CA likewise
erred in holding that he only came to know about the subject lots after the death of his father, Ponciano, for the
Page 62

fact is that he did testify that he and his cousins used to swim in the sea near the subject lots, as early as when he
was 12 years old; that the CA moreover erred in concluding that Tancredo had successfully established his
claims over the disputed portions of the subject lots because the TDs in Asuncion's name are all annotated with
Tomas's adverse claim, and that Tomas had declared said disputed portions in his name as early as 1945; that the
tax declarations supposedly in Tomas's name were neither presented nor offered in evidence; that Tancredo
admitted during his cross-examination that Tomas's 1945 tax declaration was procured notwithstanding the fact
that the subject lots had already been declared in Asuncion's name; that Tancredo did not comply with the
pertinent provisions of the Land Registration Act, as amended because he did not present evidence to prove the
specific date in 1945 when Tomas acquired the disputed portions, or how Tomas in fact acquired the same; that
besides these, Tancredo could not identify the disputed portions that he was claiming; that if Tancredo wanted to
vindicate his claims of ownership over the disputed portions, then Tancredo should institute the proper action
before a court of general jurisdiction, and not in the land registration court, as the subject lots were no longer
part of the public domain; that the issue of whether the sale by the government to Asuncion on May 17, 1937
changed the classification of the subject lots from public to private is of first impression and should be resolved
by the Supreme Court En Banc; and that the circumstances obtaining in this case are exceptions to the rule that
only questions of law are allowed in a petition filed pursuant to Rule 45 of the Revised Rules of Court; and that
to deny his Application, or to render judgment ordering the reversion to public ownership of the subject lots
would amount to grave abuse on the part of the judiciary.

The Republic's Arguments

In its Comment35 and Memorandum,36 the Republic counters that the instant Petition merely raises questions of
fact which are proscribed under Rule 45 of the Revised Rules of Court; that this Court is not a trier of facts; that
petitioner's case does not fall under any of the exceptions to the rule that factual findings of the CA are
invariably binding upon the Supreme Court; and that the assailed CA Decision should not be disturbed because
the CA had amply justified the reversal of the RTC Decision which was erected upon the petitioner's failure to
substantiate his claim of ownership over the subject lots.

Tancredo's Arguments

In his Comment37 and Memorandum38 Tancredo maintains that the disputed portions had been in the absolute
possession and dominion of Tomas; that the findings of the RTC and the CA regarding petitioner's ineligibility
to obtain title to the disputed portions due to non-compliance with the requirements of the law, and for
insufficiency of evidence, should not be disturbed; that the CA's finding that petitioner's TDs bore the annotated
claims of Tomas on the subject lots is a factual finding and should not be disturbed; that petitioner's possession
is not the possession required by law for purposes of land registration because petitioner failed to present
evidence that would prove actual, notorious, continuous, and exclusive possession and occupation of the subject
lots; that the evidence adduced by petitioner is self-serving, hence undeserving of any weight; that the origin of
the disputed portions as pointed out by the RTC is Assessor's Lot No. 337, which is individually identified after
the Cadastral Survey as Lot Nos. 1676- A, 1677-A, and 566-A, all of the Concepcion (Iloilo) Cadastre; that
petitioner is barred or estopped from questioning the identity of the disputed portions that had been adjudicated
to him (Tancredo), as the lack of sufficient identification pertained to the subject lots that petitioner himself was
trying to register; and that the issues raised by petitioner were factual in nature, and the same is proscribed
under Rule 45 of the Revised Rules of Court.
Page 63

The fundamental issues to be resolved in this case are: (1) Whether the petitioner is entitled to obtain a title over
the subject lots; and (2) Whether Tancredo has established, by his own evidence, that he was qualified to acquire
title over the disputed portions claimed by him.

The Court's Ruling

The Petition is denied.

Requisites for the confirmation and registration of an imperfect and incomplete title under CA 141 and PD
1529

"The Regalian doctrine, embodied in Section 2, Article XII of the 1987 Constitution, provides that all lands of
the public domain belong to the State, which is the source of any asserted right to ownership of
land."39 "[Commonwealth Act No. 141, in turn,] governs the classification and disposition of lands of the public
domain. Section 11 [thereof] provides, as one of the modes of disposing public lands that are suitable for
agriculture, the 'confirmation of imperfect or incomplete titles.' Section 48 [thereof], on the other hand,
enumerates those who are considered to have acquired an imperfect or incomplete title over public lands and,
therefore, entitled to confirmation and registration under the Land Registration Act [now PD 1529]." 40 The latter
law then "specifies who are qualified to apply for registration of land." 41 Taken together, all the foregoing
provide for the requisites for the confirmation and registration of an imperfect and incomplete title, thus —

x x x In particular, Section 14 (1) [of PD 1529] in relation to Section 48 (b) of [CA] 141, as amended by Section
4 of P.D. No. 1073, states:

SEC. 14. Who may apply. — The following persons may file in the proper Court of First Instance
[now Regional Trial Court] an application for registration of title to land, whether personally or through their
duly authorized representatives:

(1) Those who by themselves or through their predecessors-in-interest have been in open, continuous, exclusive
and notorious possession and occupation of alienable and disposable lands of the public domain under a bona
fide claim of ownership since June 12, 1945, or earlier.

xxx xxx xxx

Section 48. The following described citizens of the Philippines, occupying lands of the public domain or
claiming to own any such lands or an interest therein, but whose titles have not been perfected or completed,
may apply to the Court of First Instance [now Regional Trial Court] of the province where the land is located
for confirmation of their claims and the issuance of a certificate of title therefor, under [PD 1529], to wit:

xxx xxx xxx

(b) Those who by themselves or through their predecessors in-interest have been in open, continuous, exclusive
and notorious possession and occupation of [alienable and disposable lands] of the public domain, under a bona
fide claim of acquisition of ownership, since June 12, 1945, or earlier, immediately preceding the filing of the
application for confirmation of title except when prevented by war or force majeure. These shall be conclusively
presumed to have performed all the conditions essential to a Government grant and shall be entitled to a
certificate of title under the provisions of this chapter.
Page 64

Based on these legal parameters, applicants for registration of title under Section 14 (1) must sufficiently
establish: (1) that the subject land forms part of the disposable and alienable lands of the public domain; (2) that
the applicant and his predecessors-in-interest have been in open, continuous, exclusive and notorious possession
and occupation of the same; and (3) that his possession has been under a bona fide claim of ownership since
June 12, 1945, or earlier.

These triple requirements of alienability and possession and occupation since June 12, 1945 or earlier under
Section 14 (1) are indispensable prerequisites to a favorable registration of title to the property. Each element
must necessarily be proven by no less than clear, positive and convincing evidence; otherwise, the application
for registration should be denied.42

Petitioner did not cite the specific provision of CA 141 upon which he based his Application. Nevertheless, the
allegations therein seem to establish the fact that his claim is one of imperfect title under the above quoted
Section 48(b) of CA 141 in relation to Section 14(1) of PD 1529.

The subject lots are considered alienable and disposable lands of the public domain

The first requirement is complied with in the case at bench. Notwithstanding that only a CENRO certification
covering the subject lots was presented in the instant case, the subject lots are considered alienable and
disposable lands of the public domain because of this Court's ruling that an application for land registration may
be granted despite the absence of the DENR Secretary's certification, provided that the same was pending at the
time Republic v. Vega43 was promulgated on January 17, 2011. In Republic v. Alora,44 this Court expressly
clarified this matter in this wise:

x x x [I]n Republic v. T.A.N. Properties, Inc., which was promulgated on 26 June 2008 x x x we held that
applicants for land registration must present a copy of the original classification approved by the DENR
Secretary and certified as true copy by the legal custodian of the official records. x x x

x x x In Republic v. Serrano [(decided on 24 February 2010)], we allowed the approval of a land registration
application even without the submission of the certification from the DENR Secretary. As this ruling presented
an apparent contradiction with our earlier pronouncement in Republic v. TA.N. Properties, Inc., we sought to
harmonize our previous rulings in Republic v. Vega [(decided on 17 January 2011)]. We then said that the
applications for land registration may be granted even without the DENR Secretary's certification provided that
the application was currently pending at the time Republic v. Vega was promulgated. x x x45

It is worth stressing, however, that the foregoing ruling is the exception, not the rule. As explicitly elucidated
in Republic v. Vega:46

It must be emphasized that the present ruling on substantial compliance applies pro hac vice. It does not in any
way detract from our rulings in Republic v. T.A.N. Properties, Inc., and similar cases which impose a strict
requirement to prove that the public land is alienable and disposable, especially in this case when the Decisions
of the lower court and the [CA] were rendered prior to these rulings. To establish that the land subject of the
application is alienable and disposable public land, the general rule remains: all applications for original
registration under [PD 1529] must include both (1) a CENRO or PENRO certification and (2) a certified true
copy of the original classification made by the DENR Secretary.
Page 65

As an exception, however, the courts — in their sound discretion and based solely on the evidence presented on
record — may approve the application, pro hac vice, on the ground of substantial compliance showing that there
has been a positive act of government to show the nature and character of the land and an absence of effective
opposition from the government. This exception shall only apply to applications for registration currently
pending before the trial court prior to this Decision and shall be inapplicable to all future applications.
(Underscoring and emphases in the original)47

That said, we hold that both the petitioner and Tancredo failed to establish clearly and convincingly their
respective rights to registration of imperfect titles under CA 141 and PD 1529, as will be discussed below.

Petitioner failed to prove possession of the subject lots in the manner and for the period required by law

First off, petitioner failed to establish bona fide possession and ownership over the subject lots since June 12,
1945 or earlier. His contention that his predecessors-in-interest became the owners of the subject lots pursuant
to the May 17, 1937 Certificate of Sale48 of the Forfeited Real Property issued by the Provincial Treasurer of
Iloilo appears to be consistent with the fact that TD 3549 in Tomas's name which was found by the CA as issued
in 1945 bears an annotation stating that such is "[c]ontested by [Asuncion]". 49 Even then, the Certificate of
Public Sale indicated that the balance of the purchase price in the amount of P29.44, was yet to be paid
on or before December 31, 1937.50

No incontrovertible proof was, however, presented to establish the fact that this balance of the purchase price in
the said amount of 29.44 had indeed been paid on or before December 31, 1937. In addition, the CA also
correctly pointed out that even as petitioner was able to submit TDs and evidence of tax payments only for a
few years, he nevertheless failed to explain why he or his predecessors-in-interest declared the subject lots for
taxation purposes only in 1976, this despite his claim that his predecessors in-interest had been in possession
and occupation of the subject lots since 1937, as allegedly shown in the Provincial Treasurer's Certificate of
Sale. It is settled that intermittent and irregular tax payments run counter to a claim of ownership or
possession.51

Second, even assuming for argument's sake that petitioner's predecessors-in-interest had paid the balance of the
delinquent tax payment, petitioner nonetheless failed to prove his and his predecessors-in-interests actual,
notorious, exclusive and continuous possession of the subject lots for the length of time required by law.

To be sure, petitioner's failure to explain what happened after his family supposedly left the subject lots in 1941,
when the war broke out, vis-à-vis his failure to prove that he had indeed introduced valuable improvements in
the subject lots during the time that he and his parents had been allegedly in actual possession and occupation
thereof, cast doubts upon his claim of actual possession and occupation thereof. Withal, petitioner's testimony of
having swum near the subject lots, of having planted trees thereon, and his having finished high school at the
Victorino Salcedo High School in the neighboring town of Sara can hardly be considered as acts of dominion or
ownership over the subject lots. Besides, petitioner did not present clear and convincing evidence that the
subject lots had indeed been cultivated by him or by his predecessors-in-interest for the period of time required
by law. Needless to say, all these failings weaken his claim that he has been a bona fide possessor and occupant
of the subject lots in the manner and for the period prescribed by law, to wit:

The possession contemplated by Section 48 (b) of [CA] 141 is actual, not fictional or constructive. In Carlos v.
Republic of the Philippines, the Court explained the character of the required possession, as follows:
Page 66

The law speaks of possession and occupation. Since these words are separated by the conjunction and, the clear
intention of the law is not to make one synonymous with the other. Possession is broader than occupation
because it includes constructive possession. When, therefore, the law adds the word occupation, it seeks to
delimit the all-encompassing effect of constructive possession. Taken together with the words open,
continuous, exclusive and notorious, the word occupation serves to highlight the fact that for an applicant
to quality, his possession must not be a mere fiction. Actual possession of a land consists in the
manifestation of acts of dominion over it of such a nature as a party would naturally exercise over his
own property.52(Emphases in the original)

Oddly enough, while in its Decision the RTC appeared to have granted petitioner's Application, said Decision
seemed to have indulged in a bit of non-sequitur when it said that "[petitioner] and his predecessors were not in
actual possession of the [subject lots] all the time" x x x. 53 Simply said, the CA effectively ruled that since
petitioner failed to prove that he or his predecessors-in-interest had indeed performed the required acts of
possession and occupation, or specific acts of dominion over the subject lots, it stands to reason that registration
thereof in his name cannot be allowed.

Tancredo also failed to establish possession and occupation over the disputed portions in the manner and for
the period required by law

At this juncture, we shall revisit the uniform finding by both the RTC and the CA, which in effect upheld
Tancredo's right to register the disputed portions in his name (as an exception to the settled rule that questions of
fact are proscribed in a Rule 45 petition since a correct evaluation of the facts will yield a different
conclusion).54

First off, Tancredo failed to show that his or his predecessor-in interest's possession and occupation over the
disputed portions had been under a bona fide claim of ownership since June 12, 1945, or earlier. We are inclined
to agree with petitioner's posture that Tancredo failed to adduce clear and convincing evidence which
established the origin or antecedents of Tomas's straightforward possession and occupation, or claim of
ownership, over the disputed portions. Consider the following exchange/s between/among Tancredo, the
petitioner, and the Court —

More than this, Tancredo did not present clear, convincing evidence to support his claim that the disputed
portions were in fact transferred to him by his father, Tomas. Tancredo merely testified that the disputed
portions were given to him solely by Tomas, an act that was allegedly consented to by his siblings. Thus

Nonetheless, there is nothing in the records to support or confirm Tancredo's claim that the property was in fact
deeded over to him by his father, Tomas.

In Buenaventura v. Pascual,57 this Court affirmed the lower courts' dismissal of the claims for registration of
imperfect titles because, among others, both the applicant and oppositors failed to adduce evidence as to how
they acquired the subject property from their respective predecessors-in-interest, i.e., whether by succession or
by donation or by some other mode. Furthermore, we stressed therein that the applicant failed to prove the
manner by which her predecessors-in-interest possessed the subject property.
Page 67

Then, again, Tancredo also failed to establish that he and his predecessors-in-interest had/have been in open,
continuous, exclusive and notorious possession and occupation of the disputed portions since June 12, 1945, or
prior thereto.

If anything, the records showed that Tancredo merely submitted photocopies of four tax declarations which
were attached as annexes to his Opposition. These included the 1945 TD 3549 as adverted to by the CA in the
records58 pertaining to a 3.6237-hectare lot in an unstated cadastral lot, TD 0548 covering an 813-hectare lot in
Cadastral Lot No. 1676-A,59 TD 0549 for a 2.3642-hectare lot in Cadastral Lot No. 1677-A, 60 and TD 0550
concerning a 1.1782-hectare lot in Cadastral Lot No. 566-A. 61 All four TDs are in Tomas's name, without copies
of the dorsal portions thereof, and bearing annotations stating either "[c]ontested by [Asuncion]" or "[a]lso
declared in the name of [Asuncion] or [Ponciano]".

It would thus appear that Tancredo had erected his opposition/claim to the lots in question upon the said
photocopies of four tax declarations whose authenticity or genuineness is open to the most serious doubts. And,
even on the assumption that the said tax declarations are in fact authentic and genuine, still it is settled that tax
declarations are not conclusive proof of ownership. If anything, tax declarations are merely corroborative of a
person's claim of possession. More than that, as elsewhere indicated, intermittent and irregular tax payments, as
in this case, do not really provide strong support for a claim of ownership or possession.62

It is axiomatic of course that "[i]t is the policy of the State to encourage and promote the distribution of
alienable public lands as a spur to economic growth and in line with the social justice ideal enshrined in the
Constitution. At the same time, the law imposes stringent safeguards upon the grant of such resources lest
they fall into the wrong hands to the prejudice of the national patrimony."63 This ruling controls the present
case.

As a final note: All of the foregoing discussion showed that the issues raised in this case have all been
previously resolved and determined by settled jurisprudence; hence, there is no reason to grant petitioner's
prayer for this case to be referred to or heard by the Court En Banc, as this is not a case of first impression at all.

WHEREFORE, the Petition is hereby DENIED. We AFFIRM with MODIFICATION the August 13, 2009
Decision and the February 22, 2012 Resolution of the Court of Appeals in CA-G.R. CV No. 02296 in that the
award by the Regional Trial Court of Barotac Viejo, Iloilo, Branch 66 in LRC Case No. 02-195 of Lot No. 1677
with an area of 2.3642 hectares and Lot No. 566 with an area of 1.1782 hectares, both in favor of respondent
Tancredo Vargas, is OVERTURNED and NULLIFIED.

SO ORDERED.

Republic v. Nicolas, GRNo. 181435, Oct.2, 2017


REPUBLIC OF THE PHILIPPINES, Petitioner vs. ROSARIO L. NICOLAS, Respondent

Facts: This is a Petition for Review on Certiorari filed by the Republic of the Philippines to assail the Court of
Appeals and RTC Decision. The CA affirmed the Regional Trial Court (RTC) Decision, which granted the
Petition filed by respondent Rosario L. Nicolas for the registration of title to a parcel of land located in
Barangay (Brgy.) San Isidro, Rodriguez, Rizal. 6 The appellate court agreed with the conclusion of the RTC that
respondent had convincingly established her ownership of the land and was therefore entitled to judicial
confirmation and registration of title.
Page 68

On 22 March 1996, respondent filed a Petition before the RTC of San Mateo, Rizal, seeking to register her title
over Lot 2 of Survey Plan Psu- 213331, a parcel of land located in Brgy. San Isidro, Rodriguez, Rizal, with an
area of 118,448 square meters. She asserted that she was entitled to confirmation and registration of title, as she
had been in "natural, open, public, adverse, continuous, uninterrupted" possession of the land in the concept of
an owner since October 1964. Petitioner Republic of the Philippines filed an Opposition to the Petition. It
contended that:
a) neither respondent nor her predecessors-in-interest had been in open, continuous, exclusive and
notorious possession of the land since 12 June 1945;
b) the Tax Declarations attached to the Petition did not constitute sufficient evidence of the acquisition or
possession of the property;
c) respondent failed to apply for registration of title within six months from 16 February 1976 as required
by Presidential Decree No. (P.D.) 892; and
d) the land in question was part of the public domain and not subject to private appropriation.
During the trial, the respondent presented a report from CENRO stating that the land "appears to be not covered
by any public land application nor embraced by any administrative title. However, the entry with respect to
whether the land was within the alienable and disposable zone was left blank
Moreover, respondent presented three witnesses to prove her right to register the property: Leonila Alfaro, her
daughter and attorney-in-fact, who testified that respondent had occupied the land since 1940 and had paid the
real estate taxes therefor since 1969;26 Santiago Eulin, who was allegedly hired by respondent to plant
vegetables and fruit trees on the land and who acted as its caretaker since 1942; and Roberto M. Valdez of the
LRA, who identified the original tracing cloth plan for the property.

The RTC granted the Petition and ordered the issuance of a Decree of Registration in favor of respondent. It
declared that she had acquired ownership of the land by way of open, continuous, public, adverse, actual and
bona fide possession in the concept of an owner since 1940 without specifying the provision in PD 1529 from
which the Decision is based.

The CA affirmed the RTC decision with modification. According to the appellate court, the evidence presented
proved that respondent had occupied the land since 1940. Even assuming that her possession of the property
started only when she had it privately surveyed in 1964, she had been its occupant for more than 30 years. As
such, she was still entitled to registration of title under Section 14(2) of P.D. 1529.

Hence, this petition.

Issues: Based on the submissions of the parties and the Decisions of the CA and the RTC, two issues are
presented for resolution by this Court:
1) Whether the CA erroneously allowed the judicial confirmation of respondent's title to the property under
Section 14(1) of P.D. 1529; and
2) Whether the CA erred in declaring that respondent is likewise entitled to registration of title based on
ownership by acquisitive prescription under Section 14(2) of P.D. 1529.
CIVIL LAW: The SC grants the Petition.

Ruling: Applications for registration of title to land, both public and private, are governed by Section 14 of P.D.
1529: SECTION 14. Who May Apply. The following persons may file in the proper Court of First Instance an
application for registration of title to land, whether personally or through their duly authorized representatives:
1) Those who by themselves or through their predecessors-in-interest have been in open, continuous,
exclusive and notorious possession and occupation of alienable and disposable lands of the public
domain under a bona fide claim of ownership since June 12, 1945, or earlier.
2) Those who have acquired ownership of private lands by prescription under the provisions of existing
laws.
Page 69

3) Those who have acquired ownership of private lands or abandoned river beds by right of accession or
accretion under the existing laws.
4) Those who have acquired ownership of land in any other manner provided for by law. Where the land is
owned in common, all the co-owners shall file the application jointly.
Where the land has been sold under pacto de retro, the vendor a retro may file an application for the original
registration of the land, provided, however, that should the period for redemption expire during the pendency of
the registration proceedings and ownership to the property consolidated in the vendee a retro, the latter shall be
substituted for the applicant and may continue the proceedings.

A trustee on behalf of his principal may apply for original registration of any land held in trust by him, unless
prohibited by the instrument creating the trust.
Respondent has failed to prove that the property is alienable and disposable agricultural land that may be
registered under Section 14(1) of P.D. 1529.

Section 14(1) of P.D. 1529 governs applications for registration of alienable and disposable lands of the public
domain. This paragraph operationalizes Section 48(b) of Commonwealth Act No. 141 as amended.
This provision grants occupants of public land the right to judicial confirmation of their title. Based on these
two provisions and other related sections of C.A. 141, registration is allowed provided the following requisites
have been complied with:
1. The applicant is a Filipino citizen.
2. The applicant, by himself or through his predecessors-in-interest, has been in open, continuous,
exclusive and notorious possession and occupation of the property since 12 June 1945.
3. The property has been declared alienable and disposable as of the filing of the application.
4. If the area applied for does not exceed 12 hectares, the application should be filed by 31 December
2020.
As earlier stated, respondent failed to establish the third requisite, i.e., that the property subject of the
application is alienable and disposable agricultural land. The Court has emphasized in a long line of cases that
an applicant for registration under Section 14( 1) must prove that the subject property has been classified as
alienable and disposable agricultural land by virtue of a positive act of the Executive Department.
Alienable and disposable lands of the State fall into two categories, to wit: (a) patrimonial lands of the State, or
those classified as lands of private ownership under Article 425 of the Civil Code, without limitation; and (b)
lands of the public domain, or the public lands as provided by the Constitution, but with the limitation that the
lands must only be agricultural.

A positive act of the Government is necessary to enable such reclassification, and the exclusive prerogative to
classify public lands under existing laws is vested in the Executive Department, not in the courts.
Thus, until the Executive Department exercises its prerogative to classify or reclassify lands, or until Congress
or the President declares that the State no longer intends the land to be used for public service or for the
development of national wealth, the Regalian Doctrine is applicable.

Here, respondent failed to establish, by the required evidence, that the land sought to be registered has been
classified as alienable or disposable land of the public domain.

The records of this case merely bear certifications from the DENR-CENRO, Region IV, Antipolo City, stating
that no public land application or land patent covering the subject lots is pending nor are the lots embraced by
any administrative title. Said CENRO certifications, however, do not even make any pronouncement as to the
alienable character of the lands in question for they merely recognize the absence of any pending land patent
application, administrative title, or government project being conducted thereon.
But even granting that they expressly declare that the subject lands form part of the alienable and disposable
lands of the public domain, these certifications remain insufficient for purposes of granting respondent's
Page 70

application for registration. As constantly held by this Court, it is not enough for the CENRO to certify that a
land is alienable and disposable.

The applicant for land registration must prove that the DENR Secretary had approved the land classification and
released the land of the public domain as alienable and disposable, and that the land subject of the application
for registration falls within the approved area per verification through survey by the PENRO or CENRO.
Unfortunately for respondent, the evidence submitted clearly falls short of the requirements for original
registration in order to show the alienable character of the lands subject herein.

Respondent has failed to prove that the land subject of the application is part of the patrimonial property of the
State that may be acquired by prescription under Section 14(2) of P.D. 1529.

As previously noted, the CA also allowed the registration of the property under Section 14(2) of P.D. 1529
based on the following findings:
1) the property is "private in nature" as shown by the fact that it is "covered by a private survey";
2) respondent had occupied the land continuously for more than 30 years from the time of the filing of the
application in 1996; and
3) the land is not covered by any public land application based on the DENR-CENRO Certifications
submitted by respondent.
The SC did not agree the CA ratiocination.

By express provision of the law, only private lands that have been acquired by prescription under existing laws
may be the subject of applications for registration under Section 14(2). The starting point of the Court's
evaluation must, therefore, be whether the property involved falls within the scope of the paragraph.

Under the Civil Code, all things within human commerce are generally susceptible of prescription. Properties of
the public dominion, or those owned by the State, are expressly excluded by law from this general rule, unless
they are proven to be patrimonial in character.

Only private property can be acquired by prescription. Property of public dominion is outside the commerce of
man. It cannot be the object of prescription because prescription does not run against the State in its sovereign
capacity. However, when property of public dominion is no longer intended for public use or for public service,
it becomes part of the patrimonial property of the State. When this happens, the property is withdrawn from
public dominion and becomes property of private ownership, albeit still owned by the State. The property is
now brought within the commerce of man and becomes susceptible to the concepts of legal possession and
prescription.

To establish that the land subject of the application has been converted into patrimonial property of the State, an
applicant must prove the following:

1) The subject property has been classified as agricultural land.


2) The property has been declared alienable and disposable.
3) There is an express government manifestation that the property is already patrimonial, or is no longer
retained for public service or the development of national wealth.
It must be emphasized that without the concurrence of these three conditions, the land remains part of public
dominion and thus incapable of acquisition by prescription.

Here, the records show that respondent has failed to allege or prove that the subject land belongs to the
patrimonial property of the State. As earlier discussed, the evidence she has presented does not even show that
Page 71

the property is alienable and disposable agricultural land. She has also failed to cite any government act or
declaration converting the land into patrimonial property of the State.

Contrary to the ruling of the CA, the DENR-CENRO Certifications submitted by respondent are not enough;
they cannot substitute for the three conditions required by law as proof that the land may be the subject of
prescription under the Civil Code. For the same reason, the mere conduct of a private survey of a property -
even with the approval of the Bureau of Lands - does not convert the lot into private land or patrimonial
property of the State. Clearly, the appellate court erred when it relied on the survey to justify its conclusion that
the land is private in nature.

Considering the absence of sufficient evidence that the subject land is a patrimonial property of the State, we
must consider it part of public dominion and thus immune from acquisitive prescription.

REMEDIAL LAW
While a petition for review on certiorari under Rule 45 is generally limited to a review of errors of law, the
Court may conduct its own review of the evidence if the findings of the lower courts are bereft of legal and
factual bases. In this case, the conclusions of the RTC and the CA are not only contradicted by the evidence on
record; they are likewise contrary to law and jurisprudence. As a result, the Court is constrained to set aside
these pronouncements.
Republic v. Sese, GRNo. 185092, June 4, 2014
REPUBLIC OF THE PHILIPPINES, Petitioner, vs. CORAZON C. SESE and FE C. SESE, Respondents

Facts: Respondents filed with the MTC an application for original registration of land over a parcel of land with
an area of 10, 792 square meters, situated in Barangay Sto. Cristo, Municipality of Pulilan, Province of Bulacan,
and more particularly described as Lot 11247, Cad. 345, PulilanCadastre, under Plan No. AP-03-004226.

Respondents alleged that on July 22, 1972, they acquired, through a donation inter vivos from their mother,
Resurreccion L. Castro (Resurreccion), the subject agricultural land; that they, through their predecessors-in-
interest, had been in possession of the subject property; and that the property was not within a reservation.

In support of their application, respondents submitted among others a survey plan approved by the Regional
Technical Director of the Land Management Service, Region III of the DENR stating that the land subject of the
survey was alienable and disposable land, and as certified to by the Bureau of Forestry on March 1, 1927, was
outside of any civil or military reservation. On the lower portion of the plan, there was a note stating that a deed
of absolute sale over the subject property was executed by a certain Luis Santos and Fermina Santos (the
Santoses) in favor of Resurreccion on October 4, 1950.

On the lower portion of the survey plan, a note stated, that: "This survey is inside the alienable and disposable
area as per Project No. 20 LC Map No. 637 certified by the Bureau of Forestry on March 1, 1927. It is outside
any civil or military reservation." Finding the application sufficient in form and substance, the MTC issued the
Order, dated October 10, 2002, setting the case for hearing with the corresponding publication. After
compliance with all the requirements of the law regarding publication, mailing and posting, hearing on the
merits of the application followed.

During the trial, respondent Corazon C. Sese (Corazon) testified on their claim over the subject lot. Thereafter,
respondents submitted their formal offer of evidence, after which the evidence offered were admitted by the
MTC in the Order, without objection from the public prosecutor.

The OSG did not present any evidence to oppose the application.
Page 72

The MTC ordered the registration of the subject property in the name of respondents.

Later, the OSG interposed an appeal with the CA. In its brief, the OSG presented the following assignment of
errors: a) only alienable lands of the public domain occupied and possessed in concept of owner for a period of
at least thirty (30) years is entitled to confirmation of title; and b) respondents failed to prove specific acts of
possession.

The OSG argued that there was no proof that the subject property was already segregated from inalienable lands
of the public domain. Verily, it was only from the date of declaration of such lands as alienable and disposable
that the period for counting the statutory requirement of possession would start.

Also, there was absolutely no proof of respondents supposed possession of the subject property. Save for the
testimony of Corazon that "at present, the worker of (her) mother is occupying the subject property," there was
no evidence that respondents were actually occupying the subject tract of land or that they had introduced
improvement thereon.

The CA however, affirming the judgment of the MTC ordering the registration of the subject property in the
name of respondents. The CA was of the view that the doctrine of constructive possession was applicable.
Respondents acquired the subject property through a donation inter vivos executed on July 22, 1972 from their
mother. The latter acquired the said property from the Santoses on October 4, 1950 by virtue of a deed of
absolute sale. Further, respondent Corazon testified that a small hut was built on the said land, which was
occupied by the worker of her mother. Moreover, neither the public prosecutor nor any private individual
appeared to oppose the application for registration of the subject property.

The CA also stated that respondents claim of possession over the subject property was buttressed by the Tax
Declaration No. 99-19015-01557 "in the name of Corazon Sese and Fe Sese, minor, representing their mother
Resurreccion Castro, as her Natural Guardian"; the official receipt of payment of real property tax over the
subject property; and the certificate from the Office of the Municipal Treasurer of Pulilan, stating that the
registered owner of a property under Tax Declaration No. 99-19015-01557 were respondents.

The CA added that although tax declaration or realty tax payments of property were not conclusive evidence of
ownership, nevertheless, they were good indicia of possession in the concept of owner. Hence, the OSG filed
this petition.

Issues: Did the Court of Appeals err on a question of law in ruling that the approved survey plan
identified by one of the respondents is proof that the subject land is alienable and disposable? Did the
Court of Appeals err on a question of law in granting the application for registration?

HELD: In Republic v. Espinosa, G.R. No. 171514, July 18, 2012 citing Republic v. Sarmiento 547 Phil. 157,
166167 (2007) and Menguito v. Republic, 401 Phil. 274, 287-288 (2000), the Court reiterated the rule that that a
notation made by a surveyor-geodetic engineer that the property surveyed was alienable and disposable was not
the positive government act that would remove the property from the inalienable domain and neither was it the
evidence accepted as sufficient to controvert the presumption that the property was inalienable. Thus:

To discharge the onus, respondent relies on the blue print Copy of the conversion and subdivision plan approved
by the DENR Center which bears the notation of the surveyor-geodetic engineer that "this survey is inside the
alienable and disposable area, Project No. 27-B. L.C. Map No. 2623, certified on January 3, 1968 by the Bureau
of Forestry."
Page 73

Menguito v. Republic teaches, however, that reliance on such annotation to prove that the lot is alienable is
insufficient and does not constitute incontrovertible evidence to overcome the presumption that it remains part
of the inalienable public domain.

"To prove that the land in question formed part of the alienable and disposable lands of the public domain,
petitioners relied on the printed words which read: This survey plan is inside Alienable and Disposable Land
Area, Project No. 27-B as per L.C. Map No. 2623, certified by the Bureau of Forestry on January 3, 1968,
appearing on Exhibit "E" (Survey Plan No. Swo-13-000227).

This proof is not sufficient. Section 2, Article XII of the 1987 Constitution, provides: "All lands of the public
domain, waters, minerals, coal, petroleum, and other mineral oils, all forces of potential energy, fisheries, forests
or timber, wildlife, flora and fauna, and other natural resources are owned by the State..."

For the original registration of title, the applicant (petitioners in this case) must overcome the presumption that
the land sought to be registered forms part of the public domain. Unless public land is shown to have been
reclassified or alienated to a private person by the State, it remains part of the inalienable public domain.
Indeed, "occupation thereof in the concept of owner, no matter how long, cannot ripen into ownership and be
registered as a title." To overcome such presumption, incontrovertible evidence must be shown by the applicant.
Absent such evidence, the land sought to be registered remains inalienable.

In the present case, petitioners cite a surveyor geodetic engineers notation in Exhibit "E" indicating that the
survey was inside alienable and disposable land. Such notation does not constitute a positive government act
validly changing the classification of the land in question. Verily, a mere surveyor has no authority to reclassify
lands of the public domain. By relying solely on the said surveyors assertion, petitioners have not sufficiently
proven that the land in question has been declared alienable."

Thus, under Section 14(2) of P.D. No. 1529, for acquisitive prescription to commence and operate against the
State, the classification of land as alienable and disposable alone is not sufficient. The applicant must be able to
show that the State, in addition to the said classification, expressly declared through either a law enacted by
Congress or a proclamation issued by the President that the subject land is no longer retained for public service
or the development of the national wealth or that the property has been converted into patrimonial.
Consequently, without an express declaration by the State, the land remains to be a property of public dominion
and, hence, not susceptible to acquisition by virtue of prescription. The classification of the subject property as
alienable and disposable land of the public domain does not change its status as property of the public dominion
under Article 420(2) of the Civil Code. It is still insusceptible to acquisition by prescription.

For the above reasons, the respondents cannot avail of either Section 14 (1) or 14 (2) of P.O. No. 1529. Under
Section 14 (1), respondents failed to prove (a) that the property is alienable and disposable; and (b) that their
possession of the property dated back to June 12, 1945 or earlier. Failing to prove the alienable and disposable
nature of the subject land, respondents all the more cannot apply for registration by way of prescription pursuant
to Section 14 (2) which requires possession for 30 years to acquire or take. Not only did respondents need to
prove the classification of the subject land as alienable and disposable, but also to show that it has been
converted into patrimonial. As to whether respondents were able to prove that their possession and occupation
were of the character prescribed by law, the resolution of this issue has been rendered unnecessary by the
foregoing considerations. GRANTED.
City Mayor v. Ebio, GR No. 178411, June 23, 2010
OFFICE OF THE CITY MAYOR OF PARAÑAQUE CITY, OFFICE OF THE CITY ADMINISTRATOR OF
PARAÑAQUE CITY, OFFICE OF THE CITY ENGINEER OF PARAÑAQUE CITY, OFFICE OF THE CITY
PLANNING AND DEVELOPMENT COORDINATOR, OFFICE OF THE BARANGAY CAPTAIN AND
Page 74

SANGGUNIANG PAMBARANGAY OF BARANGAY VITALEZ, PARAÑAQUE CITY, TERESITA A.


GATCHALIAN, ENRICO R. ESGUERRA, ERNESTO T. PRACALE, JR., MANUEL M. ARGOTE,
CONRADO M. CANLAS, JOSEPHINE S. DAUIGOY, ALLAN L. GONZALES, ESTER C. ASEHAN,
MANUEL A. FUENTES, and MYRNA P. ROSALES, Petitioners, vs. MARIO D. EBIO AND HIS
CHILDREN/HEIRS namely, ARTURO V. EBIO, EDUARDO V. EBIO, RENATO V. EBIO, LOURDES E.
MAGTANGOB, MILA V. EBIO, and ARNEL V. EBIO, Respondents.

Facts: Respondents claim to be absolute owners of a 406 sqm. parcel of land in Parañaque City covered by Tax
in the name of respondent Mario D. Ebio. Said land was an accretion of Cut-cut creek.
Respondents assert that the original occupant and possessor land was their great grandfather, Jose Vitalez,
which was given to his son, Pedro Valdez, in 1930. From then on, Pedro continuously and exclusively occupied
and possessed the said lot. In 1966, after executing an affidavit declaring possession and occupancy. He also
paid taxes for the land.

Meanwhile, in 1961, respondent Mario Ebio married Pedro’s daughter, Zenaida. In April 1964 and in October
1971, Mario Ebio secured building permits from the Parañaque municipal office for the construction of their
house within the land. On April 21, 1987, Pedro transferred his rights over the land in favor of Ebio.
OnMarch 30, 1999, the Office of theSangguniang Barangayof Vitalez passed Resolution No. 08, series of 1990
seeking assistance from the City Government of Parañaque for the construction of an access road along Cut-cut
Creek located in the said barangay. The proposed road will run from Urma Drive to the main road of Vitalez
Compound traversing the lot occupied by the respondents. Respondents immediately opposed and the project
was suspended.

In January 2003, however, respondents were surprised when several officials from the barangay and the city
planning office proceeded to cut eight (8) coconut trees planted on the said lot. On March 28, 2005, the City
Administrator sent a letter to the respondents ordering them to vacate the area within the next thirty (30) days,
or be physically evicted from the said property. Respondents sent a reply, asserting their claim over the subject
property and expressing intent for a further dialogue. The request remained unheeded. Threatened of being
evicted, respondents went to the RTC of Parañaque City on April 21, 2005 and applied for a writ of preliminary
injunction against petitioners.

Issue: Whether or not the State may build on the land in question.

Held: No.

It is an uncontested fact that the subject land was formed from the alluvial deposits that have gradually settled
along the banks of Cut-cut creek. This being the case, the law that governs ownership over the accreted portion
is Article 84 of the Spanish Law of Waters of 1866, which remains in effect, in relation to Article 457 of the
Civil Code.
ART. 84. Accretions deposited gradually upon lands contiguous to creeks, streams, rivers, and lakes, by
accessions or sediments from the waters thereof, belong to the owners of such lands. Art. 457. To the owners of
lands adjoining the banks of rivers belong the accretion which they gradually receive from the effects of the
current of the waters.

It is therefore explicit from the foregoing provisions that alluvial deposits along the banks of a creek do not
form part of the public domain as the alluvial property automatically belongs to the owner of the estate to which
it may have been added. The only restriction provided for by law is that the owner of the adjoining property
must register the same under the Torrens system; otherwise, the alluvial property may be subject to acquisition
through prescription by third persons.
Page 75

In contrast, properties of public dominion cannot be acquired by prescription. No matter how long the
possession of the properties has been, there can be no prescription against the State regarding property of public
domain. Even a city or municipality cannot acquire them by prescription as against the State.
Hence, while it is true that a creek is a property of public dominion, the land which is formed by the gradual and
imperceptible accumulation of sediments along its banks does not form part of the public domain by clear
provision of law.
Republic v. CA and Republic Real Estate corporation, GR No. 103882, Nov. 25, 1 998
REPUBLIC OF THE PHILIPPINES, petitioner, vs. THE HONORABLE COURT OF APPEALS AND
REPUBLIC REAL ESTATE CORPORATION, respondents, CULTURAL CENTER OF THE
PHILIPPINES, intervenor.

Facts: On June 22, 1957, RA 1899 was approved granting authority to all municipalities and chartered cities to
undertake and carry out at their own expense the reclamation by dredging, filling, or other means, of any
foreshore lands bordering them, and to establish, provide, construct, maintain and repair proper and adequate
docking and harbor facilities as such municipalities and chartered cities may determine in consultation with the
Secretary of Finance and the Secretary of Public Works and Communications.

Pursuant to the said law, Ordinance No. 121 was passed by the city of Pasay for the reclamation of foreshore
lands within their jurisdiction and entered into an agreement with Republic Real Estate Corporation for the said
project.

Republic questioned the agreement. It contended, among others, that the agreement between RREC and the City
of Pasay was void for the object of the contract is outside the commerce of man, it being a foreshore land.
Pasay City and RREC countered that the object in question is within the commerce of man because RA 1899
gives a broader meaning on the term “foreshore land” than that in the definition provided by the dictionary.

RTC rendered judgment in favour of Pasay City and RREC, and the decision was affirmed by the CA with
modifications.

Issue: Whether or not the term “foreshore land” includes the submerged area. Whether or not “foreshore
land” and the reclaimed area is within the commerce of man.

Held:
The Court ruled that it is erroneous and unsustainable to uphold the opinion of the respondent court that the
term “foreshore land” includes the submerged areas. To repeat, the term "foreshore lands" refers to:

The strip of land that lies between the high and low water marks and that is alternately wet and dry according to
the flow of the tide.

A strip of land margining a body of water (as a lake or stream); the part of a seashore between the low-water
line usually at the seaward margin of a low-tide terrace and the upper limit of wave wash at high tide usually
marked by a beach scarp or berm. (Webster's Third New International Dictionary)

The duty of the court is to interpret the enabling Act, RA 1899. In so doing, we cannot broaden its meaning;
much less widen the coverage thereof. If the intention of Congress were to include submerged areas, it should
have provided expressly. That Congress did not so provide could only signify the exclusion of submerged areas
from the term “foreshore lands.”
Page 76

It bears stressing that the subject matter of Pasay City Ordinance No. 121, as amended by Ordinance No. 158,
and the Agreement under attack, have been found to be outside the intendment and scope of RA 1899, and
therefore ultra vires and null and void.
Republic v. East Silverlane Realty Development Corporation, GR No. 186961, Feb.20, 2012
REPUBLIC OF THE PHILIPPINES, Petitioner, vs. EAST SILVERLANE REALTY DEVELOPMENT
CORPORATION, Respondent.

Facts:
The respondent filed with the RTC an application for land registration, covering a parcel of land identified as
Lot 9039 of Cagayan Cadastre, situated in El Salvador, Misamis Oriental and with an area of 9,794 square
meters. The respondent purchased the portion of the subject... property consisting of 4,708 square meters (Area
A) from Francisca Oco pursuant to a Deed of Absolute Sale dated November 27, 1990 and the remaining
portion consisting of 5,086 square meters (Area B) from Rosario U. Tan Lim, Nemesia Tan and Mariano U. Tan
pursuant to a Deed of Partial Partition with Deed of Absolute Sale dated April 11, 1991. It was claimed that the
respondent's predecessors-in-interest had been in open, notorious, continuous and exclusive possession of the
subject property since June 12, 1945.

After hearing the same on the merits, the RTC issued on August 27, 2004 a Decision, granting the respondent's
petition for registration of the land in question... the CA affirmed the RTC's August 27, 2004 Decision. In the
case at bench, petitioner-appellee has met all the requirements. Anent the first requirement, both the report and
certification issued by the Department of Environment and Natural Resources (DENR) shows that the subject
land was within the alienable and disposable zone... classified under BF Project [N]o. 8 Blk. I, L.C. Map [N]o.
585 and was released and certified as such on December 31, 1925.

Indubitably, both the DENR certification and report constitute a positive government act, an administrative
action, validly classifying the land in question. It is a settled rule that the classification or re-classification of
public lands into alienable or disposable, mineral... or forest land is now a prerogative of the Executive
Department of the government.

Accordingly, the certification enjoys a presumption of regularity in the absence of contradictory evidence.
certification remains uncontested and even oppositor-appellant Republic itself did not present any evidence to
refute the contents of the said certification. Thus, the alienable and disposable character of the subject land
certified as such as early as December 31, 1925 has been clearly established by the evidence of the... petitioner-
appellee.
Anent the second and third requirements, the applicant is required to prove his open, continuous, exclusive and
notorious possession and occupation of the subject land under a bona fide claim of ownership either since time
immemorial or since June 12, 1945.

According to the petitioner, the respondent did not present a credible and competent witness to testify on the
specific acts of ownership... performed by its predecessors-in-interest on the subject property. The respondent's
sole witness, Vicente Oco, can hardly be considered a credible and competent witness as he is the respondent's
liaison officer and he is not related in any way to the respondent's... predecessors-in-interest. That coconut trees
were planted on the subject property only shows casual or occasional cultivation and does not qualify as
possession under a claim of ownership.

Issues: This Court is confronted with the sole issue of whether the respondent has proven itself entitled to
the benefits of the PLA and P.D. No. 1529 on confirmation of imperfect or incomplete titles.

Ruling:
Page 77

It is a settled rule that an application for land registration must conform to three requisites: (1) the land is
alienable public land; (2) the applicant's open, continuous, exclusive and notorious possession and occupation
thereof must be since June 12, 1945, or... earlier; and (3) it is a bona fide claim of ownership.
This Court resolves to GRANT the petition.

This Court agrees with the respondent that the issue of whether the respondent had presented sufficient proof of
the required possession under a bona fide claim of... ownership raises a question of fact, considering that it
invites an evaluation of the evidentiary record.

This Court's review of the records of this case reveals that the evidence submitted by the respondent fell short of
proving that it has acquired an imperfect title over the subject property under Section 48 (b) of the PLA. The
respondent cannot register the subject property in... its name on the basis of either Section 14 (1) or Section 14
(2) of P.D. No. 1529. It was not established by the required quantum of evidence that the respondent and its
predecessors-in-interest had been in open, continuous, exclusive and notorious possession of the subject...
property for the prescribed statutory period.

Court ruled that possession and occupation of an alienable and disposable public land for the periods provided
under the Civil Code do not automatically convert said property into private property or release it from the
public domain. There... must be an express declaration that the property is no longer intended for public service
or development of national wealth. Without such express declaration, the property, even if classified as alienable
or disposable, remains property of the State, and thus, may not be... acquired by prescription.
For as long as the property belongs to the State, although already classified as alienable or disposable, it remains
property of the public dominion if when it is "intended for some public service or for the development of the...
national wealth".

Without such express declaration, the... property, even if classified as alienable or disposable, remains property
of the public dominion, pursuant to Article 420(2), and thus incapable of acquisition by prescription. It is only
when such alienable and disposable lands are expressly declared by the State to be no... longer intended for
public service or for the development of the national wealth that the period of acquisitive prescription can begin
to run. Such declaration shall be in the form of a law duly enacted by Congress or a Presidential Proclamation in
cases where the President is... duly authorized by law.

While the subject land was supposedly declared alienable and disposable on December 31, 1925 per the April
18, 1997 Certification and July 1, 1997 Report of the Community Environment and Natural Resources Office
(CENRO),[20] the Department of Agrarian Reform (DAR) converted the same from agricultural to industrial
only on October 16, 1990.[21] Also, it was only in 2000 that the Municipality of El Salvador passed a Zoning
Ordinance, including the subject property in the industrial zone.
Therefore, it was only in 1990 that the subject property had been declared patrimonial and it is only then that
the prescriptive period began to run.

The respondent cannot benefit from the alleged possession of its predecessors-in-interest because prior to the...
withdrawal of the subject property from the public domain, it may not be acquired by prescription.

International Hardwood v. University of the Philippines, GR No.52518, Aug. 13, 1991

Summary: International Hardwood was the grantee of a License Agreement effective until 1985. On 1961,
Proc. 791 segregates from the public domain parcels of land and reserved them for use by UP. The land subject
of Hardwood’s timber concession was covered by said Proclamation. On 1964, RA 3990 was enacted fully
ceding ownership over the land described in Proc. 791 to UP. UP sought to collect forestry charges from
Page 78

Hardwood and demanded that the latter subject itself to the control and supervision of UP. Hardwood resisted
and filed a petition for declaratory relief.
The SC held that UP has the right to enjoy and dispose of the thing without other limitations than those
established by law. In this case, that exception is made for Hardwood as licensee or grantee of the concession,
which has been given the license to cut, collect, and remove timber from the area ceded and transferred to UP
until February 1985. However, Hardwood has the correlative duty and obligation to pay the forest charges or
royalties to the new owner, UP

Doctrine: The Philippines relinquished and conveyed its rights over the area to UP. Thus, UP became the owner
of the land, subject only to existing concession. Since there is an express proviso on existing concessions, this
means that the right of Hardwood as a timber licensee must not be affected, impaired, or diminished; it must be
respected BUT insofar as the Government is concerned, all its right as grantor of the license were effectively
assigned, ceded and conveyed to U.P.

Having been effectively segregated and removed from the public domain or from a public forest and, in effect,
converted into a registered private woodland, the authority and jurisdiction of the Bureau of Forestry over it
were likewise terminated. BIR also lost authority to measure the timber cut from the subject area and to collect
forestry charges and other fees thereon because of this full transfer.

Facts: International Hardwood is engaged in the manufacture, processing, and exportation of plywood. It was
granted by the Government an exclusive license for 25 years expiring on Feb 1985 to cut, collect and remove
timber from a timber land in the provinces of Quezon and Laguna.
Sometime on 1961, during the effectivity of the License Agreement, the President issued Executive
Proclamation No. 791. Under this proclamation, certain parcels of land of the public domain in Quezon and
Laguna were withdrawn from sale or settlement and were reserved for the UP College of Agriculture as
experiment station for the college.

On 1964, still during the effectivity of the License Agreement, RA 3990 was enacted establishing a central
experiment station for UP for the colleges of agriculture, veterinary medicine, arts and sciences. Under RA 3990
the land described in Proc. 791 was fully cede to UP, subject to any existing concessions, if any.
On the strength of RA 3990, UP demanded from Hardwood:

1. Payment of forest charges due and demandable under the License Agreement to UP, instead of the BIR
2. That the sale of any timber felled or cut by Hardwood within the land described in RA 3990 be
performed by UP personnel
However, despite repeated demands, Hardwood refused to accede to UP’s demands.
International Hardwood filed before the CFI a petition for declaratory relief seeking a declaration that UP does
NOT have the right to:
1. Supervise and regulate the cutting and removal of timber and other forest products,
2. Scale, measure and seal the timber cut and/or
3. Collect forest charges, reforestation fees and royalties from Hardwood and/or
4. Impose any other duty or burden upon the latter in that portion of its concession covered by a License
Agreement, ceded in full ownership to UP by RA 3990
Hardwood also prayed for an injunction and P100,000 in damages. UP filed its Answer:
1. Interposed affirmative defenses of improper venue and that the petition states no cause of action
2. Set up counterclaim for payment of forest charges on the forest products cut and felled within the area
ceded to UP under RA 3990
CFI DECISION: CFI rendered judgment in favor of Hardwood:
1. RA 3990 does not empower UP to scale, measure, and seal the timber cut by International Hardwood
within the tract of land and collect the corresponding charges prescribed by NIRC
Page 79

2. Dismissed UP’s counterclaim

CA DECISION: Elevated the case to the SC as the case involves purely legal questions.

Issue: WON UP as owner had the right to scale, measure, and seal the timber cut by Hardwood and collect
forestry charges thereon.

Held: YES, by virtue of the full cession of ownership to UP.


I. Arguments of the Parties
A. UP asserts that:
1. Under RA 3990, the Philippines may effect collection of forest charges through UP because the
License Agreement does not expressly provide that they be paid to the BIR
2. UP is vested with administrative jurisdiction over and has ownership over the land in question. Thus,
it acquired full control and benefit of the timber and other resources in the area
3. UP is entitled to the income derived from the tract of land ceded to it by RA 3990
4. UP is duty bound to operate and maintain a central experiment station
5. Supervision of the License Agreement in favor of Hardwood by UP was intended by RA 3990
6. BIR and the Bureau Of Forestry issued specific rulings recognizing the authority of UP to collect
royalties and charges
B. Hardwood contends:
1. UP has not been granted by RA 3990 the authority to collect forest charges or the authority to
supervise the operation of the timber concession
2. Cession of the land was expressly made subject to any concession, if any
3. Rulings of BIR and Bureau of Forestry are incorrect
4. It has acquired vested right to operate the timber concession under the supervision and control of the
Bureau of Forestry
II. Discussion on the effect of the laws
A. The laws:
1. Under Proc. 791 – a parcel of land of the public domain was withdrawn from sale or settlement and
was reserved for the UP College of Agriculture as experiment station, subject to private rights, if any
2. Under RA 3990 – the very same lot referred to in Proc. 791 was ceded fully to UP, subject to any
existing concessions, if any
B. Effect of the laws on the concession of Hardwood:
1. When RA 3990 ceded the property to UP, the Philippines completely removed it from the public
domain and segregated the areas covered by the timber license from the public forest
2. The Philippines relinquished and conveyed its rights over the area to UP
a. Thus, UP became the owner of the land, subject only to existing concession
3. Since there is an express proviso on existing concessions, this means that the right of Hardwood as a
timber licensee must not be affected, impaired, or diminished; it must be respected
4. BUT insofar as the Government is concerned, all its rights as grantor of the license were effectively
assigned, ceded and conveyed to UP
a. Having been effectively segregated and removed from the public domain or from a public forest
and, in effect, converted into a registered private woodland, the authority and jurisdiction of the
Bureau of Forestry over it were likewise terminated
b. BIR also lost authority to measure the timber cut from the subject area and to collect forestry
charges and other fees thereon because of this full transfer.
III. As owner, UP has the right to enjoy and dispose of the thing without other limitations than those established
by law. In this case, that exception is made for Hardwood as licensee or grantee of the concession, which has
been given the license to cut, collect, and remove timber from the area ceded and transferred to UP until
February 1985.
Page 80

A. However, Hardwood has the correlative duty and obligation to pay the forest charges or royalties to the
new owner, UP
B. Thus, the charges should not be paid to the Government but to UP.
C. It follows then that respondent UP is entitled tosupervise, through its duly appointed personnel, the
logging, felling and removal of timber within the area covered by R.A. No. 3990
DISPOSITIVE: Judgment is rendered reversing the decision of the trial court. Thus:
1. Forest charges due from and payable by petitioner for timber cut pursuant to its License Agreement
within the area ceded and transferred to UP pursuant to R.A. No. 3990 shall be paid to UP;
2. UP is entitled to supervise, through its duly appointed personnel, the logging, felling and removal of
timber within the aforesaid area covered by R.A. No. 3990.
Republic v. Go, GR No. 197297, Aug 02, 2017
REPUBLIC OF THE PHILIPPINES, Petitioner, v. SPOUSES DANILO GO AND AMORLINA
GO, Respondents.

DECISION
LEONEN, J.:
Public land remains inalienable unless it is shown to have been reclassified and alienated to a private person. 1

This resolves a Petition for Review assailing the Court of Appeals Decision dated January 21, 2011 and
Resolution dated June 6, 2011 in CA-G.R. CV No. 93000, which affirmed the Decision of the Municipal Trial
Court in Cities dated December 12, 2008 issuing the Decree of Registration for Lot No. 4699-B of Subdivision
Plan Csd-04-022290-D in favor of the Spouses Danilo and Amorlina Go.

On August 26, 2006, respondents Spouses Danilo and Amorlina Go (the Spouses Go) applied for the
registration and confirmation of title over Cadastral Lot No. 4699-B (Lot No. 4699-B), a parcel of land in
Barangay Balagtas, Batangas City covering an area of 1,000 square meters.2

The Spouses Go registered Lot No. 4699-B in their names for taxation purposes. They had paid the real
property taxes, including the arrears, from 1997 to 2006, as shown in Tax Declaration No. 026-04167. 3 They
had also established a funeral parlor, San Sebastian Funeral Homes, on the lot. 4 According to them, there were
no other claimants over the property.5

The Spouses Go claimed to be in an open, continuous, exclusive, notorious, and actual possession of the
property for seven (7) years since they bought it.6 They also tacked their possession through that of their
predecessors-in-interest, as follows:

Sometime in 1945,7 Anselmo de Torres (Anselmo) came to know that his parents, Sergia Almero and Andres de
Torres (the Spouses de Torres),8 owned Lot No. 4699,9 a bigger property where Lot No. 4699-B came from.
According to Anselmo, the Spouses de Torres paid the real property taxes during their lifetime and planted
bananas, mangoes, calamansi, and rice on this lot.10 His mother, Sergia Almero (Sergia), allegedly inherited Lot
No. 4699 from her parents, Celodonio and Eufemia Almero (the Spouses Almero).11

In the 1960s, Anselmo and his siblings inherited Lot No. 4699 from their parents upon their deaths. 12

One of Anselmo's sisters, Cristina Almero de Torres Corlit (Cristina), then built a residential house on Lot No.
4699-B,13 declaring this parcel of land under her name for tax purposes, as evidenced by Tax Declaration No.
026-03492.14 Meanwhile, Anselmo and his other siblings built their homes on another portion of Lot No.
4699.15 Anselmo, who was then 28 years old, started living in the eastern portion from 1966. 16
Page 81

On January 26, 2000, the Spouses Go bought Lot No. 4699-B from the previous owners, siblings Anselmo,
Bernardo Almero de Torres, Leonila Almero de Torres Morada, and Cristina, as evidenced by a Deed of
Absolute Sale.17

On August 26, 2006, the Spouses Go (respondents) applied for the registration and confirmation of title of Lot
No. 4699-B.18 They attached the Report dated January 31, 2007 of Special Land Investigator I Ben Hur
Hernandez (Hernandez) and the Certification dated January 29, 2008 of Forester I Loida Maglinao (Maglinao)
of the Batangas City Community Environment and Natural Resources Office (CENRO) of the Calamba,
Laguna, Batangas, Rizal, and Quezon (CALABARZON) Region of the Department of Environment and Natural
Resources (DENR).19

Hernandez's January 31, 2007 Report and Maglinao's January 29, 2008 Certification stated that the property was
located in an alienable and disposable zone20 since March 26, 1928, under Project No. 13, Land Classification
Map No. 718.21 No patent or decree was previously issued over the property.22

On November 3, 2006, the Republic of the Philippines (petitioner) opposed respondents' application for
registration for the following reasons: 1) Lot No. 4699-B was part of the public domain; 2) neither the Spouses
Go nor their predecessors-in-interest had been in open, continuous, exclusive, and notorious possession and
occupation of the property since June 12, 1945 or even before then; 3) the tax declaration and payment were not
competent or sufficient proof of ownership, especially considering that these were relatively recent. 23

Anselmo and his siblings had no proof of their inheritance. He claimed that the office having custody of the
documentary proof of their inheritance was burned 24 and they no longer had the original copy of the
documents.25

In the Decision26 dated December 12, 2008, the Municipal Trial Court in Cities confirmed the title of the lot in
the name of the Spouses Go. The dispositive portion read:
Considering that the applicants have duly established essential facts in support of the application, the Court
hereby confirms title to Lot 4699-B, Cad 264 Batangas Cadastre covered in approved plan Csd-04-22290-D,
containing an area of ONE THOUSAND (1,000) SQUARE METERS situated at Barangay Balagtas, Batangas
City in the name of Spouses Danilo Go and Amorlina A. Go, of legal age, Filipino and residents of San Jose
Subdivision, Barangay San Sebastian, Lipa City.

Once the Decision becomes final, let the corresponding Decree of Registration be issued.

SO ORDERED.27
Petitioner appealed directly to the Court of Appeals. In the Decision 28 dated January 21, 2011, the Court of
Appeals denied the appeal:chanRoblesvirtualLawlibrary
WHEREFORE, premises considered, the appeal is DENIED. The assailed Decision, dated December 12, 2008,
of the Municipal Trial Court in Cities (MTCC), Branch 2, Pallocan West, Batangas City in Land Registration
Case No. 2006-162, is AFFIRMED.

No pronouncement as to costs.

SO ORDERED.29
Petitioner filed its Motion for Reconsideration,30 which was denied on June 6, 2011.31

Petitioner elevated32 the case before this Court, arguing that Maglinao testified having investigated only 200
square meters of the 1,000-square-meter land for registration.33 She also admitted that her certification was
Page 82

based on the approved plan and not on the Land Classification Map. She certified the lot only to determine "the
point or monument of the entire or whole area" and not to identify its alienable character. Thus, petitioner
argues that Maglinao's certification should not have been used to determine that the land was alienable and
disposable.34

Petitioner assails respondents' failure to submit a copy of the original classification map that bears the DENR
Secretary's approval and its legal custodian's certification as a true copy.35Petitioner argues that a CENRO
Certification is insufficient to establish that a land applied for registration is alienable. 36

In the Resolution dated August 15, 2011, this Court required respondents to submit a certified true copy of any
Presidential or DENR Secretary's issuance stating Lot No. 4699-B as alienable and disposable.37

In their Compliance38 dated September 25, 2011, the Spouses Go attached a certified photocopy of the CENRO
Certification dated January 29, 2008,39 which this Court noted.40 In the Resolution dated November 14, 2011,
this Court informed the Spouses Go that the CENRO Certification was not the submission required of them. 41

On June 20, 2012, the Spouses Go's counsel, Atty. Jose Amor M. Amorado, was ordered "to show cause why he
should not be disciplinarily dealt with or held in contempt" for failure to comply with this Court's August 15,
2011 Resolution.42 The Spouses Go manifested that they had already complied with this Court's Resolution
through their September 25, 2011 Compliance.43 They re-attached the CENRO Certification dated January 29,
2008.44

On September 24, 2012, this Court resolved45 to require respondents to file their Comment. The Spouses Go
failed to do so, which led this Court to again require 46 their counsel to show cause for their failure to comply
with the September 24, 2012 Resolution.

In their Compliance47 dated August 15, 2013, the Spouses Go informed this Court that they would dispense with
the filing of their Comment.

For resolution before this Court is whether the Court of Appeals erred in issuing the Spouses Go a Decree of
Registration over Lot No. 4699-B.
I

Any application for confirmation of title under Commonwealth Act No. 141 48 already concedes that the land is
previously public.

For a person to perfect one's title to the land, he or she may apply with the proper court for the confirmation of
the claim of ownership and the issuance of a certificate of title over the property.49 This process is also known as
judicial confirmation of title.50

Section 48(b) of Commonwealth Act No. 141, as amended 51 by Presidential Decree No. 1073, 52 states who can
apply for judicial confirmation of title:chanRoblesvirtualLawlibrary Section 48. The following described
citizens of the Philippines, occupying lands of the public domain or claiming to own any such lands or an
interest therein, but whose titles have not been perfected or completed, may apply to the Court of First Instance
[Regional Trial Court] of the province where the land is located for confirmation of their claims and the
issuance of a certificate of title therefor, under the Land Registration Act, to wit:
....
(b) Those who by themselves or through their predecessors in interest have been in the open, continuous,
exclusive, and notorious possession and occupation of agricultural lands of the public domain, under a bona
fide claim of acquisition or ownership, except as against the government, since July twenty-sixth, eighteen
Page 83

hundred and ninety-four, except when prevented by war or force majeure. Those shall be conclusively presumed
to have performed all the conditions essential to a Government grant and shall be entitled to a certificate of title
under the provisions of this chapter. (Emphasis supplied)

Commonwealth Act No. 141 is a special law that applies to agricultural lands of the public domain, not to
forests, mineral lands, and national parks.53 The requisite period of possession and occupation is different from
that of land classification.

In an application for judicial confirmation of title, an applicant already holds an imperfect title to an agricultural
land of the public domain after having occupied it from June 12, 1945 or earlier. 54 Thus, for purposes of
obtaining an imperfect title, the date it was classified is immaterial.55

Classifying a land of the public domain as agricultural is essential only to establish the applicant's "eligibility
for land registration, not the ownership or title over it." 56Heirs of Malabanan v. Republic of the
Philippines57 explained:chanRoblesvirtualLawlibrary

[T]he applicant's imperfect or incomplete title is derived only from possession and occupation since June 12,
1945, or earlier. This means that the character of the property subject of the application as alienable and
disposable agricultural land of the public domain determines its eligibility for land registration, not the
ownership or title over it.58

In Malabanan, the Court En Banc affirmed that June 12, 1945 is the "reckoning point of the
requisite possession and occupation" and not of the land classification as alienable and
disposable:chanRoblesvirtualLawlibrary

[T]he choice of June 12, 1945 as the reckoning point of the requisite possession and occupation was the sole
prerogative of Congress, the determination of which should best be left to the wisdom of the lawmakers. Except
that said date qualified the period of possession and occupation, no other legislative intent appears to be
associated with the fixing of the date of June 12, 1945. Accordingly, the Court should interpret only the plain
and literal meaning of the law as written by the legislators.

[A]n examination of Section 48 (b) of the Public Land Act indicates that Congress prescribed no requirement
that the land subject of the registration should have been classified as agricultural since June 12, 1945, or
earlier.59 (Emphasis supplied)
Thus, the land may be declared alienable and disposable at any time, not necessarily before June 12, 1945. The
moment that the land is declared alienable and disposable, an applicant may then initiate the proceedings for the
judicial confirmation of title.

On the other hand, for the requisite duration of possession, an applicant must have had possession of the
property under a bona fide claim of ownership or acquisition, from June 12, 1945 or earlier. Such possession
must have also been open, continuous, exclusive, and notorious.60

Under Section 11(4)(a) of Commonwealth Act No, 141, the judicial confirmation of imperfect or incomplete
titles, which the law describes as "judicial legalization," allows for agricultural public lands to be disposed of by
the State and acquired by Filipino citizens.61

Meanwhile, Section 14(1) of Presidential Decree No. 1529 62 provides for the procedure to register a title under
the Torrens system:chanRoblesvirtualLawlibrary
Page 84

Section 14. Who may apply. — The following persons may file in the proper Court of First Instance an
application for registration of title to land, whether personally or through their duly authorized representatives:

(1) Those who by themselves or through their predecessors-in-interest have been in open, continuous, exclusive
and notorious possession and occupation of alienable and disposable lands of the public domain under a
bona fide claim of ownership since June 12, 1945, or earlier.
Section 14(1) of Presidential Decree No. 1529 does not vest or create a title to a public land that has already
existed or has been vested under Commonwealth Act No. 141. 63 The procedure of titling under Presidential
Decree No. 1529 "simply recognizes and documents ownership and provides for the consequences of issuing
paper titles."64

Thus, under Section 48(b) of Commonwealth Act No. 141, as amended, and Section 14(1) of Presidential
Decree No. 1529, Filipino citizens applying for the judicial confirmation and registration of an imperfect title
must prove several requisites. First, they must prove that they, by themselves or through their predecessors-in-
interest, have been in open, continuous, exclusive, and notorious possession of the property. Second, it must be
settled that the applicants' occupation is under a bona fide claim of acquisition or ownership since June 12, 1945
or earlier, immediately before the application was filed. Third, it should be established that the land is an
agricultural land of public domain. Finally, it has to be shown that the land has been declared alienable and
disposable.65

The Spouses Go's possession, by themselves or through their predecessors-in-interest, does not meet the
statutory requirements.

The evidence the Spouses Go submitted to prove their required length of possession consist of Anselmo's
testimony, Cristina's sole Tax Declaration, and the Spouses Go's sole Tax Declaration. Other than these pieces of
evidence, the Spouses Go could not support their claim of possession in the concept of an owner, by themselves
or through their predecessors-in-interest, from June 12, 1945 or earlier.

The records do not show that the Spouses Go's predecessors-in-interest fenced the original 3,994-square-meter
Lot No. 4699, claiming it as exclusively theirs or that they introduced improvements on it since June 12, 1945
or earlier. Cristina built a residential house on Lot No. 4699-B 66 when her parents died in the 1960s, 67 while
Anselmo started living in the eastern portion of Lot No. 4699 in 1966 when he was 28 years old. 68 These events
happened at least 15 years after 1945. Moreover, the siblings could not produce any documentary proof of their
alleged inheritance of this land from their parents.69

Apart from Cristina's single tax declaration and the Spouses Go's single tax declaration covering even Cristina's
arrears from 1997 to 2000, nothing in the records shows that the Spouses Go's predecessors-in-interest
religiously paid real property taxes. Payment of real property taxes is a "good indicia of the possession in the
concept of owner for no one in his [or her] right mind would be paying taxes for a property that is not in his [or
her] actual, or at the least constructive, possession."70

Anselmo only gave bare assertions that his parents paid the real property taxes during their lifetime. 71 Neither
did the Spouses Go give any proof of the alleged tax payments of the Spouses de Torres or of Anselmo's
grandparents, the Spouses Almero.

Although not adequate to establish ownership, a tax declaration may be a basis to infer possession. 72 This Court
has highlighted that where tax declaration was presented, it must be the 1945 tax declaration because June 12,
Page 85

1945 is material to the case.73 The specific date must be ascertained; otherwise, applicants fail to comply with
the requirements of the law.74 In Republic v. Manna Properties:75

It is unascertainable whether the 1945 tax declaration was issued on, before or after 12 June 1945. Tax
declarations are issued any time of the year. A tax declaration issued in 1945 may have been issued in December
1945. Unless the date and month of issuance in 1945 is stated, compliance with the reckoning date in
[Commonwealth Act No.] 141 cannot be established.76 (Emphasis in the original)II

Even assuming that there is sufficient evidence to establish their claim of possession in the concept of an owner
since June 12, 1945, the Spouses Go nevertheless failed to prove the alienable and disposable character of the
land.

The 1987 Constitution declares that the State owns all public lands. 77 Public lands are classified into
agricultural, mineral, timber or forest, and national parks. Of these four (4) types of public lands, only
agricultural lands may be alienated. Article XII, Sections 2 and 3 of the Constitution
provide:chanRoblesvirtualLawlibrary

Section 2. All lands of the public domain, waters, minerals, coal, petroleum, and other mineral oils, all forces of
potential energy, fisheries, forests or timber, wildlife, flora and fauna, and other natural resources are owned by
the State. With the exception of agricultural lands, all other natural resources shall not be alienated . . .

Section 3. Lands of the public domain are classified into agricultural, forest or timber, mineral lands, and
national parks. Agricultural lands of the public domain may be further classified by law according to the uses
[to] which they may be devoted. Alienable lands of the public domain shall be limited to agricultural lands . . .
(Emphasis supplied)

Thus, an applicant has the burden of proving that the public land has been classified as alienable and
disposable.78 To do this, the applicant must show a positive act from the government declassifying the land from
the public domain79 and converting it into an alienable and disposable land. 80 "[T]he exclusive prerogative to
classify public lands under existing laws is vested in the Executive Department."81 In Victoria v. Republic:82
To prove that the land subject of the application for registration is alienable, an applicant must establish the
existence of a positive act of the government such as a presidential proclamation or an executive order; an
administrative action; investigation reports of Bureau of Lands investigators; and a legislative act or statute. The
applicant may secure a certification from the government that the lands applied for are alienable and disposable,
but the certification must show that the DENR Secretary had approved the land classification and released the
land of the public domain as alienable and disposable[.]83 (Emphasis supplied, citations omitted)

Section X(1)84 of the DENR Administrative Order No. 1998-24 and Section IX(1) 85 of DENR Administrative
Order No. 2000-11 affirm that the DENR Secretary is the approving authority for "[l]and classification and
release of lands of the public domain as alienable and disposable." Section 4.6 of DENR Administrative Order
No. 2007-20 defines land classification as follows:chanRoblesvirtualLawlibrary

Land classification is the process of demarcating, segregating, delimiting and establishing the best category,
kind, and uses of public lands. Article XII, Section 3 of the 1987 Constitution of the Philippines provides that
lands of the public domain are to be classified into agricultural, forest or timber, mineral lands, and national
parks.

These provisions, read with Victoria v. Republic86 establish the rule that before an inalienable land of the public
domain becomes private land, the DENR Secretary must first approve the land classification into an agricultural
land and release it as alienable and disposable.87 The DENR Secretary's official acts "may be evidenced by an
Page 86

official publication thereof or by a copy attested by the officer having legal custody of the record, or by his
deputy."88

The CENRO or the Provincial Environment and Natural Resources Officer will then conduct a survey to verify
that the land for original registration falls within the DENR Secretary-approved alienable and disposable zone. 89

The CENRO certification is issued only to verify the DENR Secretary issuance through a survey. "Thus, the
CENRO Certification should have been accompanied by an official publication of the DENR Secretary's
issuance declaring the land alienable and disposable." 90 A CENRO certification, by itself, is insufficient to prove
the alienability and disposability of land sought to be registered.91 In Republic v. Lualhati:92
[I]t has been repeatedly ruled that certifications issued by the CENRO, or specialists of the DENR, as well as
Survey Plans prepared by the DENR containing annotations that the subject lots are alienable, do not constitute
incontrovertible evidence to overcome the presumption that the property sought to be registered belongs to the
inalienable public domain. Rather, this Court stressed the importance of proving alienability by presenting a
copy of the original classification of the land approved by the DENR Secretary and certified as true copy by the
legal custodian of the official records.93 (Emphasis supplied)

Here, in its Decision94 dated December 12, 2008, the Court of Appeals concluded that the January 29, 2008
CENRO Certification, which stated that Lot No. 4699-B was within alienable and disposable zone, was
conclusive proof that this land applied for registration was alienable. This Court disagrees.

To establish that a land is indeed alienable and disposable, applicants must submit the application for original
registration with the CENRO certification and a copy of the original classification approved by the DENR
Secretary and certified as a true copy by the legal custodian of the official records. 95

Judicially entrenched96 is the rule that it is the DENR Secretary who has the authority to approve land
classification and release a land of public domain as alienable and disposable. In Republic v. T.A.N. Properties:97

[I]t is not enough for the PENRO or CENRO to certify that a land is alienable and disposable. The applicant for
land registration must prove that the DENR Secretary had approved the land classification and released the land
of the public domain as alienable and disposable, and that the land subject of the application for registration
falls within the approved area per verification through survey by the PENRO or CENRO. In addition, the
applicant for land registration must present a copy of the original classification approved by the DENR
Secretary and certified as a true copy by the legal custodian of the official records. These facts must be
established to prove that the land is alienable and disposable.98

Republic v. Hanover99 ruled that a CENRO certification does not constitute incontrovertible proof that a piece of
land is alienable and disposable. This is because "the CENRO is not the official repository or legal custodian of
the issuances of the DENR Secretary declaring the alienability and disposability of public lands." 100Republic v.
Vda. De Joson explained:101

This doctrine unavoidably means that the mere certification issued by the CENRO or PENRO did not suffice to
support the application for registration, because the applicant must also submit a copy of the original
classification of the land as alienable and disposable as approved by the DENR Secretary and certified as a true
copy by the legal custodian of the official records.102
III

The pieces of evidence the Spouses Go adduced fall short of the requirements of the law.

First, the Spouses Go failed to present a certified true copy of the original classification of the DENR Secretary.
Page 87

This Court has given them enough chances to prove their claim. As a rule, this Court can only consider the
evidence submitted before the trial court. 103 Nevertheless, this Court gave respondents the opportunity to submit
"a certified true copy of the Presidential or Department of Environment and Natural Resources Secretary's
issuance declaring the property alienable and disposable." 104 They failed to comply despite being given a show-
cause order.105

This Court also required them to file their Comment on petitioner's opposition to their original
registration.106 Instead of complying, they asked that their Comment be dispensed with. 107

Second, although the Spouses Go submitted a CENRO certification stating that the land was verified to be
within alienable and disposable zone under Project No. 13, Land Classification Map No. 718, Maglinao, the
person who issued the CENRO Certification, testified otherwise. She admitted in her testimony that, she
certified the lot only to determine "the point or monument of the entire or whole area" and not to identify its
alienable character.108

The Spouses Go have the burden to show that the land for registration is alienable or disposable, 109 which they
miserably failed to do so. Without the original land classification approved by the DENR Secretary, the Spouses
Go's application for registration must be denied.110 The land remains inalienable.

In sum, the Court of Appeals gravely erred in affirming the trial court's Decision that granted the Spouses Go's
application for registration of Lot No. 4699-B. The Spouses Go failed to adequately prove their claim of
possession in the concept of an owner since June 12, 1945. They likewise failed to establish that the land
applied for registration is alienable and disposable. Thus, their occupation of this land, no matter how long,
cannot ripen into ownership and cannot be registered as a title.111

WHEREFORE, the Petition is GRANTED. The Court of Appeals Decision dated January 21, 2011 and
Resolution dated June 6, 2011 in CA-G.R. CV No. 93000, which affirmed the Decision of the Municipal Trial
Court in Cities dated December 12, 2008, are REVERSED and SET ASIDE. The application for registration of
the Spouses Danilo Go and Amorlina Go of Lot No. 4699-B of Subdivision Plan Csd-04-022290-D
is DENIED for lack of merit.

SO ORDERED.

Republic Act No. 10023, March 9, 2010 (An Act Authorizing the Issuance of Free Patents to Residential
Lands)

What is the New Residential Free Patent Act?


Republic Act No. 10023, otherwise known as an Act Authorizing the Issuance of Free Patents to Residential
Lands or the Free Patent Act which was signed into law by former President Gloria Macapagal-Arroyo last
March 9, 2010. The newly enacted law aims to ease the requirements and procedures in the titling of residential
lands. This law will benefit about 40 million Filipinos who are living on unregistered land today. This law
amended RA 9176, otherwise known as the Public Land Act.
What is a free patent?
Under the Public Land Act, free patent is the acquisition of public lands by means of an administrative
confirmation of imperfect title. It is intended to legalize the land rights of Filipinos who are founded to be
occupying and cultivating such lands for a certain period of time.
What are the salient features of R.A. 10023?
1. RA 10023 reduces the period of eligibility for titling from 30 years to 10 years. However, only alienable land
not needed for public service or public use may be given a free patent.
Page 88

2. Any Filipino who has paid all the real estate taxes for 10 years shall be entitled to free patent for such parcel
of land in all municipalities and cities.
3. The Law avoids the lengthy and tedious process of acquiring a title through the courts under the Public Land
Act which usually takes a long period of time.
4. In order to qualify, the land should not exceed 200 square meters if it is in a highly urbanized city, 500 meters
in other cities, 750 meters in first-class and second-class municipalities, and 1,000 meters in third-class
municipalities.
5. The law allows the issuance of free patents without payment of outstanding real estate taxes and removal of
restrictions after issuance of free patents.
6. Local government units may also apply for a free patent for public land being used for public schools,
municipal halls, public plazas or parks, and other government institutions for public use.
7.The Law provides security of the property rights of the owner and will also facilitate the title holders to credit
facilities of banks and other financial institutions using their land titles as collateral for loans.
8. With the new law signed, landowners can now apply with the Department of Environment and Natural
Resources (DENR) to acquire a title through a free patent.
What is a residential free patent?
A residential free patent is issued on all land that are zoned as residential areas, including town sites as defined
under the Public Land Act, provided that none of the provisions of PD 705 shall be violated.
Who are eligible to apply for a residential free patent?
Any Filipino citizen who is in actual occupation of the residential land for at least 10 years may apply for
residential patent. The applicant must be at least 18 years old, if the applicant is a minor he or she must be
represented by their legal guardians. The heirs of a deceased applicant may substitute the applicant provided
that they themselves possess the required application. Only one application shall be allowed per applicant.
Where to file an application for a residential free patent?
Application shall be filed at the Community Environment and Natural Resources Office (DENR – CENRO)
which has jurisdiction over the area being applied for.

A2. Registration Under CA No. 141 (Public Land Act)

Republic v. Doldol, GR No. 132963, Sept. 10, 1998


REPUBLIC OF THE PHILIPPINES, (represented by Opol National Secondary Technical School), Petitioner,
vs. NICANOR DOLDOL, Respondent.

Summary: In 1959, a Nicanor Doldol occupied a lot in Opol, Misamis Oriental. In 1965, a portion of a land,
including the lot occupied by Doldol, was reserved by the city government as a school site. The school, Opol
High School eventually transferred to the said site in 1970. Seventeen years later, or in 1987, Pres. Corazon
Aquino reserved the said land to Opol National School. Demands were made for Doldol to vacate the land.
Since Doldol did not heed the demands, Opol National School filed an action to recover possession of the said
lot. RTC ruled in favor of the school, but the CA reversed, ruling that since Doldol had been in possession of the
lot for 32 years, he had acquired an imperfect title over the said lot. The Court reversed the CA, holding that the
correct legal basis is PD 1073, which states that for there to be an acquisition of a right over a lot, there should
have been an open, continuous, exclusive and notorious possession since June 12, 1945 or earlier. Doldol only
started occupying the said lot on 1959. Since he did not meet the requirements, he did not acquire a better right
over the said lot. Opol National School now has a better right by virtue of it being declared a reserved site by
the President.
Doctrine: Under PD 1073, a person acquires a right to a government rant over a particular land, without the
necessity of a certificate of title being issued, if: (a) The land is alienable public land (b) The person has open,
continuous, exclusive and notorious possession and occupation of the same which must be for the period
prescribed by the law, which is since June 12, 1945, or earlier.
Page 89

Facts:
 1959: Nicanor Doldol occupied a portion of land in Barrio Pontacan, Municipalty of Opol, Misamis
Oriental.
o In 1963, he filed an application for saltwork purposes for the said area but the same was rejected
by the Bureau of Forest Development in 1968.
 While his application was pending, in 1965, the Provincial Board of Misamis Oriental passed a
resolution reserving Lot 4932 as a school site, which was eventually occupied by Opol High School in
1970.
o T he reserved lot included the land area occupied by Doldol.
 Seventeen years later, or on November 2, 1987, Pres. Corazon Aquino issued a Proclamation still
reserving the said area to Opol High School, which was then renamed to Opol National Secondary
Technical School.
o By virtue of said declaration, the school demanded that Doldol vacate the land, but he refused.
1991: Opol National School filed a complaint for accion possesoria with the Cagayan de Oro
RTC.
 The trial court ruled in favor of the school and ordered Doldol to vacate.
 CA reversed and held that since Doldol has possessed the land from 1959 up to 1991 or for 32 years, he
is now entitled to the same by virtue of prescription.
o CA cites as basis for its decision Sec. 48 of CA 141 or the Public Land Act, as amended by RA
1942: Section 48. The following described citizens of the Philippines, occupying lands of the
public domain or claiming interest therein, but whose titles have not been perfected or
completed, may apply to the Court of First Instance (now Regional Trial Court) of the province
where the land is located for confirmation of their claims and the issuance of a certification of
title therefor under the Land Registration Act, to wit:
xxxxxxxxx
(b) Those who by themselves or through their predecessors-in-interest have been in open,
continuous, exclusive and notorious possession and occupation of agricultural lands of the public
domain, under a bona fide claim of acquisition or ownership for at least thirty years immediately
preceding the filing of the application for confirmation of title, except when prevented by wars or
force majeure. Those shall be conclusively presumed to have performed all the conditions
essential to a Government grant and shall be entitled to a certificate of title under the provisions
of this chapter.
 Hence, the instant petition.
Ruling: Petition granted.

Whether Doldol, having occupied the lot for 32 years, acquired a right over the land - NO
 While the laws provide for prescription as a way for acquiring ownership over a particular land, the CA
erred in applying Sec. 48 of the Public Land Act which is the outdated version of the said law.
 Sec. 48 has now already been amended by PD 1073, which now states that: (b) Those who by themselves or
through their predecessors-in-interest have been in open, continuous, exclusive and notorious possession
and occupation of agricultural lands of the public domain, under a bona fide claim of acquisition or
ownership, since June 12, 1945, or earlier, immediately preceding the filing of the application for
confirmation of title, except when prevented by wars or force majeure. Those shall be conclusively
presumed to have performed all the conditions essential to a Government grant and shall be entitled to a
certificate of title under the provisions of this chapter.
 Under the said act, a person acquires a right to a government rant over a particular land, without the
necessity of a certificate of title being issued, if:
a) The land is alienable public land
Page 90

b) The person has open, continuous, exclusive and notorious possession and occupation of the same
which must be for the period prescribed by the law, which is since June 12, 1945, or earlier IN
 THIS CASE:
o The land is alienable and disposable, in accordance with the District Foresters Certification
o However, Doldol had been occupying the land reserved for the school site only since 1959.
 The law requires that the possession of lands of public domain must be from June 12, 1945 or earlier, for the
same to be acquired through judicial confirmation of imperfect title. Thus, Doldol could not have acquired
an imperfect title or a right to the disputed lot. He cannot, therefore, assert a right superior to the school,
given that the President have reserved the said lot as a school site. Having been reserved in its favor, Opol
National School has a better right of possession over the land in dispute.
o The privilege of occupying public lands with a view of preemption confers no contractual or vested
right in the lands occupied and the authority of the President to withdraw such lands for sale or
acquisition by the public, or to reserve them for public use, prior to the divesting by the government
of title thereof stands, even though this may defeat the imperfect right of a settler. Lands covered by
reservation are not subject to entry, and no lawful settlement on them can be acquired.
Notes:
 For those details which are not important but seems important.

Republic v. Espinosa, GR No. l7l5l4, July 18, 2012


REPUBLIC OF THE PHILIPPINES, Petitioner, vs. DOMINGO ESPINOSA, Respondent.

FACTS:
1. Espinosa filed with the MTC an application for land registration of a parcel of land located in Cebu.
o The land was alienable and disposable
o He purchased the property from his mother Isabel on July 4, 1970
o He and his predecessors in interest had been in possession of the property in the concept
of an owner for more than 30 years.
o Submitted the blue print of Advance Survey Land to prove the identity of the land as Alienable
and Disposable Area.
2. Tax Declarations to prove that she had been in possession of the property since 1965. 2. The
Republic opposed Espinosa’s registration claiming that:
o Section 48(b) of the PLA had not been complied with as Espinosa’s predecessors in
interest had possessed the property only after June 12, 1945.
o The Tax Declaration does not prove that his possession and that of his predecessors in interest
are in the character and for the length of time required by law.
3. MTC’s decision:
o Granted Espinosa’s registration.
o Espinosa complied with the requirements of Section 14(1) of PRD and that the land was
alienable and disposable.
4. Upon appeal to the CA, Republic pointed out:
o As shown by the TD, Isabel’s possession commenced only on 1965 and not on June 12, 1945 or
earlier.
o Espinosa’s contention that he and his predecessors in interest were in possession for more
than 30 years is inconsequential absent proof that such possession began on June 12, 1945 or
earlier.
o Republic contend that the failure of Espinosa to present the original tracing cloth of the survey
plan is fatal to his application. It is mandatory in establishing the identity of the land
subject of the application.
Page 91

5. CA affirmed RTC’s decision


o Established jurisprudence has consistently pronounced that, open, continuous and exclusive
possession for at least 30 years of alienable public land ipso jure converts the same into
private property. This means that occupation and cultivation for more than 30 years by
applicant and his predecessor-in-interest vests title on such applicant so as to segregate the land
from the mass of public land
o It is true that the requirement of possession since June 12, 1945 is the latest amendment of
Section 48(b) of the Public Land Act (C.A. No. 141), but a strict implementation of the law
would in certain cases result in inequity and unfairness to [Espinosa]. The fact that the
applicant or his predecessor in interest had been in OPENCO of the land for 30 years or more
is enough.
o Registration can be based on other documentary evidence not necessarily the original tracing
cloth plan. While in some cases the SC ruled that the submission of the original tracing
cloth plan is mandatory and fatal to the application, such pronouncement need not be taken
as an iron clad rule not to be strictly applied in all cases.
o As long as the identity of and location of the lot can be established by other competent evidence
like a duly approved blueprint copy of the advance survey plan of Lot 8499 and technical
description of Lot 8499, containing and identifying the boundaries, actual area and
location of the lot, the presentation of the original tracing cloth plan may be excused.
o Thus, Espinosa had duly proven that the property is alienable and disposable.
6. Hence, this petition. Republic’s contention:
o Espinosa failed to prove by competent evidence that the subject property is alienable and
disposable
o Jurisprudence dictates that a survey plan identifies the property in preparation for a
judicial proceeding but does not convert the property into alienable
o Submission of original tracing cloth plan is mandatory
o A blue print copy of the survey plan may be admitted as evidence only if it bears the approval of
the Director of Lands.
Issues and Ruling:
1) W/N Espinosa had acquired an imperfect title over the subject property that is worthy of
confirmation and registration (In summary reasons): Espinosa failed to prove: (1) Isabel’s
possession of the property dated back to June 12, 1945 or earlier; (2) The property is alienable
and disposable; (3) Applying Section 14(2) failed to prove that the property is patrimonial.
Held: NO
Ratio:
o Espinosa’s claim is hinged on Section 14(2) and not based on Section 14(1) in relation to Section 48
(b)- The lower courts failed to apprise themselves of the changes that Section 48(b) of the PLA
underwent
 Claim under Section 14(2) is to those who acquired ownership of private lands by
prescription under the provisions of existing laws.
 In the case at bar, Espinosa repeatedly alleged that he acquired title thru his possession and
that of his predecessor in interest, Isabel of the subject property for 30 years or through
prescription.
 The lower courts failed to apprise themselves of the changes that Section 48(b) of the
PLA underwent
 From the requirement that possession must be from July 26, 1948 or earlier →
amendment (RA 1942) →30 year prescriptive period for judicial confirmation of
imperfect title →amendment (PD No. 1073) → Occupation since June 12, 1945.
Page 92


PD No. 1529 was then enacted and adopted the requirement of possession and
occupation since June 12 1945.
 In effect, PD 1073 repealed RA 1942. Vested rights may have been impaired for those who
registered their land under the requirement under RA 1942( possession of 30 years) and
the promulgation of PD 1073 denied this right due to the new requirement.
 The Court resolved this dilemma and held:Since the law cannot impair vested rights such as
a land grant. More clearly stated, Filipino citizens who by themselves or their
predecessors-in-interest have been, prior to the effectivity of P.D. 1073 on January 25,
1977, in open, continuous, exclusive and notorious possession and occupation of
agricultural lands of the public domain, under a bona fide claim of acquisition of
ownership, for at least 30 years, or at least since January 24, 1947 may apply for judicial
confirmation of their imperfect or incomplete title under Section 48(b) of the PLA
 Thus, for one to invoke Section 48(b) on the basis of 30 year possession and occupation,
it must be demonstrated that possession and occupation commenced on January 24,
1947 and the 30 year period was completed prior to the effectivity of PD 1073.
 There is nothing in Section 48(b) that would suggest that it provides for two (2)
modes of acquisition. It is not the case that there is an option between possession and
occupation for thirty (30) years and possession and occupation since June 12, 1945 or
earlier. It is neither contemplated under Section 48(b) that if possession and occupation of
an alienable and disposable public land started after June 12, 1945, it is still possible
to acquire an imperfect title if such possession and occupation spanned for thirty (30)
years at the time of the filing of the application.
o APPLICATION IN THE CASE AT BAR:
 In the case at bar, the lower courts concluded that Espinosa complied with the
requirements of Section 48(b) in relation to Section 14(1) based on the supposed
evidence that he and his predecessor in interest had been in possession of the property of
at least 30 years prior to the time he filed for his application.
 However, nothing on record showed that as of January 25, 1977 or prior to the effectivity of
PD No. 1073, he or Isabel had already acquired title by means of possession and
occupation of the property for 30 years.
 The TD presented showed that only 12 years had lapsed from the time Isabel
first came supposedly into possession.
 Thus assuming that it is Section 48(b) in relation to Section 14(1) that should
apply in the case, it was incumbent upon Espinosa to prove that Isabel’s
possession dated back at least to June 12, 1945.
 Since evidence showed that Isablel’s possession was much later, the application should
have been dismissed. o Since it is clear, that Espinosa’s claim is based through
prescription, he must show that the properties were not for public use, public service
or intended for the development of national wealth. This must be coupled with an official
declaration to the effect before the property may be rendered susceptible for prescription.
 Thus, granting that Isabel and, later, Espinosa possessed and occupied the property for an
aggregate period of thirty (30) years, this does not operate to divest the State of its
ownership. The property, albeit allegedly alienable and disposable, is not patrimonial. As
the property is not held by the State in its private capacity, acquisition of title thereto
necessitates observance of the provisions of Section 48(b) of the PLA in relation to Section
14(1) of P.D. No. 1529 or possession and occupation since June 12, 1945. For
prescription to run against the State, there must be proof that there was an official
declaration that the subject property is no longer earmarked for public service or the
development of national wealth. Moreover, such official declaration should have been
issued at least ten (10) or thirty (30) years, as the case may be, prior to the filing of the
Page 93

application for registration. The period of possession and occupation prior to the
conversion of the property to private or patrimonial shall not be considered in
determining completion of the prescriptive period. Indeed, while a piece of land is still
reserved for public service or the development of national wealth, even if the same is
alienable and disposable, possession and occupation no matter how lengthy will not
ripen to ownership or give rise to any title that would defeat that of the State’s if such did
not commence on June 12, 1945 or earlier.
2) W/N the blueprint of the advance survey plan substantially complies with Section 17 of PD 1529
Held: NO.
o The blueprint copy of the advanced survey plan may be admitted as evidence of the identity and
location of the subject property if: (a) it was duly executed by a licensed geodetic engineer; (b)
it proceeded officially from the Land Management Services (LMS) of the DENR; and (c) it is
accompanied by a technical description of the property which is certified as correct by the
geodetic surveyor who conducted the survey and the LMS of the DENR.
o However, while such blueprint copy of the survey plan may be offered as evidence of the identity,
location and the boundaries of the property applied for, the notation therein may not be
admitted as evidence of alienability and disposability.
3) W/N the notation on the blueprint copy of the plan made by the geodetic engineer who conducted
the survey sufficed to prove that the land applied for is alienable and disposable.
Held: NO
o The burden of proof in overcoming the presumption of State ownership of the lands of the
public domain is on the person applying for registration (or claiming ownership), who must prove
that the land subject of the application is alienable or disposable. To overcome this
presumption, incontrovertible evidence must be established that the land subject of the
application (or claim) is alienable or disposable
o The notation made by a surveyor-geodetic engineer that the property surveyed is alienable and
disposable is not the positive government act that would remove the property from the
inalienable domain. Neither it is the evidence accepted as sufficient to controvert the
presumption that the property is inalienable

Republic v. Regulto, GR No. 202051, April 18, 2016


REPUBLIC OF THE PHILIPPINES, represented by the DEPARTMENT OF PUBLIC WORKS &
HIGHWAYS; ENGINEER SIMPLICIO D. GONZALES, District Engineer, Second Engineering District of
Camarines Sur; and ENGINEER VICTORINO M. DEL SOCORRO, JR., Project Engineer, DPWH, Baras,
Canaman, Camarines Sur, Petitioners, vs. SPOUSES ILDEFONSO B. REGULTO and FRANCIA R.
REGULTO, Respondents.

Facts: Respondents spouses Ildefonso B. Regulto and Francia R. Regulto are the registered owners of the
property in controversy located at Mabel, Naga City, Camarines Sur consisting of 300 square meters covered by
Transfer Certificate of Title (TCT) No. 086-2010000231.

The subject property originated from a Free Patent property consisting of 7,759 square meters registered and
covered by Original Certificate of Title (OCT) No. 235 dated April 14, 1956.

Sometime in April 2011, the DPWH Second Engineering District of Camarines Sur apprised the Spouses
Regulto of the construction of its road project, the Naga City-Milaor Bypass Road, which will traverse their
property and other adjoining properties.[5] The DPWH initially offered the spouses the sum of P243,000.00 or
P1,500.00 per square meter for the 162 square-meter affected area as just compensation.
Page 94

However, in a letter dated May 11, 2006, the DPWH, through District Engr. Rolando P. Valdez, withdrew the
offer, and informed the Spouses Regulto that they were not entitled to just compensation since the title of their
land originated from a Free Patent title acquired under Commonwealth Act (C.A.) No. 141,... which contained a
reservation in favor of the government of an easement of right-of-way of twenty (20) meters, which was
subsequently increased to sixty (60) meters by Presidential Decree (P.D.) No. 635.

The Spouses Regulto... protested the findings of the DPWH and ordered them to cease from proceeding with the
construction.

They alleged that since their property is already covered by TCT No. 086-2010000231, it ceased to be a public
land.

Furthermore, they requested that they be furnished, within five (5) days from the receipt of their letter, with a
Program of Works and Sketch Plan showing the cost of the project and the extent or area covered by the road
that will traverse their property.

The DPWH furnished the Spouses Regulto with the sketch plan showing the extent of the road right-of-way that
will cut across their property.[12] It also reiterated its earlier position that the title to the land was acquired
under C.A. No. 141.

Souses Regulto with the sketch plan showing the extent of the road right-of-way that will cut across their
property.[12] It also reiterated its earlier position that the title to the land was acquired under C.A. No. 141.[
On October 8, 2011, the Spouses Regulto filed a complaint for payment of just compensation, damages with
prayer for issuance of temporary restraining order and/or writ of preliminary injunction.

Consequently, the RTC, in its Order dated May 24, 2012, ordered the petitioners to pay the Spouses Regulto the
amount of P243,000.00 as just compensation for the affected portion of their property.[

The RTC concluded that the government waived the encumbrance provided for in C.A. No. 141 when it did not
oppose the further subdivision of the original property covered by the free patent or made an express intent on
making its encumbrance before the residential lots, which are part of the said subdivision, were sold to other
innocent purchasers for value, especially after the 25-year period has lapsed since the free paten

Issues:
 The RTC erred in holding that respondents are entitled to and in ordering petitioners to pay just
compensation despite the undisputed fact that the land was originally public land awarded to
respondents' predecessors-in-interest by free patent, and thus a legal easement of right-of-way exists in
favor of the government.
 The trial court's ratiocination - that the subject-property has ipso facto ceased to be "public land" and
thus no longer subject to the lien imposed by said provision of c.a. no. 141, by virtue of the subject
property being already covered by a transfer certificate of title in their name - contravenes section 44 OF
P.D. no. 1529 and national irrigation administration vs. Court of appeals.
 The RTC erred in holding that section 8 ("expropriation"), not section 5 ("quit claim"), of the
implementing rules and regulations of R.A. no. 8974 is the applicable provision regarding the mode of
acquisition of respondents' property.

Essentially, the issue for resolution of this Court is whether the petitioners are liable for just compensation in
enforcing the Government's legal easement of right-of-way on the subject property which originated from the
7,759 square-meter of public land awarded by free patent to the predecessor-in-interest of the Spouses Regulto.
Page 95

Petitioners allege that a legal easement of right-of-way exists in favor of the Government since the land in
controversy was originally public land awarded by free patent to the Spouses Regulto's predecessors-in-interest.

Ruling: This Court finds that the RTC erroneously ruled that the provisions of C.A. No. 141 are not applicable
to the case at bar. On the contrary, this Court held that "a legal easement of right-of-way exists in favor of the
Government over land that was originally a public land awarded by free patent even if the land is subsequently
sold to another."[45] This Court has expounded that the "ruling would be otherwise if the land was originally a
private property, to which just compensation must be paid for the taking of a part thereof for public use as an
easement of right-of-way."[46]

It is undisputed that the subject property originated from and was a part of a 7,759-square-meter property
covered by free patent registered under OCT No. 235.[47] furthermore, the Spouses Regulto's transfer
certificate of title, which the RTC relied, contained the reservation: "subject to the provisions of the Property
Registration Decree and the Public Land Act, as well as to those of the Mining Law, if the land is mineral, and
subject, further, to such conditions contained in the original title as may be subsisting."[

Jurisprudence settles that one of the reservations and conditions under the Original Certificate of Title of land
granted by free patent is that the said land is subject "to all conditions and public easements and servitudes
recognized and prescribed by law especially those mentioned in Sections 109, 110, 111, 112, 113 and 114,
Commonwealth Act No. 141, as amended.

Section 112 of C.A. No. 141, as amended, provides that lands granted by patent shall be subjected to a right-of-
way in favor of the Government... with damages for the improvements only.

Lands granted by patent shall be subject to a right-of-way not exceeding 60 meters in width for public
highways, irrigation ditches, aqueducts, and other similar works of the government or any public enterprise, free
of charge, except only for the value of the improvements existing thereon that may be affected
We are not persuaded with the ruling of the RTC that the government waived the encumbrance imposed by C.A.
No. 141 (Public Land Act) when it did not oppose the subdivision of the original property covered by the free
patent.

The reservation and condition contained in the OCT of lands granted by free patent, like the origins of the
subject property, is not limited by any time period, thus, the same is subsisting.
With the existence of the said easement of right-of-way in favor of the Government, the petitioners may
appropriate the portion of the land necessary for the construction of the bypass road without paying for it,
except for damages to the improvements. Consequently, the petitioners are ordered to obtain the necessary
quitclaim deed from the Spouses Regulto for the 162-square-meter strip of land to be utilized in the bypass road
project.

While this Court concurs that the petitioners are not obliged to pay just compensation in the enforcement of its
easement of right-of-way to lands which originated from public lands granted by free patent, we, however, rule
that petitioners are not free from any liability as to the consequence of enforcing the said right-of-way granted
over the original 7,759-square-meter property to the 300-square-meter property belonging to the Spouses
Regulto.

There is "taking," in the context of the State's inherent power of eminent domain, when the owner is actually
deprived or dispossessed of his property; when there is a practical destruction or material impairment of the
value of his property or when he is deprived of the ordinary use thereof... it is apparent that there is taking of the
remaining area of the property of the Spouses Regulto
Page 96

The fact that more than half of the property shall be devoted to the bypass road will undoubtedly result in
material impairment of the value of the property

Thus, the petitioners are liable to pay just compensation over the remaining area of the subject property, with
interest thereon at the rate of six percent (6%) per annum from the date of writ of possession or the actual taking
until full payment is made.
Consequently, the case is remanded to the court of origin for the purpose of determining the final just
compensation for the remaining area of the subject property. The RTC is thereby ordered to make the
determination of just compensation payable to the respondents Spouses Regulto with deliberate dispatch

WHEREFORE, the petition for review on certiorari dated July 10, 2012 filed by the Republic of the Philippines
as represented by the Department of Public Works and Highways; Engineer Simplicio D. Gonzales, District
Engineer, Second Engineering District of Camarines Sur; and Engineer Victorino M. Del Socorro, Jr., Project
Engineer, DPWH, Baras, Canaman, Camarines Sur, is hereby PARTIALLY GRANTED... case is hereby
REMANDED to the Regional Trial Court of Naga City, Branch 62 for the determination of the final just
compensation

Principles:
It is undisputed that the subject property originated from and was a part of a 7,759-square-meter property
covered by free patent registered under OCT No. 235.[47] furthermore, the Spouses Regulto's transfer
certificate of title, which the RTC relied, contained the reservation: "subject to the provisions of the Property
Registration Decree and the Public Land Act, as well as to those of the Mining Law, if the land is mineral, and
subject, further, to such conditions contained in the original title as may be subsisting."

Jurisprudence settles that one of the reservations and conditions under the Original Certificate of Title of land
granted by free patent is that the said land is subject "to all conditions and public easements and servitudes
recognized and prescribed by law especially those mentioned in Sections 109, 110, 111, 112, 113 and 114,
Commonwealth Act No. 141, as amended."[49]

Section 5 of the IRR of R.A. No. 8974 provides:SECTION 5. Quit Claim - If the private property or land is
acquired under the provisions of Special Laws, particularly Commonwealth Act No. 141, known as the Public
Land Act, which provides a 20-meter strip of land easement by the government for public use with damages to
improvements only, P.D. No. 635 which increased the reserved area to a 60-meter strip, and P.D. No. 1361
which authorizes government officials charged with the prosecution of projects or their representative to take
immediate possession of portion of the property subject to the lien as soon as the need arises and after due
notice to the owners, then a quit claim from the owners concerned shall be obtained by the Implementing
Agency. No payment by the government shall be made for land acquired under the quit claim mode.

With the existence of the said easement of right-of-way in favor of the Government, the petitioners may
appropriate the portion of the land necessary for the construction of the bypass road without paying for it...
except for damages to the improvements

Consequently, the petitioners are ordered to obtain the necessary quitclaim deed from the Spouses Regulto for
the 162-square-meter strip of land to be utilized in the bypass road project.
Gozo v. Philippine Union Mission, GR No. 195990, Aug. 5, 2015
HEIRS OF RAFAEL GOZO REPRESENTED BY CASTILLO GOZO AND RAFAEL GOZO,
JR., Petitioners, v. PHILIPPINE UNION MISSION CORPORATION OF THE SEVENTH DAY ADVENTIST
CHURCH (PUMCO), SOUTH PHILIPPINE UNION MISSION OF SDA (SPUMCO) AND SEVENTH DAY
Page 97

ADVENTIST CHURCH AT SIMPAK, LALA, LANAO DEL NORTE REPRESENTED BY BETTY


PEREZ , Respondents.

The facts: Petitioners claim that they are the heirs of Spouses Gozo. The respondents claim that they own a
5,000 square-meter portion of the property. The assertion is based on the 28 February 1937 Deed of Donation in
favor of respondent Philippine Union Mission Corporation of the Seventh Day Adventist (PUMCO-SDA). On
the date the Deed of Donation is executed in 1937, the Spouses Gozo were not the registered owners of the
property yet although they were the lawful possessors thereof. The land in question was part of the inalienable
public domain.

Ruling: The Deed of Donation was executed by the Spouses Gozo on 28 February 1937, the subject property
was part of the inalienable public domain. It was only almost after two decades later or on 5 October 1953 that
the State ceded its right over the land in favor of the Spouses Gozo by granting their patent application and
issuing an original certificate of title in their favor. Prior to such conferment of title, the Spouses Gozo
possessed no right to dispose of the land which, by all intents and purposes, belongs to the State.

Under the Regalian doctrine, which is embodied in Article XII, Section 2 of our Constitution, all lands of the
public domain belong to the State, which is the source of any asserted right to any ownership of land. All lands
not appearing to be clearly within private ownership are presumed to belong to the State. Accordingly, public
lands not shown to have been reclassified or released as alienable agricultural land or alienated to a private
person by the State remain part of the inalienable public domain.

The classification of public lands is an exclusive prerogative of the executive department of the government and
not the Courts. In the absence of such classification, the land remains as an unclassified land until it is released
therefrom and rendered open to disposition. This is in consonance with the Regalian doctrine that all lands of
the public domain belong to the State and that the State is the source of any asserted right to ownership in land
and charged with the conservation of such patrimony.

All lands not appearing to be clearly within private ownership are presumed to belong to the State. Accordingly,
all public lands not shown to have been reclassified or released as alienable agricultural land or alienated to a
private person by the State remain part of the alienable public domain. As already well-settled in jurisprudence,
no public land can be acquired by private persons without any grant, express or implied, from the government;
and it is indispensable that the person claiming title to public land should show that his title was acquired from
the State or any other mode of acquisition recognized by law. To prove that the land subject of an application for
registration is alienable, the applicant must establish the existence of a positive act of the government such as a
presidential proclamation or an executive order, an administrative action, investigation reports of Bureau of
Lands investigators, and a legislative act or a statute. The applicant may also secure a certification from the
Government that the land applied for is alienable and disposable.

It is beyond question that at the time the gratuitous transfer was effected by the Spouses Gozo on 28 February
1937, the subject property was part of the public domain and is outside the commerce of man. It was only on 5
October 1953 that the ownership of the property was vested by the State to the Spouses Gozo by virtue of its
issuance of the OCT pursuant to the Homestead Patent granted by the President of the Philippines on 22 August
1953. Hence, the donation of the subject property which took place before 5 October 1953 is null and void from
the very start.

As a void contract, the Deed of Donation produces no legal effect whatsoever. Quod nullum est, nullum
producit effectum. That which is a nullity produces no effect Logically, it could not have transferred title to the
subject property from the Spouses Gozo to PUMCO-SDA and there can be no basis for the church's demand for
the issuance of title under its name. Neither does the church have the right to subsequently dispose the property
Page 98

nor invoke acquisitive prescription to justify its occupation. A void contract is not susceptible to ratification, and
the action for the declaration of absolute nullity of such contract is imprescriptible.

A3. Applications: Registration under RA No. 8371 (Indigenous Peoples Rights Act- IPRA)

Cruz v. Secretary of Environment and Natural Resources, GR No. 135385, Dec. 6, 2000

See page 16
Central Mindanao University v. Executive Secretary, GR No. 184869, Sept. 21, 2010
CENTRAL MINDANAO UNIVERSITY, Represented by Officer-In-Charge Dr. Rodrigo L.
Malunhao, Petitioner, v. THE HONORABLE EXECUTIVE SECRETARY, THE HONORABLE SECRETARY
OF THE DEPARTMENT OF ENVIRONMENT AND NATURAL RESOURCES, THE CHAIRPERSON AND
COMMISSIONERS OF THE NATIONAL COMMISSION ON INDIGENOUS PEOPLES, and THE LEAD
CONVENOR OF THE NATIONAL ANTI-POVERTY COMMISSION, Respondents.

Summary: (PROC 310 UNCONSTITUTIONAL) The education of the youth and agrarian reform are
admittedly among the highest priorities in the government socio-economic programs. In this case, neither need
give way to the other. THUS, THE LANDS BY THEIR CHARACTER HAVE BECOME INALIENABLE
FROM THE MOMENT PRESIDENT GARCIA DEDICATED THEM FOR CMU’S USE IN SCIENTIFIC
AND TECHNOLOGICAL RESEARCH IN THE FIELD OF AGRICULTURE. THEY HAVE CEASED TO BE
ALIENABLE PUBLIC LANDS. Besides, when Congress enacted the Indigenous Peoples’ Rights Act (IPRA)
or Republic Act 8371 in 1997, it provided in Section 56 that "property rights within the ancestral domains
already existing and/or vested" upon its effectivity "shall be recognized and respected." In this case, ownership
over the subject lands had been vested in CMU as early as 1958. Consequently, transferring the lands in 2003 to
the indigenous peoples around the area is not in accord with the IPRA.

Nature: Concerns the constitutionality of a presidential proclamation that takes property from a state university,
over its objections, for distribution to indigenous peoples and cultural communities.

Facts:
 Petitioner Central Mindanao University (CMU) is a chartered educational institution owned and run by
the State. –
 In 1958, the President issued Presidential Proclamation 476, reserving 3,401 hectares of lands of the
public domain in Musuan, Bukidnon, as school site for CMU. –
 Forty-five years later or on January 7, 2003 President Gloria Macapagal-Arroyo issued Presidential
Proclamation 310 that takes 670 hectares from CMU’s registered lands for distribution to indigenous
peoples and cultural communities in Barangay Musuan, Maramag, Bukidnon. –
 CMU filed a petition for prohibition against respondents Executive Secretary, et al seeking to stop the
implementation of Presidential Proclamation 310 and have it declared unconstitutional. –
 The NCIP, et al moved to dismiss the case on the ground of lack of jurisdiction of the Malaybalay RTC
over the action SINCE jurisdiction lies with the Manila RTC. - DENIED –
 NCIP – filed a MR – GRANTED motion partial reconsideration and dismissed CMU’s action for lack of
jurisdiction. –
 THE RTC RULED THAT PRESIDENTIAL PROCLAMATION 310 WAS CONSTITUTIONAL,
BEING A VALID STATE ACT. THE RTC SAID THAT THE ULTIMATE OWNER OF THE
LANDS IS THE STATE AND THAT CMU MERELY HELD THE SAME IN ITS BEHALF. –
 CMU filed for MR – DENIED –
Page 99

 Appeal to the CA, the CA dismissed CMU’s appeal for lack of jurisdiction, -
 CMU filed for MR denied –
 THUS - CMU to file the present petition for review.

Issues:
1. Whether or not the CA erred in not finding that the RTC erred in dismissing its action for prohibition against
NCIP, et al for lack of jurisdiction and at the same time ruling that Presidential Proclamation 310 is valid and
constitutional;
2. Whether or not the CA correctly dismissed CMU’s appeal on the ground that it raised purely questions of law
that are proper for a petition for review filed directly with this Court; and
3. Whether or not Presidential Proclamation 310 is valid and constitutional.

Held: PETITION GRANTED – Proclamation 310 is UNCONSTITUTIONAL - null and void for being
contrary to law and public policy.

Ratio:
The Court’s Rulings –
 The key question lies in the character of the lands taken from CMU. In CMU v. Department of Agrarian
Reform Adjudication Board (DARAB), the DARAB, a national government agency charged with taking
both privately-owned and governmentowned agricultural lands for distribution to farmers-beneficiaries,
ordered the segregation for this purpose of 400 hectares of CMU lands. The Court nullified the
DARAB action considering the inalienable character of such lands, being part of the long term
functions of an autonomous agricultural educational institution. Central Mindanao University v.
Exec Secretary Miscellaneous Sean –
 The education of the youth and agrarian reform are admittedly among the highest priorities in the
government socio-economic programs. In this case, neither need give way to the other. –
 Certainly, there must still be vast tracts of agricultural land in Mindanao outside the CMU land
reservation which can be made available to landless peasants, assuming the claimants here, or some of
them, can qualify as CARP beneficiaries. -
 TO OUR MIND, THE TAKING OF THE CMU LAND WHICH HAD BEEN SEGREGATED
FOR EDUCATIONAL PURPOSES FOR DISTRIBUTION TO YET UNCERTAIN
BENEFICIARIES IS A GROSS MISINTERPRETATION OF THE AUTHORITY AND
JURISDICTION GRANTED BY LAW TO THE DARAB. –
 The decision in this case is of far-reaching significance as far as it concerns state colleges and
universities whose resources and research facilities may be gradually eroded by misconstruing the
exemptions from the CARP. –
 These state colleges and universities are the main vehicles for our scientific and technological
advancement in the field of agriculture, so vital to the existence, growth and development of this
country. -
 AS ALREADY STATED, THE LANDS BY THEIR CHARACTER HAVE BECOME
INALIENABLE FROM THE MOMENT PRESIDENT GARCIA DEDICATED THEM FOR
CMU’S USE IN SCIENTIFIC AND TECHNOLOGICAL RESEARCH IN THE FIELD OF
AGRICULTURE. THEY HAVE CEASED TO BE ALIENABLE PUBLIC LANDS. –
 Besides, when Congress enacted the Indigenous Peoples’ Rights Act (IPRA) or Republic Act 8371 in
1997, it provided in Section 56 that "property rights within the ancestral domains already existing and/or
vested" upon its effectivity "shall be recognized and respected." –
 In this case, ownership over the subject lands had been vested in CMU as early as 1958. Consequently,
transferring the lands in 2003 to the indigenous peoples around the area is not in accord with the IPRA.
Page 100

 Furthermore, the land registration court considered the claims of several tribes belonging to the area’s
cultural communities in the course of the proceedings for the titling of the lands in CMU’s name.
Indeed, eventually, only 3,080 hectares were titled in CMU’s name under OCTs 0-160, 0-161 and 0-162.
More than 300 hectares were acknowledged to be in the possession of and subject to the claims of those
tribes.
Sec. 12, RA No. 8371 (Indigenous peoples Rights Act)

SECTION 12. Option to Secure Certificate of Title Under Commonwealth Act 141, as amended, or the Land
Registration Act 496. — Individual members of cultural communities, with respect to their individually-owned
ancestral lands who, by themselves or through their predecessors-in-interest, have been in continuous
possession and occupation of the same in the concept of owner since time immemorial or for a period of not less
than thirty (30) years immediately preceding the approval of this Act and uncontested by the members of the
same ICCs/IPs shall have the option to secure title to their ancestral lands under the provisions of
Commonwealth Act 141, as amended, or the Land Registration Act 496.

For this purpose, said individually-owned ancestral lands, which are agricultural in character and actually used
for agricultural, residential, pasture, and tree farming purposes, including those with a slope of eighteen percent
(18%) or more, are hereby classified as alienable and disposable agricultural lands.
The option granted under this section shall be exercised within twenty (20) years from the approval of this Act.
Begnaen v. Caligtan, GR No. 189852, Aug. 17, 2016
THOMAS BEGNAEN, Petitioner, v. SPOUSES LEO CALIGTAN AND ELMACALIGTAN, Respondents.
Jurisdiction of National Commission on Indigenous People. The NCIP is vested with jurisdiction over (1) the
parties, who are all members of the same Indigenous Cultural Communities, and (2) the subject property, which
is ancestral land.

Summary: On 3 August 2006, petitioner Thomas Begnaen (Begnaen) filed a Complaint with Prayer for
Preliminary Injunction against respondents Spouses Leo and Elma Caligtan (Sps. Caligtan) for "Land Dispute
and Enforcement of Rights" before the Regional Hearing Office (RHO) of the NCIP at La Trinidad, Benguet.7
The RHO thereafter issued an Order complaint based on respondents' argument that the case should have gone
to the council of elders and not through the Barangay Lupon, as mandated by the Indigenous Peoples' Rights
Act (IPRA). 9chanrobleslaw

However, instead of abiding by the Order of the RHO, Begnaen filed against the Sps. Caligtan a Complaint for
Forcible Entry with a Prayer for a Writ of Preliminary Mandatory Injunction10 before the Municipal Circuit
Trial Court (MCTC) of Bauko-Sabangan, Mt. Province.

Begnaen alleged that he was the owner of a 125 square meter parcel of land situated in Supang, Sabangan, Mt.
Province. He claimed that on two occasions,11 respondents - by using force, intimidation, stealth, and threat
-entered a portion of the subject property, hurriedly put up a chicken-wire fence, and started building a shack
thereon without Begnaen's knowledge and consent.12chanrobleslaw

Meanwhile, respondents averred that they owned the area in question as part of the land they had purchased
from a certain Leona Vicente in 1959 pursuant to age-old customs and traditions. They introduced
improvements evidencing their prior physical possession.13 Respondents further contended that when
petitioner's father Alfonso Begnaen (Alfonso) was still alive, he had always respected their boundary wherein a
"GIKAD" or old pine tree lumber was buried and recovered. The "GIKAD" established their boundary pursuant
to age-old Igorot customs and traditions. To further mark their boundary, respondents also p
Page 101

Facts:
The petitioner in this case is the owner of a land situated in Supang, Sabangan, Mt. Province. He alleged that the
respondents – by using force, intimidation, stealth and threat – entered his property and put up a shack bordered
with chicken fence without his consent. On the other hand, the respondents averred that the area in question is a
land that they had purchased pursuant to age-old customs and traditions which the father of the petitioner had
affirmed when he was still alive.

Consequently, the petitioner filed a complaint before the Regional Hearing Office (RHO) of NCIP at La
Trinidad, Benguet. The RHO thereafter dismissed the complaint because the case should have gone to the
council of elders and not through the Barangay Lupon, as mandated by the Indigenous Peoples’ Right Act
(IPRA). However, instead of obeying the orders of NCIP, the petitioner filed another complaint before the
Municipal Circuit Trial Court (MCTC) of Bauko-Sabangan Mt. Province. MCTC recognized the original and
exclusive jurisdiction of NCIP-RHO thereby dismissing the ejectment complaint. As the case was elevated,
RTC reversed and set aside the decision of MCTC saying that the latter court had jurisdiction over the case.
Undaunted, the respondents appealed to the CA. CA reversed and set aside the RTC rulings, and reinstated the
Resolution of the MCTC saying that the provisions of IPRA has divested regular courts of their jurisdiction
when the parties involved are members of ICCs/IPs and the disputed property forms part of their ancestral
domain. Hence, this Petition.

Issue:
1. Whether or not NCIP-RHO has original and exclusive jurisdiction over all claims and disputes
involving rights of ICCs/IPs that have not been settled under customary laws.
2. Whether or not original courts have concurrent jurisdiction with NCIP-RHO on matters mentioned
above.

Held:
1. NEGATIVE. As elucidated by the High Court, NCIP cannot be said to have even primary jurisdiction over
all ICC/IP cases. On a careful perusal of Section 66 of the IPRA, there is no specificity in the grant of
jurisdiction to the NCIP. Neither does the IPRA confer original and exclusive jurisdiction to the NCIP over all
claims and disputes involving rights of the ICCs/IPs. As such, the provisions of NCIP’s Administrative
Circulars’ classification of its RHO’s jurisdiction cannot be warranted by the Court for it broadens the scope of
the jurisdiction conferred by the IPRA on NCIP. Such implementing rules and regulations of a law cannot
extend the law or expand its coverage rather it must remain consistent with the law they seek to apply and
implement.

2. AFFIRMATIVE. The regular courts may exercise jurisdiction over cases which involve rights of ICCs/IPs,
the governing law for these kinds of disputes necessarily include the IPRA and the rights the law bestows on
ICCs/IPs. The Court intimates that while the doctrine of concurrent jurisdiction means equal jurisdiction to deal
with the same subject matter, the body or agency that first takes cognizance of the complaint shall exercise
jurisdiction to the exclusion of the others.

Alternative digest: https://www.scribd.com/document/424675476/Begnaen-Case-Digest

THE CASE: a Petition for Review on Certiorari under Rule 45 of the Rules of Court, assailing the Decision2
and Resolution3 of the Court of Appeals (CA) in CA-G.R. SP No. 104150. The CA reversed and set aside the
Decision4 and Order5 rendered by the Regional Trial Court (RTC) of Bontoc, Mountain (Mt.) Province, and
reinstated the Resolution6 of the Municipal Circuit Trial Court (MCTC) of Bauko, Mt. Province. The case
concerns an ancestral land dispute between members of an Indigenous Cultural Community (ICC), particularly
the Kankanaey Tribe of Mt. Province.
FACTS:
Page 102

On 3 August 2006, petitioner Thomas Begnaen (Begnaen) filed a Complaint with Prayer for Preliminary
Injunction against respondents Spouses Leo and Elma Caligtan (Sps. Caligtan) for "Land Dispute and
Enforcement of Rights" before the Regional Hearing Office (RHO) of the NCIP at La Trinidad, Benguet.7 The
RHO thereafter issued an Order8 dismissing the complaint based on respondents' argument that the case should
have gone to the council of elders and not through the Barangay Lupon, as mandated by the Indigenous Peoples'
Rights Act, instead of abiding by the Order of the RHO, Begnaen filed against the Sps. Caligtan a Complaint for
Forcible Entry with a Prayer for a Writ of Preliminary Mandatory Injunction10 before the Municipal Circuit
Trial Court (MCTC) of Bauko-Sabangan, Mt. Province. AllegING that he was the owner of a 125 square meter
parcel of land situated in Supang, Sabangan, Mt. Province. He claimed that on two occasions,11 respondents -
by using force, intimidation, stealth, and threat - entered a portion of the subject property, hurriedly put up a
chicken-wire fence, and started building a shack thereon without Begnaen's knowledge and consent.
Meanwhile, respondents averred that they owned the area in question as part of the land they had purchased
from a certain Leona Vicente in 1959 pursuant to age-old customs and traditions. They introduced
improvements evidencing their prior physical possession.13 Respondents further contended that when
petitioner's father Alfonso Begnaen (Alfonso) was still alive, he had always respected their boundary wherein a
"GIKAD" or old pine tree lumber was buried and recovered. The "GIKAD" established their boundary pursuant
to age-old Igorot customs and traditions. To further mark their boundary, respondents also planted bushes and a
mango tree, all of which Alfonso had likewise respected.
MCTC RULING
Dismissed the ejectment complaint in favor of respondents. However, this was without prejudice to the filing of
a case before the RHO of the NCIP. Aggrieved, petitioner-
appellant filed an appeal before Regional Trial Court Branch 35 of Bontoc, Mt. Province (RTC).
RTC RULING
RTC reversed and set aside the Resolution and Order of the MCTC, saying that it was the latter court that had
jurisdiction over the case for forcible entry. The RTC reasoned that the provisions of the IPRA pertaining to
jurisdiction do not espouse exclusivity and thus cannot divest the MCTC of its jurisdiction over forcible entry
and unlawful detainer cases as provided by B.P. Big. 129. respondents appealed to the CA.
CA RULING
CA reversed and set aside the RTC rulings and reinstated the Resolution of the MCTC. In upholding the
jurisdiction of the NCIP over the present case, the CA ruled that the passage of the IPRA has divested regular
courts of their jurisdiction when the parties involved are members of ICCs/IPs and the disputed property forms
part of their ancestral land/domain.21 Petitioner filed a Motion for Reconsideration, but it was denied by the CA
. Hence, this Petition.
ISSUE:
RULING OF THE COURT:
In its assailed Decision, the CA reversed the RTC and held that jurisdiction properly lies with the NCIP, to the
exclusion of the regular courts.
R.A. 8371 or the Indigenous Peoples' Rights Act of 1997, particularly Sections 65 and 66 thereof, provide:
SECTION 65. Primacy of Customary Laws and Practices, When disputes involve ICCs/IPs, customary laws and
practices shall be used to resolve the dispute.
SECTION 66. Jurisdiction of the NCIP. The NCIP, through its regional offices, shall have jurisdiction over all
claims and disputes involving rights of ICCs/IPs: Provided, however, That no such dispute shall be brought to
the NCIP unless the parties have exhausted all remedies provided under their customary laws. AND A
certification shall be issued by the Council of Elders/Leaders who participated in the attempt to settle the
dispute that the same has not been resolved.
The IPRA confers jurisdiction on the NCIP over "all claims and disputes involving rights of ICCs/IPs," without
qualification as to whether such jurisdiction is original and/or exclusive. However, Section 5, Rule III of NCIP
Administrative Circular No. 1-03 dated 9 April 2003, known as "The Rules on Pleadings, Practice, and
Procedure Before the NCIP" (NCIP Rules), went beyond the provisions of the IPRA to provide:24
Sec. 5. Jurisdiction of the NCIP. The NCIP through its Regional Hearing Offices shall
Page 103

exercise original and exclusive jurisdiction over all claims and disputes involving rights of
whether or not the CA, in upholding the jurisdiction of the National Commission
on Indigenous Peoples (NCIP) over the aforementioned dispute, to the exclusion of
regular courts, committed reversible error.
The NCIP Rule purporting to establish the jurisdiction of the NCIP-Regional
Hearing Officer as original and exclusive has been declared VOID for expanding
the law.

ICCs/IPs and all cases pertaining to the implementation, enforcement, and interpretation of R.A. 8371(IPRA). ,
including but not limited to the following:
a. Cases involving disputes and controversies over ancestral lands/domains of ICCs/IPs.
b. Cases affecting property rights, claims of ownership, hereditary succession, and settlement of land disputes,
between and among ICCs/IPs that have not been settled under customary laws.
c. Cases involving disputes and controversies over ancestral lands/domains of ICCs/IPs; In several cases the
court ruled that NCIP’s jurisdiction is not exclusive in cases
involving claims and disputes regarding the rights of IP’s/ICC’s.;
in the case of Lim v. gamosa, the issue involved is "whether the NCIP's jurisdiction is limited to cases where
both parties are ICCs/IPs or primary and concurrent with regular courts, and/or original and exclusive, to the
exclusion of the regular courts, on all matters involving rights of ICCs/IPs." At the outset,the court ruled In
Unduran, et at. v. Aberasturi, et al, that Section 66 of the IPRA does not endow the NCIP with primary and/or
exclusive and original jurisdiction over all claims and disputes involving rights of ICCs/IPs. Based on the
qualifying proviso, the court held that the NCIP's jurisdiction over such claims and disputes occur only when
they arise between or among parties belonging to the same ICC/IP.
The City Government of Baguio City, et al. v. Atty. Masweng, et al. of wether the NCIP's jurisdiction over cases
where one of the parties are not ICCs/IPs.
The court ruled that they cannot declare a primary and/or exclusive and original grant of jurisdiction to the
NCIP over all claims and disputes involving rights of ICCs/IPs where there is no clear intendment by the
legislature nor the NCIP cannot be said to have even primary jurisdiction over all the ICC/IP cases in the
absence of the grant of jurisdiction to the NCIP in Section 66 of the IPRA. Neither does the IPRA confer
original and exclusive jurisdiction to the NCIP over all claims and disputes involving rights of ICCs/IPs."
Furthermore,
That NCIP Administrative Circular 44 expands the jurisdiction of the NCIP as original and exclusive in Sections
5 and 1, respectively of Rule III x x x is of no moment. The power of administrative officials to promulgate
rules in the implementation of a statute is necessarily limited to what is provided for in the legislative
enactment.
The implementing rules and regulations of a law cannot extend the law or expand its coverage, as the power to
amend or repeal a statute is vested in the legislature. Indeed, administrative issuances must not override, but
must remain consistent with the law they seek to apply and implement. They are intended to carry out, not to
supplant or to modify, the law.
Perforce, in this case, the NCIP's Administrative Circulars1 classification of its RHO's jurisdiction as original
and exclusive, supplants the general jurisdiction granted by Batas Pambansa Bilang 129 to the trial courts and
ultimately, modifies and broadens the scope of the jurisdiction conferred by the IPRA on the NCIP. We cannot
sustain such a classification.
Lim v. Gamosa, GR No. 193964, Dec. 2, 2015
ENGINEER BEN Y. LIM, RBL FISHING CORPORATION, PALAWAN AQUACULTURE CORPORATION,
AND PENINSULA SHIPYARD CORPORATION, Petitioners, v. HON. SULPICIO G. GAMOSA, OFFICER-
IN-CHARGE, NCIP REGIONAL HEARING OFFICE, REGION IV AND TAGBANUA INDIGENOUS
CULTURAL COMMUNITY OF BARANGAY BUENAVISTA, CORON, PALAWAN, AS REPRESENTED
BY FERNANDO P. AGUIDO, ERNESTO CINCO, BOBENCIO MOSQUERA, JURRY CARPIANO,
Page 104

VICTOR BALBUTAN, NORDITO ALBERTO, EDENG PESRO, CLAUDINA BAQUID, NONITA SALVA,
AND NANCHITA ALBERTO, Respondents

The IPRA emphasizes customs and customary law to govern in the lives of the ICCs/IPs. It, however,
recognizes that customs and customary law cannot be applied to non-ICCs/IPs since ICCs/IPs are recognized as
a distinct sector of the Philippine society.
In Cruz v. DENR, Custom, from which customary law is derived, is also recognized under the Civil Code as a
source of law x x x. Customary law is a primary, not secondary, source of rights under the IPRA and uniquely
applies to ICCs/IPs. Its recognition does not depend on the absence of a specific provision in the civil law.
FACTS.
Respondent Tagbanua Indigenous Cultural Community of Barangay Buenavista, Coron, Palawan filed a petition
before the National Commission on Indigenous People (NCIP) against petitioners for “Violation of Rights to
Free and Prior and Informed Consent (FPIC) and Unauthorized and Unlawful Intrusion with Prayer for the
Issuance of Preliminary Injunction and TRO.”
Despite a motion to dismiss being a prohibited pleading under NCIP Administrative Circular No. 1-03,
petitioners moved to dismiss the petition on the ground, among others, that NCIP lack jurisdiction over the
subject matter of the petition because petitioners are not members of the ICC/IP. The NCIP, however, resolved
to deny the motion to dismiss. Likewise, the Court of Appeals affirmed the NCIP’s denial and reasoned out that
from the wording of Section 66 of the IPRA, the NCIP was bestowed with an all-encompassing grant of
jurisdiction over all claims and disputes involving rights of ICCs/IPs and that the requirement in the proviso
contained in the section i.e. obtaining certification from the Council of Elders/Leaders that the parties had
exhausted all remedies provided under their customary law prior to the filing of an action, applied only to
instances where both parties were members of an ICC/IP. In all, the Court of Appeals upheld that when a claim
or dispute involves rights of the ICCs/IPs, the NCIP has jurisdiction over the case regardless of whether the
opposing party is a non- ICC/IP.
Petitioners thus filed this petition for review on certiorari.
ISSUE.
Does the NCIP have jurisdiction over the subject matter of the instant case? HELD.
NO. SECTION 66 of the IPRA is exclusionary, specifically excluding disputes involving rights of ICCs/IPs
where opposing party is a non-ICC/IP. This provision reflects IPRA’s emphasis of customs and customary law
to govern in the lives of the ICCs/IPs. In fact, even the IPRA itself recognizes that customs and customary law
cannot be applied to non-ICCs/IPs since ICCs/IPs are recognized as a distinct sector of the Philippine society.

The limited or special jurisdiction of the NCIP, confined only to a special cause involving ICCs/IPs, can only be
exercised under the limitations and circumstances prescribed by the statute.
Former Chief Justice Reynato Puno, in his separate opinion in Cruz v. Secretary of Environment and Natural
Resources, emphasizes the primacy of customs and customary law in the lives of the members of the ICCs/IPs:
“Custom, from which customary law is derived, is also recognized under the Civil Code as a source of law x x
x. [I]n the absence of any applicable provisions in the Civil Code, custom, when duly proven, can define rights
and liabilities.
Customary law is a primary, not secondary, source of rights under the IPRA and uniquely applies to ICCs/IPs.
Its recognition does not depend on the absence of a specific provision in the civil law. The indigenous concept
of ownership under customary law is specifically acknowledged and recognized, and coexists with the civil law
concept and the laws on land titling and land registration.”
Once again, the primacy of customs and customary law sets the parameters for the NCIP’s limited and special
jurisdiction and its consequent application in dispute resolution. Demonstrably, the proviso in Section 66 of the
IPRA limits the jurisdiction of the NCIP to cases of claims and disputes involving rights of ICCs/IPs where both
parties are ICCs/IPs because customs and customary law cannot be made to apply to non-ICCs/IPs within the
parameters of the NCIP’s limited and special jurisdiction.
Page 105

Clearly, the phraseology of “all claims and disputes involving rights of ICCs/IPs” does not necessarily grant the
NCIP all-encompassing jurisdiction whenever the case involves rights of ICCs/IPs without regard to the status
of the parties, i.e, whether the opposing parties are both ICCs/IPs.
WHEREFORE, the appeal is granted. The NCIP’s Resolution is reversed and set aside and respondents may
refile their complaint against petitioners in a court of general jurisdiction

Tunged v. Sta. Lucia Realty, GR No. 231737. March 6, 2018


HEIRS OF TUNGED namely: ROSITA YARIS-LIWAN, VIRGIE S. ATIN-AN, BELTRAN P. SAINGAN,
MABEL P. DALING, MONICA Y. DOMINGO, and ELIZABETH Q. PINONO, Petitioners vs. STA. LUCIA
REALTY AND DEVELOPMENT, INC. and BAGUIO PROPERTIES, INC., Respondents

SUMMARY:The Heirs of Tunged are members of the Ibaloi Tribe who seek to protect their vested rights in the
subjectproperty under the IPRA and the environmental laws against the earthmoving activity of the respondents.
The RTC dismissed the case outright because the RTC, sitting as an environmental court, mistakenly believed
that the NCIP has jurisdiction over said complaint and that the petitioners had no legal personality to file the
petition as the RTC believed that the petitioners have admitted through their complaint that their right to said
property wasn’t yet established. The SC reverses the RTC decision because the NCIP had no jurisdiction over
the matter and that the cause of action wasn’t grounded on the recognition of their ownership but rather on the
enforcement of vested rights under the IPRA. Furthermore, the SC declared the petitioners to have sufficient
legal personality as the successors of Tunged as they haveproven such through documentary and testimonial
evidence.
FACTS:
 Petitioners in their complaint claimed, as members of the Ibaloi Tribe and as recognized IP’s, that the subject
property is ancestral land
o They claim that they have been occupying it in the concept of an owner since time immemorial and that said
ownership is recognized under the IPRA
 IPRA should protect their rights to sustainable traditional resource and against unlawful intrusion and
usurpation
o Their application for the issuance of Certificate of Ancestral Land Titles is pending with the NCIP
o They claim that respondents demolished and bulldozed throughout the land, causing destruction of trees
and Sayote plants along with other resources
 violated their rights pursuant to the IPRA
 violate environmental laws such as PD 1586
 violate the Environmental compliance certificate that was issued to respondents
 Baguio properties’ defense
o They invoked their ownership
o And that the petitioners were ultimately collaterally attacking the Torrens Titles of Respondent showing
their ownership.
 RTC Decision whilst acting as an Environmental Court: Dismissed complaint for lack of jurisdiction
o Recognition of petitioners’ rights as Indigenous People’s weren’t the proper subject of an environmental case
o RTC believed that the NCIP had the exclusive jurisdiction to settle said dispute
o RTC believed that even if the case was within its jurisdiction as an environmental court, the complaint
must be filed by the real party in interest
 RTC believes that petitioners aren’t the Real parties in interest since the relief they seek is
recognition of their ownership and hence the petitioners have admitted they have not
conclusively established their ownership as of that moment
 Filing was premature and hence have no legal personality to initiate the complaint.
 Petitioner’s MR
o NCIP has no jurisdiction since the NCIP only takes cognizance of claims between IPs and since Sta Lucia
Page 106

Realty and Baguio Properties aren’t IPs, the RTC has jurisdiction
o They insist that the relief they seek is their recognition of their rights under IPRA to their ancestral land
by virtue of their native title and NOT THE ISSUANCE OF THE Certificate of Ancestral Domain Title.  RTC
rejected their MR
 Hence this petition ISSUES + RULING:
1. Was the lower court’s outright dismissal proper? NO
o In determining which court has jurisdiction over the action, the allegations contained in the complaint
control, irrespective of whether or not the plaintiff is entitled to recover upon all or some of the claims asserted
therein. Averments in the complaint and character of the relief sought are to be consulted. (Unduran v
Aberasturi)
 IPRA Sec 66: Jurisdiction of the NCIP
 “Shall have jurisdiction over all claims and disputes involving rights of ICCs/IPs”
 “no such dispute shall be brought to the NCIP unless the parties have exhausted all
remedies provided in their customary laws. For this purpose, a certification shall be issued by the council of
Elders/Leaders who participated in the attempt to settle the dispute that the same has not been resolved.”
 AO 23-2008 in relation to BP 129
 Designates the RTC as a special court to hear, try and decide violations of environmental
law committed within its territorial jurisdiction
o SC examines the complaint
 Petitioners are members of Ibaloi Tribe
 Rightful ownership and possession have been established by testimonial and documentary
evidence since 1924
 They have possessed and exercised ownership since their ancestor died.
 Respondent’s intrusion and usurpation is alleged as well as a violation of the issued
Environmental Compliance Certificate.
 The Relief sought is an Environmental Protection Order to stop Respondent’s earthmoving
activities due to violation of the rights protected by IPRA and the ECC
 Recognize the rights of petitioners as IPs to their ancestral land  Compel respondents to restore the denuded
areas
 Damages for the lost resources
o SC believes outright dismissal improper on 4 grounds
 NCIP has no jurisdiction
 Sec 66 of the IPRA and the case cited above show that there are 2 conditions before the NCIP may take
cognizance of the dispute
 Exhaustion of remedies under customary laws of the parties
 Compliance with the precedent through certification by the council of elders
 These 2 conditions can only be complied with when the disputing parties are of the same
ICC/IP
 If 1 of the disputing parties is a non-IP/ICC, then said party is not bound by the customary
laws as contemplated by the IPRA
 Thus, the proper courts of justice will take cognizance
 RTChasjurisdiction
 Action is not one for a claim of ownership nor issuance of Certificate of Ancestral Domain Title
 Petitioners have already stated that the petition for Identification, delineation and recognition of their
ancestral claim is already pending with the NCIP
 Their Cause of action is grounded upon alleged earthmoving activities of respondents
 These activities allegedly violate their rights under IPRA and PD 1586 and cause
irreparable damage to the environment life and property
 The case falls under the RTC jurisdiction sitting as a special environmental court
 Petitioner’s entitlement to their claim is irrelevant.
Page 107

 PETITIONERS HAVE LEGAL PERSONALITY TO FILE THEIR COMPLAINT


 Petitioners supported their allegations with pertinent documents
 Report and recommendation of the NCIP in their petition for Identification,
Delineation and recognition of Ancestral Claim and Issuance of CALTs pending

before the NCIP  The report concluded that


 the petitioners have established themselves as the heirs of Tunged
 the subject land was proven to be part of the vast tract of land that Tunged and
his successors have possessed and occupied
 Petitioner’s averments and documents are sufficient to establish their Locus Standi in instituting this action’
 Arguendo: the complaint could have been re-raffled to a regular court
 Even if the case isn’t within the RTC’s jurisdiction as an environmental court
 It is the regular court, and not the NCIP which has jurisdiction as per AM NO 09-6-8-SC
 Explicitly states that if the complaint is not an environmental complaint the presiding judge shall refer it to
the executive judge for a re-raffle to a regular court
Disposition: premises considered the instant petition is granted

A4. Applications: Form and Contents. Dealings with Land

Benin v. Tuason, GIt No. GR No. L-26127, June 28, 1974


(Civil Case No. 3621 VICTOR BENIN, ET AL., plaintiffs-appellees,vs. MARIANO SEVERO TUASON y DE
LA PAZ, ET AL., defendants. J. M. TUASON & CO., INC., defendant-appellant.
Facts:
On May 19, 1955 three sets of plaintiffs filed three separate complaints containing substantially the same
allegations. In Civil Case No. 3621, the plaintiffs alleged that they were the owners and possessors of the three
parcels of agricultural lands, described in paragraph V of the complaint, located in the barrio of La
Loma (now barrio of San Jose) in the municipality (now city) of Caloocan, province of Rizal and that they
inherited said parcels of land from their ancestor Sixto Benin; In Civil Case No. 3622 the plaintiffs alleged that
they were the owners and possessors of two parcels of agricultural land, described in paragraph V of the
complaint, and that these parcels of land were inherited by them from their deceased father Bonoso Alcantara.
In Civil Case No. 3623, plaintiffs alleged that they are the owners and possessors of a parcel of agricultural land
located in the Barrio of La Loma (now San Jose), municipality of Caloocan, province of Rizal, having an area
of approximately 62,481 square meters; that this parcel of land was inherited by plaintiffs from their ancestor
Candido Pili who in turn inherited the same from his parents; and they and their predecessors in interest had
been in open, adverse and continuous possession of the same; had said lands declared for taxation purposes.
The plaintiffs in these three civil cases uniformly alleged, in their respective complaint, that sometime in the
year 1951 while they were enjoying the peaceful possession of their lands, the defendants, particularly the
defendant J.M. Tuason and Co. Inc., through their agents and representatives, with the aid of armed men, by
force and intimidation, using bulldozers and other demolishing equipment, illegally entered and started
defacing, demolishing and destroying the dwellings and constructions of plaintiffs' lessees, as well as the
improvements. They made inquiries regarding the probable claim of defendants, and in 1953 they discovered
for the first time that their lands, as described in their respective complaint, had either been fraudulently or
erroneously included, by direct or constructive fraud, in what appears as Parcel No. 1 (known as Santa Mesa
Estate) in Original Certificate of Title No. 735 of the Land Records of the province
Page 108

of Rizal in the names of the original applicants for registration, now defendants, Mariano Severo Tuason y de la
Paz, Teresa Eriberta Tuason y de la Paz, Juan Jose Tuason y de la Paz, Demetrio Asuncion Tuason y de la Paz,
and Augusto Huberto Tuason y de la Paz.
The plaintiffs in each of the three complaints also alleged that the registered owners had applied for the
registration of two parcels of land (known as the Santa Mesa Estate and the Diliman Estate; that the registration
proceedings were docketed as LRC No. 7681 of the Court of Land Registration; They allege that the application
for registration in LRC No. 7681, containing the boundaries, technical descriptions and areas of parcel No. 1
(Santa Mesa Estate) and parcel No. 2 (Diliman Estate) was published in the Official Gazette; that before the
decision was handed down in LRC No. 7681, the area, boundaries and technical descriptions of parcel No. 1
were altered and amended; that the area of parcel No. 1 as mentioned in Decree No. 17431 is bigger than the
area of parcel No. 1 appearing in the application for registration as published in the Official Gazette; that the
amendments and alterations, which were made after the publication of the original application, were never
published; that on March 7, 1914 a decision was rendered in LRC No. 7681 based on the amended plan; that
pursuant to the decision of March 7, 1914 a decree of registration was issued on July 6, 1914, known as Decree
No. 17431, decreeing the registration in the names of the applicants of the two parcels of land (Santa Mesa
Estate and Diliman Estate). They contend that the decision dated March 7, 1914 in LRC No. 7681 is null and
void because the Land Registration Court had no jurisdiction to render the decision for lack of publication; that
Decree No. 17431 issued pursuant to the decision of March 7, 1914 in LRC No. 7681 is likewise null and void
from the beginning, because it was issued pursuant to a void decision and that Original Certificate of Title No.
735, referring to parcel 1 (Santa Mesa Estate), is also null and void from the beginning because it was issued
pursuant to a void decree of registration. The lower court rendered a decision in favor of the plaintiffs. A motion
for new trial was filed by defendant J.M. Tuason & Co., Inc. on January 30, 1965. However, before the motion
for new trial was resolved by the court, said defendant, on February 11, 1965, filed a notice of appeal to this
Court and an appeal bond, and on February 12, 1965 he filed the record on appeal.
The record on appeal, after it had been corrected and amended, as ordered and/or authorized by the trial court,
was approved on September 29, 1965

Issue:
Whether or not the LRC had jurisdiction to render the decision for the reason that the amendment to the original
plan was not published.
Held:
The records show, and it is established by the evidence, that sometime in 1911 Mariano Severo Tuason y de la
Paz, Teresa Eriberta Tuason y de la Paz, Juan Jose Tuason y de la Paz, Demetrio Asuncion Tuason y de la Paz,
and Augusto Huberto Tuason y de la Paz, filed with the Court of Land Registration an application for the
registration of their title over two parcels of land, designated in the survey plans accompanying the application
as Parcel 1 with an area of 8,798,617 square meters, and Parcel 2 with an area of 16,254,037 square meters. The
application and the notice of hearing, containing the technical descriptions of the two parcels of land applied
for, were published in the issue of the Official Gazette of October 25, 1911. On December 23, 1911 the court
issued an order authorizing the amendment of the plan in LRC No. 7681. November 11, 1913 the applicants and
the Government entered into an agreement whereby the Government agreed to withdraw its opposition to the
application for registration of title over the portion known as Hacienda Diliman (Parcel 2) on condition that the
roads existing on said tract of land be allowed to remain. On December 29, 1913 the Court of Land Registration
rendered a decision which, among others, stated that during the registration proceedings the plans
accompanying the two applications were amended in order to exclude certain areas that were the subject of
opposition, that the order of general default was confirmed, that the Chief of the Surveyor's Division of the
Court of Land Registration was ordered to submit a report as to whether or not the new (amended) plans had
included lands which were not by the original plans, and whether or not the new plans had excluded the lands
that had already been covered by the decree in LRC No. 3563. In compliance with the Chief of the Survey
Division of the Court of Land Registration submitted a stating that the new plan of Parcel 1 in LRC No. 7681
did not include any land that had not been previously included in the original plan. The decree contains the
Page 109

technical description of the two parcels of land in accordance with the plan as amended. It appears in the decree
that Parcel 1 has an area of 8,798,644.10 square meters, more or less, or an increase of 27.10 square meters over
the area of 8,798,617 square meters that was stated in the

application for registration and in the notice of hearing which were published in the Official Gazette.
The trial court stressed on the point that publication is one of the essential bases of the jurisdiction of the court
to hear and decide an application for registration and to order the issuance of a decree of registration, as
provided in Act 496 (Land Registration Act). We believe that the lower court erred when it held that the Land
Registration Court was without jurisdiction to render the decision in LRC No. 7681. Under Section 23 of Act
496, the registration court may allow, or order, an amendment of the application for registration when it appears
to the court that the amendment is necessary and proper. If the amendment consists in the inclusion in the
application for registration of an area or parcel of land not previously included in the original application, as
published, a new publication of the amended application must be made. The purpose of the new publication is
to give notice to all persons concerned regarding the amended application. Without a new publication the
registration court cannot acquire jurisdiction over the area or parcel of land that is added to the area covered by
the original application, and the decision of the registration court would be a nullity insofar as the decision
concerns the newly included land.
The reason is because without a new publication, the law is infringed with respect to the publicity that is
required in registration proceedings, and third parties who have not had the opportunity to present their claim
might be prejudiced in their rights because of failure of notice.
But if the amendment consists in the exclusion of a portion of the area covered by the original application and
the original plan as previously published, a new publication is not necessary. In the case at bar, we find that the
original plan covering Parcel 1 and Parcel 2 that accompanied the application for registration in LRC No. 7681
was amended in order to exclude certain areas that were the subject of opposition, or which were the subject of
another registration case. , when the lower court said that the area of Parcel 1 in the decree of registration is
bigger than the area of Parcel 1 in the application as published, it did not mention the fact that the difference in
area is only 27.10 square meters. We believe that this difference of 27.10 square meters is too minimal to be of
decisive consequence in the determination of the validity of Original Certificate of Title No. 735. We believe
that this very slight increase of 27.10 square meters would not justify the conclusion of the lower court that "the
amended plan ... included additional lands which were not

originally included in Parcel 1 as published in the Official Gazette." It being undisputed that Parcel 1 has an
area of more than 8,798,600 square meters (or 879.86 hectares), we believe that this difference of 27.10 square
meters, between the computation of the area when the original plan was made and the computation of the area
when the amended plan was prepared, cannot be considered substantial as would affect the identity of Parcel 1.
It is the settled rule in this jurisdiction that only in cases where the original survey plan is amended during the
registration proceedings by the addition of lands not previously included in the original plan should publication
be made in order to confer jurisdiction on the court to order the registration of the area that was added after the
publication of the original plan.
The settled rule, further, is that once the registration court had acquired jurisdiction over a certain parcel, or
parcels, of land in the registration proceedings in virtue of the publication of the application, that jurisdiction
attaches to the land or lands mentioned and described in the application. If it is later shown that the decree of
registration had included land or lands not included in the original application as published, then the registration
proceedings and the decree of registration must be declared null and void in so far — but only in so far — as the
land not included in the publication is concerned. This is so, because the court did not acquire jurisdiction over
the land not included in the publication-the publication being the basis: of the jurisdiction of the court. But the
proceedings and the decree of registration, relating to the lands that were included in the publication, are valid.
Thus, if it is shown that a certificate of title had been issued covering lands where the registration court had no
jurisdiction, the certificate of title is null and void insofar as it concerns the land or lands over which the
registration court had not acquired jurisdiction.
Page 110

The court sited the Santiago case which states that,


(T)he mere fact that appellants herein were not personally notified of the registration proceedings that resulted
in a decree of registration of title in favor of the Tuasons in 1914 does not constitute in itself a case of fraud that
would invalidate the decree. The registration proceedings, as proceedings in rem, operate as against the whole
world and the decree issued therein is conclusive adjudication of the ownership of the lands registered, not only
against those parties who appeared in such proceedings but also against parties who were summoned by
publication

but did not appear. The registration by the appellees predecessors in interest freed the lands from claims and
liens of whatever character that existed against the lands prior to the issuance of the certificates of title, except
those noted in the certificate and legal encumbrances saved by law (Yumol vs. Sivera and Dizon, 64 Phil. 13, 17
and cases cited therein). In addition, there being no allegation that the registered owners procured the non-
appearance of appellants at the registration proceedings, and very much more than one year having elapsed
from the issuance of the decree of registration in 1914, neither revocation of such decree nor a decree of
reconveyance are obtainable any more.
The joint decision of the Court of First Instance, appealed from, is REVERSED and SET ASIDE
Lopez v. Querubin, GR NO. 155405, March 18, 2015

THE HEIRS OF EUGENIO LOPEZ, SR. NAMELY, OSCAR M. LOPEZ, MANUEL M. LOPEZ AND
PRESENTACION L. PSINAKIS, Petitioners, v. THE HONORABLE FRANCISCO QUERUBIN, IN HIS
CAPACITY AS PRESIDING JUDGE OF THE REGIONAL TRIAL COURT OF ANTIPOLO, BRANCH 74,
THE HEIRS OF ALFONSO SANDOVAL AND HIS WIFE ROSA RUIZ, REPRESENTED BY THEIR
ATTORNEY-IN-FACT, MRS. IMELDA RIVERA, Respondents

FACTS:
 Sandoval and Ozaeta filed an application for registration of title before the RTC of Pasig City (LRC No. N-
18887).The land registration court granted the application. The decision became final and executory.
 The National Land Titles and Deeds Administration (now LRA) issued Decree Nos. N-217643 and N-217644
in the names of Sandoval and his wife Rosa Ruiz, and Ozaeta and his wife Ma. Salome Lao.[5]
 Petitioners, heirs of Eugenio Lopez, Sr., filed a motion in LRC No. N-18887 alleging that Sandoval and
Ozaeta sold the lots subject of the application to the late Eugenio Lopez, Sr. Petitioners prayed that the court
consider in the land registration case the Deed of Absolute Sale over the lots executed by Sandoval and Ozaeta
and their respective spouses in favor of Eugenio Lopez, Sr. Invoking Section 22 of Presidential Decree No.
1529 (PD 1529), petitioners also prayed that the court issue the decree of registration in their names as the
successors-in-interest of Eugenio Lopez, Sr.
 18 August 1998: The Register of Deeds of Marikina City issued the corresponding OCT Nos. O-1603 and O-
1604 in favor of Sandoval and Ozaeta and their spouses.
 Petitioners filed another motion to declare void Decree Nos. N-217643 and N-217644 and Original
Certificate of Title (OCT) Nos. O-1603 and O-1604. Petitioners pointed out that the OCTs show that incumbent
Administrator Alfredo R. Enriquez signed the Decrees on 20 October 1997, before he assumed office on 8 July
1998.
 Petitioners questioned the inconsistencies in the dates and requested the LRA to recall the decrees. The LRA
Administrator denied the request and explained that the inconsistencies in the date was due to oversight and that
the decrees were actually issued sometime between August 8 and 13 1998.
 Petitioners filed with the Register of Deeds of Marikina City an application to annotate the notice of lis
pendens at the back of OCT Nos. O-1603 and O-1604 on the ground that petitioners have filed with the land
registration court a motion to declare OCT Nos. O-1603 and O-1604 void.
Page 111

 The Register of Deeds of Marikina City denied the application to annotate the notice of lis pendens.
 Three days after receipt of the letter, petitioners elevated the denial in consulta to the LRA. The case was
docketed
as Consulta No. 2879.
 The Ruling of the Land Registration Authority
o The LRA agreed with the Register of Deeds that a notice of lis pendens based on a motion is not registrable.
Relying on Section 24, Rule 14 of the Rules of Court, the LRA ruled that only a party to a case has the legal
personality to file a notice of lis pendens relative to the pending case.
o The LRA declared that petitioners are not parties in LRC No. N-18887. Since a land registration case is a
proceeding in rem, an order of general default binds the whole world as a party in the case. Petitioners are mere
movants whose personality the court has not admitted. Based on Section 26 of PD 1529, the LRA ruled that
petitioners should have filed a motion to lift the order of general default.
 The Ruling of the Court of Appeals
o The appellate court dismissed the petition for lack of merit and reiterated the LRAs ruling that only a party to
a case has the legal personality to file a notice of lis pendens. Petitioners have no legal personality because they
failed to file a motion to lift the order of general default in the land registration case
ISSUE(S): Whether a notice of lis pendens is registrable based on a motion to declare void the decrees and
titles.
HELD: NO

RATIO:
 Section 76 of PD 1529 states:
SECTION 76. Notice of lis pendens. No action to recover possession of real estate, or to quiet title thereto, or to
remove clouds upon the title thereof, or for partition or other proceedings of any kind in court directly affecting
the title to land or the use or occupation thereof or the buildings thereon, and no judgment, and no proceeding to
vacate or reverse any judgment, shall have any effect upon registered land as against persons other than the
parties thereto, unless a memorandum or notice stating the institution of such action or proceeding and the court
wherein the same is pending, as well as the date of the institution thereof, together with a reference to the
number of the certificate of title, and an adequate description of the land affected and the registered owner
thereof, shall have been filed and registered.
 As decreed by Section 76 of PD 1529, a notice of lis pendens should contain a statement of the institution of
an action or proceeding, the court where the same is pending, and the date of its institution. A notice of lis
pendens should also contain a reference to the number of the certificate of title of the land, an adequate
description of the land affected and its registered owner.
 The Register of Deeds denied registration of the notice of lis pendens because the application was bereft of
the original petition or complaint upon which this office will base its action
 Both the LRA and the appellate court denied the application for a notice of lis pendens because petitioners
are mere movants, and not original parties, in LRC No. N-18887. As petitioners are not parties to an action as
contemplated in Section 76 of PD 1529, they failed to present the requisite pleading to the Register of Deeds of
Marikina City. We hold that the Register of Deeds correctly denied the application for a notice of lis pendens

B. Publication, Opposition and Default

Republic v. Manna Properties, GR No. 146527, Jan. 31,2005


REPUBLIC OF THE PHILIPPINES, Petitioner, v. MANNA PROPERTIES, INC., Represented by its President,
JOSE TANYAO, Respondent
Page 112

FACTS: Applicant-appellee filed an Application for the registration of title of two (2) parcels of land. Copies of
the application, postal money orders for publication purposes and record were forwarded to the Land
Registration Authority by the Court a quo. The Opposition to the application stated, among others, that the
applicant is a private corporation disqualified under the new Philippine Constitution to hold alienable lands of
public domain. Applicant-appellee presented its president Jose Tanyao, who testified on the acquisition of the
subject property as well as Manuel Sobrepeña, co-owner of the subject property, who testified on the possession
of the applicant-appellee’s predecessors-in-interest.
The documentary evidence presented were: 1.) Survey Plan 2.) Technical Description of lots; 3) Certificate in
lieu of Lost Surveyor’s Certificate; 4) Certificate of Latest Assessment;5) Notice of Initial Hearing; 6)
Certificate of Publication of the Notice of Initial Hearing by the LRA, by the National Printing Office; and by
the Circulation Manager of the Ilocos Herald; 7) Clipping of the Notice of Initial Hearing; 8) Whole Issue of the
Ilocos Herald dated July 12, 1995; 9) Page 3 of Ilocos Herald dated January 12, 1995; 10) Sheriff’s Return of
Posting; 11) Certificate of Notification of all adjoining owners of the Notice of Initial Hearing on July 18, 1995.
Thereafter, the court a quo rendered a Decision granting the application. The OSG, appearing on behalf of
petitioner Republic of the Philippines ("petitioner"), promptly appealed the trial court’s decision to the CA. CA
dismissed petitioner’s appeal. Hence, this petition.
Petitioner asserts that Manna Properties has failed to prove its possession of the land for the period of time
required by law. Petitioner alleges that the trial court and the Court of Appeals based their findings solely on
their evaluation of the tax declarations presented by Manna Properties. Petitioner claimed in its opposition to
the application of Manna Properties that, as a private corporation, Manna Properties is disqualified from holding
alienable lands of the public domain, except by lease. Petitioner cites the constitutional prohibition in Section 3
of Article XII in the 1987 Constitution. Petitioner also claims that the land in question is still part of the public
domain.
On the other hand, Manna Properties claims that it has established that the land in question has been in the open
and exclusive possession of its predecessors-in-interest since the 1940s. Thus, the land was already private land
when Manna Properties acquired it from its predecessors-in-interest.
ISSUE: Whether or not Manna Properties Sufficiently Established Possession of the Land For the Period
Required by Law.
HELD: NO – The evidence on record does not support the conclusions of both the trial court and the Court of
Appeals. The governing law is Commonwealth Act No. 141 (Public Land Act) Sec. 48(b): Those who by
themselves or through their predecessors-in-interest have been in open, continuous, exclusive, and notorious
possession and occupation of agricultural lands of the public domain, under a bona fide claim of acquisition of
ownership, since June 12, 1945 or earlier, immediately preceding the filing of the application for confirmation
of title except when prevented by war or force majeure. These shall be conclusively presumed to have
performed all the conditions essential to a Government grant and shall be entitled to a certificate of title under
the provisions of this chapter.
Under CA 141, the reckoning point is June 12, 1945. If the predecessors-in-interest of Manna Properties have
been in possession of the land in question since this date, or earlier, Manna Properties may rightfully apply for
confirmation of title to the land. SC ruled that the land in question has not become private land and remains part
of the public domain.
The evidence submitted by Manna Properties to prove the required length of possession consists of the
testimony of one of its predecessors-in-interest, Manuel Sobrepeña ("Manuel"), transferee’s affidavits, and
several tax declarations covering the land in question.
While a tax declaration by itself is not sufficient to prove ownership, it may serve as sufficient basis for
inferring possession. However, the tax declarations presented by Manna Properties do not serve to prove their
cause. Although Manna Properties claimed during trial that they were presenting the tax declaration proving
possession since 12 June 1945, a scrutiny of the tax declaration reveals that it is not the tax declaration Manna
Properties claimed it to be. It was in fact a substitute tax declaration allegedly issued on 28 November 1950.
The annotation at the back of this tax declaration indicates that it was issued to replace the 1945 tax declaration
covering the land in question. A substitute is not enough.
Page 113

The 1945 tax declaration must be presented considering that the date, 12 June 1945, is material to this case. CA
141 specifically fixes the date to 12 June 1945 or earlier. A tax declaration simply stating that it replaces a
previous tax
declaration issued in 1945 does not meet this standard. It is unascertainable whether the 1945 tax declaration
was issued on, before or after 12 June 1945. Tax declarations are issued any time of the year. A tax declaration
issued in 1945 may have been issued in December 1945. Unless the date and month of issuance in 1945 is
stated, compliance with the reckoning date in CA 141 cannot be established.
Also, the tax declaration allegedly executed in 1950 and marked as it bears several irregularities. A small
annotation found at the bottom of the back page states that it cancels a previous tax declaration. Beyond stating
that the cancelled tax declaration was issued in 1945, it does not provide any of the required information that
will enable this Court or any interested party to check whether the original 1945 tax declaration ever existed.
The form used to prepare the tax declaration states that it was "FILED UNDER SECTION 202 OF R.A. 7160."
Republic Act No. 7160 is the Local Government Code of 1991. The sworn undertaking by the Deputy Assessor
who allegedly prepared the tax declaration reads, "Subscribed and sworn before me this 28 day of Nov. 1950..."
This means that the tax declaration was issued more than forty (40) years before the form used came into
existence. Manna Properties gave no explanation why its tax declaration used a form that did not exist at the
time of the alleged issuance of the tax declaration. The totality of these circumstances leads this Court to
conclude that the tax declaration was fabricated for the sole purpose of making it appear that Manna Properties’
predecessors-in-interest have been in possession of the land in question since 12 June 1945

Director of Lands v. Court of Appeals and Abistado, GR No. 102858, July 28, 1997

THE DIRECTOR OF LANDS, Petitioner, v. COURT OF APPEALS and TEODORO ABISTADO, substituted
by MARGARITA, MARISSA, MARIBEL, ARNOLD and MARY ANN, all surnamed ABISTO, Respondents.

Facts: Teodoro Abistado, private respondent, Filed a petition for original registration of his title over 648 square
meters of land under P.D. No. 1529 or the Property Registration Decree. The application was docketed as Land
Registration Case (LRC) No. 86 and assigned to Branch 44 of the Regional Trial Court of Mamburao,
Occidental Mindoro. During the pendency of the case, Teodoro Abistado died and was substituted by his
children - Margarita, Marissa, Maribel, Arnold, and Mary Ann, all surnamed Abistado, who were all represented
by their aunt Josefa Abistado, ad litem ( act in which a lawsuit has a representative in behalf of children not
capable of representation.)

Land Registration Court dismissed the petition for want of jurisdiction in compliance with the mandatory
provision requiring publication of initial public hearing in a newspaper of general circulation. Records show
that applicants failed to comply with P.D. No. 1529 Section 23 (1) requiring publication of notice of initial
hearing in a newspaper of general circulation.
Initial public hearing was only published in the Official Gazette.

The case was elevated to the Court of Appeals which granted the application and ordered the registration of title
to Teodoro Abistado, since publication in a newspaper of general Circulation is merely procedural, hence
dispensable. The Director of Land, represented by the Solicitor General, elevated this case to the Supreme
Court.

Issue: Whether or Not the Director of Land is correct that the publication of Notice of Initial hearing in a Land
Registration Case is mandatory.
Page 114

Held: Yes. Section 23 of P.D. No. 1529 shall be followed requiring a publication once both in the Official
Gazette and newspaper of general circulation. The Land Registration Case is an in Rem proceeding, meaning
the applicant must prove his title over the land against all persons concerned, who might have interest to right in
the property and should effectively be invited in the court to prove why the title should not be granted.

Such provision used the term "shall" which indicated that it is mandatory.
When the law speaks in clear and categorical language, there is no room for interpretation, vacillation, or
equivocation, there is room only for application.

Thus. Supreme Court affirmed the decision of the Lower Court dismissing the petition for registration of Land
Title to the respondents

Benin v. Tuason, GR No. L-26127, June 28, 1974

Roxas v. Court of Appeals, GR No. 118436, March 2l, 1997

Director, LMB v. Court of Appeals, GR No. 112567, Feb. 7, 2000

Republic v. Tiotioen, GR No. 167215, Oct. 8, 2008

Valisno v. Plan, GR No. L-551 52, Aug.19, 1986

Director of Lands v. Agustin, 42Ph1L.227 Regalado v. Republic, GR No. 168155, Feb. 15, 2007

Secs.23, 24 and27,PD No. 1529

C. Hearing, Judgment and Decree of Registration

Director of Lands v. Intermediate Appellate Court and Espartinez, GR No. 70825, March 11, l99l

Republic v. TA.N Properties, Inc., G.R. No. 154953, June 26,2008

Gaerlan v. Republic, GR No. GR No. 192717, March 12,2014

Republic v. Castuera, GR No. 203384 , Jan. 14,2015

Tan v. Republic, GR No. 177797, Dec. 4, 2008

Intestate Estate of the late Don Mariano San Pedro y Esteban v. Court of Appeals, 103727. December 18,
1996

Del Rosario v. Limcaoco, GR No.177392, Nov. 26,2012

Beumer v. Amores, GR No. 195670, Dec. 3, 2012

Secs. 7 and 8, Art. XII, Constitution

Republic v. CA and Maxino, GR No. L-56077,Feb.25, 1985


Page 115

Republic v. CA and Arquillo, GR No .62572, Feb. 15, 1990

Citizenship requirement

Borromeo v. Descallar, GR No. 159310, Feb. 24,2009

Republic v. Court of Appeals and Lapifia, GR No. 108998, N4.24,1994

Top Management Programs Corp. v. Fajardo, GR No.150462,Iune 15,2011

Republic v. CA and Lapina, GR No. 108998, Aug.24, 1994

Philippine Banking Corporation v. Li She, GR No. L-17587, Sept. 12, 1967

Cheesman v. Intermediate, Appellate Court, GR No. 74833, Jan.2l, l991

Galano v. Roxas, GR No. L-3T241, Sept. 12, 1975 o Beumer v. Amores, GR No. 195670, Dec. 3, 2012

Ong Ching Po v. Court of Appeals, GR No. I13472,Dec.20, 1994

Secs. 7 and 8, Art. XII, Constitution

BP Blg. 185, dated March 16,1982 (An Act to Implement Section Fifteen of Article XIV of the
Constitution)

RA No. 7042, as amended by RA No. 8179 (Foreign Investments Act of 1991

RA No. 9225 (Citizenship Retention and Re-acquisition Act)

Classification of Public Lands

Republic v. Fabio, GRNo. 159589, Dec.23,2008

Malabanan v. Republic, GRNo. 179987, April 29,2009

Resolution on motion for reconsideration, Sept. 3,2013

Fortuna v. Republic, GR No. 173423, March 5,2014

Republic v. Espinosa, G.R. No. 186603, April 5,2017

Secretary of the DENR v. Yap, GR No. 167707, Oct. 8, 2008

Sec. 2 and3, Art. XII, Constitution

Sec. 15, PD No. 705 (Revised Forestry Code)

Non-Registrable Properties
Page 116

Valiao v. Republic. GR No. 170757 . Nov. 28. 12011

Republic v. Sioson, GR No. L-13687. Nov. 29. 1 963

Central Mindanao Universitl, v. Republic. GR 195026, Feb.22.2016

Navv officers' Village Association. Inc, v. Republic. GR No. 177168, Aug. 3. 2015

Amunategui v. Director of Forestry, GR No. L-27873, Nov. 29, l983

Republic v. Court of Appeals and De la Rosa, GR No. L-43938, April 15, 1980

Republic v. Court of Appeals and Republic Real Estate Corporation. GR No. 103882. Nov. 25. 1998
Chavez v. Public Estates Authoritr. GR No. 1332250 May 6, 2003

Collado v. Court of Appeals, GR No. 107764, Oct. 4. 2002

Republic v. Southside Homeowners Association. Inc., GR No. 156951. Sept. 22, 2006

Arts. 419, 420,421 and 422, Civil Code Aft. 51, PD No. 1067 (Water Code)

Sec. 4, RA No. 7942 (Mining Act)

RA No. 7586 (National Integrated Protected Areas System Act)

Specific Evidence of Ownership

Republic v. East Silverlane Realty, GRNo. 196961, Feb.20,2012

Amunategui v. Director of Forestry, GRNo. L-30035, Nov.2, 1983

Intestate Estate of Don Mariano San Pedro v. Court of Appeals, GR No. 103727, Dec. 18,1996

Republic v. Alconaba, GRNo. 155012, April 14,2004

PD No. 892 (Discontinuance of the Spanish Mortgage System of Registration and of the Use of Spanish
Titles as Evidence in Land Registration Proceedings)

DENR Memorandum No. 564 (inrelation to DENR Administrative Order No. 2012-Oe)

DAO No. 20, dated May 30, 1988

Judgment

Manotok Realty v. CLT Realty, GR No. 123346

Dec. 14,2OO7 o Del Rosario v. Limcaoco. GR No. 177392

Nov. 26,2012. o Pascual v. Daquioag, GR No. 162063, March 31,2014


Page 117

Gomez v. Court of Appeals. 168 SCRA 503

Cayanan v. De los Santos,GR No. L-21150, Dec.26, 1967

Lucero v. Loot, GR No. L-16995, Oct. 26, 1968

Galano v. Rojas, GR No. L-31241, Sept. 12, 1975

Vencilao v. Vano, GR No. L-25660, Feb. 25, 1960

Laburada v. Land Registration Authority, G.R. No. 101387, March 11, 1998

Wee v. Mardo, GR No, 202474, June 4, 2014

Innocent Purchaser for Value and in Good Faith

Eland Philippines, Inc. v. Garcia, GR No. 173289, Feb. 17, 2010 (Footnotes 57 & 61, citing Agcaoili,
“Property Registration Decree”)

Dy v. Aldea, GR No. 219500, Aug. 9, 2017

Lozada v. Bracewell. GR No. 179155, April 2, 2014

Walstrom v. Mapa. GR No. 38387, Jan.29.1990

Sterling Investment Corporation v. Ruiz, GR No. L-30694, oct. 3 r, 1969

Narvasa v. Imbornal. GR No. 182908, Aug. 6, 2014

Roque v. Aguado. GR No. 193787 , April 7 , 2014

Republic v. Mangatora, GR No. 17037 5, luly 7, 2010

Caragav-Layno v. Court of Appeals, GR No. 52064, Dec. 6, 1984

Ono v. Lim, GR No. 154270, March 9, 2010

Concha v. Lumocso, GR No. 158121, Dec.21, 2007

Gonzales v. IAC, GR No. L-59622, Jan. 29, 1988

Yujuico v. Republic, GR No. 168661, oct. 26, 2007 (Footnote 17, citing Agcaoili, "Property Registration
Decree")

Pinero v. Director of Lands, GR No. L-36507, June 14, 1974

Roxas v. Garcia, GRNo. 146208, Aug. 13,2004 o De Guzman v. National Treasurer, GR No. l43ZB1, Aug.
3, 2000

Treasurer of the Philippines v. Court of Appeals, GR No. L-42805, Aug. 3 l, 1987


Page 118

Yared v. Tiongco, GR No. 161360, Oct. 19,2017

Sajonas v. Court of Appeals, GR No. 102377, July 5, 1996

Tajonerav. Courtof Appeals, GRNo. L-Z667l,March 27, 1981

St. Dominic Corporation v. IAC, GR No. 70623, June 30, 1987

Blanco v. Esquierdo, GR No. L- 15 I 82, Dec. 29, 7960

Cruz v. Bancom Corporation, GR No. L-l5182,Dec. 29 1960 9

PNB v. Jumanoy, GR No. 169901, Aug. 3, 2011

Uyv. Fule, GR No. 164961, June 30, 2014

Sajonas v. Court of Appeals, GR No. 102377, July 5, 1996

Mahinay v. Gako, GR No. 165338, Nov. 28, 201 1

CHAPTER 4
Certificate of Title
Manotok Realty v. CLT Realty, GR No. 123346.Dec. 14, 2007
MANOTOK REALTY, INC. and MANOTOK ESTATE CORPORATION, Petitioners, v. CLT REALTY
DEVELOPMENT CORPORATION, Respondent.

Facts:
G.R. No. 134385 (Dimson vs. Araneta)
In 1979, Jose Dimson filed with the RTC of Rizal, Caloocan City a complaint for recovery of possession and
damages against Araneta Institute of Agriculture, Inc. (Araneta). Dimson alleged that he was the absolute owner
of a 50-hectare land covered by TCT No. R- 15169 [Lot 25-A-2]. In November 1979, Dimson sold the property
to Virgilio Enriquez. Unfortunately, they discovered that the property is being occupied by Araneta who built
thereon an "agricultural school house”. Araneta maintained that it had been in possession of the subject land
since 1974. During trial, Araneta presented certifications from the Land Registration Commission attesting that
TCTs Nos. 13574 and 26538, covering the disputed property, are in the names of Araneta and Jose Rato,
respectively. Araneta also offered TCT No. 7784 to prove that it is the registered owner of the land described
therein. The trial court upheld the title of Dimson over the disputed property. The Court of Appeals sustained the
RTC Decision. The Court of Appeals likewise invalidated the titles of Araneta, relying on the Supreme Court
ruling in MWSS vs. Court of Appeals, which declared null and void the certificates of title derived from OCT
No. 994 registered on 3 may 1917.

G.R. No. 123346 (CLT vs. Manotok)


In 1992, CLT Realty Development Corporation (CLT) sought to recover from Manotok Realty, Inc. (MRI) and
Manotok Estate Corporation (MEC) (collectively, Manotoks) the possession of Lot 26 of the Maysilo Estate in
an action filed before the RTC of Caloocan City. CLT claims it is the owner of Lot 26 covered by TCT No. T-
177013 issued in its name by the Caloocan City Register of Deeds, which title was derived from Estelita
Hipolito. Hipolito’s title in turn emanated from Jose Dimson, the registered owner of TCT No. 15166, the latter
having acquired the same by virtue of a Court Order in 1966 issued by the court in a Civil Case. Dimson’s title
appears to have been sourced from OCT No. 994. Manotoks claimed that Dimson’s title, the proximate source
Page 119

of CLT’s title, was irregularly issued and, hence, the same and subsequent titles flowing therefrom are likewise
void.

Like CLT, the Manotoks likewise traced its title to OCT No. 994. TCT No. 4210, which cancelled OCT No.
994, had been issued to Alejandro Ruiz and Mariano Leuterio who had previously acquired the property in 1918
by virtue of an “Escritura de Venta” executed by Don Tomas Arguelles and Don Enrique Lopes. In 1920, Ruiz
and Leuterio sold the property to Francisco Gonzalez who held title thereto until 1938 when the property was
subdivided amongst the Gonzalez children.

The properties covered by said seven certificates of title (TCT Nos. 1368-1374) were expropriated by the
Republic of the Philippines. These properties were then later subdivided by the National Housing Authority
[NHA] into 77 lots and thereafter sold to qualified vendees. A number of said vendees sold 19 of these lots to
Manotok Realty, Inc. while 1 lot was purchased by the Manotok Estate Corporation.

The trial court, found in favor of CLT by adopting the factual findings arrived at by the majority commissioners
appointed to resolve the conflict of titles. It was found that that there were inherent technical infirmities or
defects on the face of TCT No. 4211 (4210), from which the Manotoks derived their titles
The Court of Appeals affirmed the decision of the RTC. Hence, the Manotoks filed the a petition for review
with the Supreme Court.

2005 Decision
The Supreme Court, through its Third Division, affirmed the RTC and Court of Appeals, which declared the
titles of CLT and Dimson as valid. The Supreme Court relied on the factual and legal findings of the trial courts,
as affirmed by the Court of Appeals. Both Manotok and Araneta filed their respective motions for
reconsideration.

2007 Resolution
In the 2007 Resolution on the MRs, the Supreme Court reversed and nullified its 2005 Decision and
categorically invalidated OCT No. 994 dated 19 April 1917, which was the basis of the claims of CLT and
Dimson. However, the Supreme Court resolved to remand the cases to this Special Division of the Court of
Appeals for reception of evidence. Both the heirs of Dimson and CLT had primarily relied on the validity of
OCT No. 994 dated 19 April 1917 as the basis of their claim of ownership. However, the Court in its 2007
Resolution held that OCT No. 994 dated 19 April 1917 was inexistent. The proceedings before the Special
Division afforded the Heirs of Dimson and CLT the opportunity to prove the validity of their respective claims
to title based on evidence other than the inexistent 19 April 1917 OCT No. 994.

Held:
The titles of Dimson, and its successor CLT, are declared void for being derived from an inexistent mother title
1. The mother title of TCT No. 15169, the certificate of title of Dimson covering the now disputed Lot 25-
A-2, is OCT No. 994 registered on 19 April 1917. Manifestly, the certificate of title issued to TCT No.
15169, and as a matter of course, the derivative title later issued to CLT, should both be voided inasmuch
as the OCT which they emanated had already been declared inexistent
2. The Special Division noted that the Heirs of Dimson did not offer any explanation why their titles reflect
the erroneous date of 19 April 1917. At the same time, it rejected CLT's explanation that the transcription
of the erroneous date was a "typographical error."
Nullity of Dimson's title
3. The heirs of Dimson relied on the 1977 Order of Judge Sayo, which was allegedly sourced from the
1966 Order of Judge Palma. Specifically, they point out that their title was issued pursuant to a court
order issued by Judge Palma in Case No. 4557 and entered in the memorandum of Encumbrance of OCT
No. 994. However, what is perplexing to this Court is not only the loss of the entire records of Case No.
Page 120

4557 but the admission of Judge Sayo that he had not seen the original of the Palma Order. Neither was
the signature of Judge Palma on the Order duly proven because all that was presented was an unsigned
duplicate copy with a stamped notation of "original signed." Equally perplexing is that while CFI Pasig
had a Case No. 4557 on file, said file pertained not to an LRC case but to a simple civil case. The case
did not partake of the nature of a registration proceeding and thus, evidently did not observe the
requirements in land registration cases.
4. Equally worthy of consideration is the fact that TCT No. 15169 indicates that not only was the date of
original registration inexistent, but the remarks thereon tend to prove that OCT No. 994 had not been
presented prior to the issuance of the said transfer certificate. The issuance of a transfer certificate of
title to the purchaser without the production of the owner's duplicate is illegal and does not confer any
right to the purchaser. (see Rodriguez v. Llorente, PNB vs. Fernandez)
5. The heirs of Dimson tracce their title from Jose Dimson's 25% share in the alleged hereditary rights of
Bartolome Rivera as an alleged grandson of Maria Concepcion Vidal. However, records show that
Rivera was 65 years old in 1963, thus, he must have been born around 1898. On the other hand, Vidal
was only 9 years in 1912; hence, she could have been born only on 1905. This alone creates an
unexplained anomalous wherein the alleged grandmother is 7 years younger than her alleged grandson.
Nullity of CLT's title
6. The findings regarding the titles of Jose Dimson necessarily affect and even invalidate the claims of all
persons who seek to derive ownership from the Dimson titles. These include CLT, which acquired the
properties they laid claim on from Estelita Hipolito who in turn acquired the same from Jose Dimson.
7. Considering that the land title of CLT carried annotations identical to those of Dimson and consequently
included the defects in Dimson's title, the fact that whatever typographical errors were not at anytime
cured by subsequent compliance with the administrative requirements or subjected to administrative
correction bolsters the invalidity of the CLT title due to its complete and sole dependence on the void
Dimson title.
Validity of Araneta's title
8. From the titles submitted, the predecessor-in-interest of Araneta was Jose Ma. Rato y Tuazon, one of the
co-heirs named in OCT No. 994, who was allotted a share of the Maysilo Estate. Rato was issued TCT
No. 8692 covering Lot No. 25-A-3, which was then subdivided into five lots under TCT Nos. 21855-59.
Further subdividing the property, Rato was again issued TCT Nos. 26538 and 26539, still covering Lot
No. 25 A-3-C. In all the certificates of title, including those that ultimately passed ownership to Araneta,
the designation of the lot as either belonging to or portions of Lot 25-A-3 was retained, thereby proving
identity of the land. More importantly, the documentary trail of land titles showed that all of them were
derived from OCT No. 994 registered on 3 May 1917.
9. This Court finds that the incorrect entry with respect to the Decree and Record Number appearing on the
title of Araneta's predecessor-in-interest cannot, by itself, invalidate the titles of Araneta's predecessors-
in-interest and ultimately, that of Araneta. The incorrect entries alluded to would not have the effect of
rendering the previous titles void sans any strong showing of fraudulent or intentional wrongdoing on
the part of the person making such entries. Fraud is never presumed but must be established by clear and
convincing evidence.
In case of discrepancy, the technical description in the title should prevail over the record number
10. The court has acknowledged that certain defects on a certificate of title, specifically, the interchanging
of numbers, may occur and "it is certainly believable that such variance in the copying of entries could
be merely a typographical or clerical error” In such cases, the technical description in the title should
prevail over the record number. (see Encinas v. National Bookstore, Inc.)
11. What is of utmost importance is that the designation and the technical description of the land, as stated
on the face of the title, had not been shown to be erroneous or otherwise inconsistent with the source of
titles. In Araneta's case, despite the incorrect entries on the title, the properties, covered by the subject
certificates of title can still be determined with sufficient certainty.
Certificate of title number may bear the same number as another title to another land
Page 121

12. There is nothing fraudulent for a certificate of title to bear the same number as another title to another
land. This came about because under General Land Registration Office (GLRO) Circular No. 17, dated
February 19, 1947, and Republic Act No. 26 and Circular No. 6, RD 3, dated August 5, 1946, the titles
issued before the inauguration of the Philippine Republic were numbered consecutively and the titles
issued after the inauguration were numbered also consecutively starting with No. 1, so that eventually,
the titles issued before the inauguration were duplicated by titles issued after the inauguration of the
Philippine Republic.
Validity of the Manotok title : Titles acquired by the State by way of expropriation are deemed
cleansed of whatever previous flaws may have attended these titles
13. As mentioned above, the properties covered by TCT Nos. 1368-1374 were expropriated by the Republic
of the Philippines and were eventually subdivided and sold to various vendees. The fact of expropriation
is extremely significant, for titles acquired by the State by way of expropriation are deemed cleansed of
whatever previous flaws may have attended these titles.
14. In an rem proceeding, condemnation acts upon the property. After condemnation, the paramount title is
in the public under a new and independent title; thus, by giving notice to all claimants to a disputed title,
condemnation proceedings provide a judicial process for securing better title against all the world than
may be obtained by voluntary conveyance."(see Republic v. Court of Appeals, andReyes v. NHA)
15. In annulling the Manotok titles, focus was laid on the alleged defects of TCT No. 4211 issued in
September of 1918. However, TCT No. 4211 was issued decades before the property was expropriated.
Thus, any and all defects that may have attended that particular title would have been purged when the
property covered by it was subsequently acquired by the State through eminent domain.
16. The majority report focused on the alleged flaws and inherent technical defects of TCT Nos. 4211, 5261
and 35486, ranging from the language of the technical descriptions, absence of subdivision plan, lot
number and survey plan. The imputed flaws affect only those certificates of title issued prior to those
registered in the name of the Republic. Remarkably, no specific flaw was found on the MANOTOKS'
titles indicating any irregularity on their issuance.
Difference between the imputed flaws tainting the contending titles
17. The crucial difference between the imputed flaws allegedly tainting said contending titles, Dimson and
CLT on one hand, and the Manotoks and Araneta, on the other, is that the imputed flaws purportedly
beleaguering the respective certificates of title of the Manotoks and Araneta relate to the mechanical and
technical aspect of the transcription of their titles and are therefore inconsequential to the import and
validity thereof. Said imputed flaws do not depart from the fact that the predecessors-in-interest of the
Manotoks and Araneta had been clothed with the right of ownership over the disputed portions of the
Maysilo Estate.
18. On the other hand, the flaws attending the titles of Dimson and CLT primarily stem from infirmities
attending or otherwise affecting the very crux of their claim of ownership. Having derived their titles
from Rivera, whose title is questionable and dubious to the core, Dimson and CLT cannot rightly insist
on the validity of their titles. Such flaws are hard to overcome as they delve into the substance of their
proprietary claims.

Ping v. Ayala Land, Inc., GR No.173120, April 10, 2019 (Footnotes 40, 41 & 42, citing Agcaoili, Property
Registration Decree')
SPOUSES YU HWA PING and MARY GAW, Petitioners vs. AYALA LAND, INC., Respondent

Facts: (Consolidated cases: Diaz case and Yu case)


1. Petitioners Spouses Diaz submitted to the General Land Registration Office for approval of the Director
of Lands a survey plan designated as Psu-25909, which covered a parcel of land located at Sitio of Kay
Monica, Barrio Pugad Lawin, Las Piñas, Rizal, which was then approved.
Page 122

2. Another survey plan was done covering the same parcel of land designated for a certain Mayuga. The
said survey, however, stated that the lot was situated at Sitio May Kokek, Barrio Almanza, Las Piñas,
Rizal.
3. Another survey (PSU-80886) was undertaken for a certain Guico. Again, the survey indicated a
different address that the lots were situated in Barrio Tindig na Mangga, Las Piñas, Rizal.
4. An additional survey plan (Psu-80886/SWO-20609) was executed over the similar parcel of land for a
certain Yaptinchay.
5. OCT No. 242 and 244 were issued in favor of Yaptinchay covering Lot 2 and Lot 3.
6. OCT 1609 covering Lot 3 was issued in favor of Mayuga.
7. Some of properties were sold to CPJ Corporation resulting in the issuance of TCT No. 190713 in its
name.
8. Petitioner Andres Diaz filed a petition for original registration before the CFI of Pasay for Lot No. 1 of
Psu-25909 which was granted. OCT 8510 was issued in the name of Spouses Diaz. The Spouses Diaz
subdivided their 460,626-square- meter property covered by such OCT into 10 lots, described as Lots
No. 1-A to 1- J and conveyed to different third parties.
9. CPJ Corporation, then owner of the land covered by TCT No. 190713, which originated from OCT No.
242, filed a Land Registration before the RTC of Pasig City against Spouses Diaz (Diaz Case). It sought
to review OCT No. 8510 in the names of Spouses Diaz on the ground that the interested persons were
not notified of the application.
10. On August 30, 1976 and December 4, 1976, Andres Diaz sold to Cabautan the following parcels of
land, which originated from OCT No. 8510.
11. Petitioner Spouses Yu acquired ownership over 67,813 square meters representing the undivided half-
portion of Lot 1-A originating from OCT No. 8510 of Spouses Diaz. The said property was co-owned
by Spouses Diaz with Spouses Librado and Susana Cabautan resulting from a civil case decided by the
RTC of Makati.
12. Spouses Yu then acquired ownership over Lot 1-B originating from OCT No. 8510 of Spouses Diaz
with an area of 135,000 square meters. Pursuant to the transfers of land to Spouses Yu, TCT Nos. 39408
and 64549 were issued in their names.
13. On the other hand, CPJ Corporation transferred their interest in the subject properties to third persons.
14. Later, in 1988, Ayala Corporation obtained the subject properties from Goldenrod, Inc. and PESALA. In
1992, pursuant to the merger of respondent Ayala Land, Inc. (ALI) and Las Piñas Ventures, Inc., ALI
acquired all the subject properties.

Issues:
1. Whether the validity of the surveys of OCT nos. 242, 244 and 1609 as against OCT no. 8510 can be
assailed in the present case.
2. Whether the alleged errors in PSU-80886 and PSU-47035 are of such degree so as to invalidate OCT
nos. 242, 244 and 1609 and its transfer certificates of titles.

Ratio:
The rule that between 2 conflicting titles, the title registered earlier prevails is not absolute. If the inclusion of
the land in the earlier registered title was a result of a mistake, then the latter registered title will prevail. The
ratio decidendi of this exception is to prevent a title that was earlier registered, which erroneously contained a
parcel of land that should not have been included, from defeating a title that was later registered but is
legitimately entitled to the said land. It reinforced the doctrine that “registering a piece of land under the Torrens
System does not create or vest title because registration is not a mode of acquiring ownership. A certificate of
title is merely an evidence of ownership or title over the particular property described therein.

The survey of the registered land may be scrutinized by the courts when compelling reasons exist. Although a
certificate of title serves as evidence of an indefeasible and incontrovertible title to the property in favor of the
Page 123

person whose name appears therein, it is not a conclusive proof of ownership. It is a well-settled rule that
ownership is different from a certificate of title. The fact that a person was able to secure a title in his name does
not operate to vest ownership upon him of the subject land. Registration of a piece of land under the Torrens
System does not create or vest title, because it is not a mode of acquiring ownership. A certificate of title is
merely an evidence of ownership or title over the particular property described therein. It cannot be used to
protect a usurper from the true owner; nor can it be used as a shield for the commission of fraud; neither does it
permit one to enrich himself at the expense of others. Its issuance in favor of a particular person does not
foreclose the possibility that the real property may be co- owned with persons not named in the certificate, or
that it may be held in trust for another person by the registered owner.

Psu-47035, Psu-80886 and Psu-80886/SWO-20609 contain numerous and serious irregularities which cast
doubt on the validity of OCT Nos. 242, 244 and 1609

In Cambridge Realty and Resources Corp. v. Eridanus Development, Inc., it was ruled that a case of
overlapping of boundaries or encroachment depends on a reliable, if not accurate, verification survey; barring
one, no overlapping or encroachment may be proved successfully, for obvious reasons. The first step in the
resolution of such cases is for the court to direct the proper government agency concerned to conduct a
verification or relocation survey and submit a report to the court, or constitute a panel of commissioners for the
purpose.

The court agrees with the findings of the RTC of Las Pinas and the CA that there were numerous defects and
anomalies in the survey plans
1. It is dubious how the same surveyor conducted Psu-47035, Psu-80886 and Psu- 80886/SWO-20609
even though an earlier survey on Psu-25909, which the surveyor should obviously be aware, was
already conducted on the same parcel of land.
2. Even though a single entity conducted the surveys, the lands therein were described to be located in
different places.
3. There is a discrepancy as to who requested the survey of Psu-47035. The photocopy of Psu-47035 as
submitted by ALI shows that it was done for a certain Estanislao Mayuga. On the other hand, the
certified true copy of Psu-47035 depicts that it was made for Dominador Mayuga.
4. Psu-80886 does not contain the signature of then Director of Lands, Serafin P. Hidalgo; rather, the
prefix “Sgd.” was simply indicated therein.
5. Psu-80886 was issued on July 28, 1930 but it referred to a specific monument described as B.L.L.M
No. 4. According to the LMB-DENR, the said monument was only established on November 27, 1937,
more than seven years after Psu- 80886 was issued.\
6. ALI attempted to explain this anomaly by stating that Psu- 80886 was amended by Psu-80886/SWO-
20609, a Special Work Order, in view of the discrepancies of the former. While Psu- 80886/SWO-
20609 is dated March 6, 1931, ALI insists that it was actually conducted in 1937 and approved in 1940.
However, the CA noted that said testimony crumbled under cross-examination as ALI’s witness, Engr.
Cortez, could not reaffirm the said justification for Psu- 80886’s manifest error of including a latter
dated monument.
7. ALI explained that Psu-80886/SWO-20609 was undertaken to correct a discrepancy in Psu-80886. It
does not, however, erase the fact that Psu-80886, from which ALI’s titles originated, is marred with
irregularities. This is a badge of fraud that further runs counter to the legitimacy of the surveys that ALI
relied upon.
8. RTC Las Piñas stated that “the total area of the property covered by the document bear many erasures,
particularly two erasures as to the total area in terms of number and one erasure as to that total area in
terms of unit of measurement.
Page 124

9. RTC of Las Piñas added that “[a]n examination of the same reveals that the lower right hand corner of
the plan, which bears the serial number PSU-80886, is manifestly different from the main document in
terms of the intensity of its contrast.
10. A reading of the dispositive portion of the CA decision in Guico v. San Pedro does not categorically
state that Lot Nos. 2 and 3 were absolutely and completely awarded to Guico. The award of the said lots
was subject to the vital and primordial condition or obligation to present to the court an amended,
properly approved, plan to the Director of Lands. Evidently, the Court was not satisfied with Psu- 80886
because it lacked the requisites for a valid survey. Thus, it required Guico to secure an amended and
correctly approved plan, signed by the Director of Lands. The records, however, did not show that
Guico indeed secured an amended and properly approved plan.
11. The Court also agrees with the finding of the CA that Psu-25909 bears all the hallmarks of verity. It
was established that Andres Diaz was the very first claimant of the subject property and was the
proponent of Psu- 25909. The said survey clearly contained the signature of the surveyor and the
Director of Lands, as can be seen on its face. In stark contrast with Psu-80886, which contained
alterations and erasures, Psu-25909 has none. The original of Psu-25909 was likewise on file with the
Bureau of Lands and a microfilm reproduction was readily obtained from the file of the said office,
unlike in Psu-80886 and Psu-47909.

The Court cannot subscribe to the finding of the CA that the numerous defects are “not enough to
deprive the assailed decree of registration of its conclusive effect, neither are they sufficient to arrive at
the conclusion that the survey was definitely, certainly, conclusively spurious.” The Court cannot close
its eyes to the blatant defects on the surveys upon which the original titles of ALI were derived simply
because its titles were registered. To allow these certificates of title in the registration books, even
though these were sourced from invalid surveys, would tarnish and damage the Torrens system of
registration, rather than uphold its integrity.

It is an enshrined principle in this jurisdiction that registration is not a mode of acquiring ownership. A
certificate of title merely confirms or records title already existing and vested. The indefeasibility of a
Torrens title should not be used as a means to perpetrate fraud against the rightful owner of real
property. Good faith must concur with registration because, otherwise, registration would be an exercise
in futility. A Torrens title does not furnish a shield for fraud, notwithstanding the long- standing rule that
registration is a constructive notice of title binding upon the whole world. The legal principle is that if
the registration of the land is fraudulent, the person in whose name the land is registered holds it as a
mere trustee.

When a land registration decree is marred by severe irregularity that discredits the integrity of the Torrens
system, the Court will not think twice in striking down such illegal title in order to protect the public against
unscrupulous and illicit land ownership.

Thus, due to the numerous, blatant and unjustifiable errors in Psu-47909, Psu- 80886, and Psu-80886/SWO-
20609, these must be declared void. Likewise, OCT Nos. 242, 244, and 1609, their transfer certificates, and
instruments of conveyances that relied on the anomalous surveys, must be absolutely declared void ab initio.

When a land registration decree is marred by severe irregularity that discredits the integrity of the Torrens
system, the Court will not think twice in striking down such illegal title in order to protect the public against
unscrupulous and illicit land ownership.

Thus, due to the numerous, blatant and unjustifiable errors in Psu-47909, Psu- 80886, and Psu-80886/SWO-
20609, these must be declared void. Likewise, OCT Nos. 242, 244, and 1609, their transfer certificates, and
instruments of conveyances that relied on the anomalous surveys, must be absolutely declared void ab initio.
Page 125

Footnote:
Republic v. Ayala y Cia,40 the Court confronted the validity of the surveys conducted on the lands to determine
whether the title was properly subdivided. It was ruled therein that subdivision plan Psd-27941 was erroneous
because it was "prepared not in accordance with the technical descriptions in TCT No. T-722 but in disregard of
it, support the conclusion reached by both the lower court and the Court of Appeals that Lots 49 and 1 are
actually part of the territorial waters and belong to the State."41 Accordingly, the sole method for the Court to
determine the validity of the title was to dissect the survey upon which it was sourced. As a result, it was
discovered that the registered titles therein contained areas which belong to the sea and foreshore lands.

Here, only a direct review of the surveys of OCT Nos. 242, 244, and 1609, as well as OCT No. 8510 can
resolve the issue on the validity of these titles. The findings of the RTC of Las Piñas and the CA differ with
respect to the cited errors in the surveys. The Court is convinced that through a rigorous study of the affected
surveys, the valid owners of the subject properties are can be finally adjudicated.

Finally, after resolving the various preliminary issues, the Court can now tackle the crux of these petitions - the
validity of Psu-25909, Psu-47035, Psu-80886, and Psu-80886/SW0-20609. The resolution of this issue will
decisively determine the true and rightful owner of the subject properties.

Psu-47035, Psu-80886 and


Psu-80886/SW0-20609 contain
numerous and serious irregularities
which cast doubt on the validity of
OCT Nos. 242, 244 and 1609

At the onset, the present case poses an issue on the validity of registered and overlapping titles based on their
surveys. The Court must commend the RTC of Las Piñas for taking the correct procedure in resolving such
issue.

In Cambridge Realty and Resources Corp. v. Eridanus Development, Inc.,42 it was ruled that a case of
overlapping of boundaries or encroachment depends on a reliable, if not accurate, verification survey; barring
one, no overlapping or encroachment may be proved successfully, for obvious reasons. The first step in the
resolution of such cases is for the court to direct the proper government agency concerned to conduct a
verification or relocation survey and submit a report to the court, or constitute a panel of commissioners for the
purpose. In that case, the Court lamented that the trial court therein did not order the conduct of a verification
survey and the appointment of geodetic engineers as commissioners, to wit:

This is precisely the reason why the trial court should have officially appointed a commissioner or panel of
commissioners and not leave the initiative to secure one to the parties: so that a thorough investigation, study
and analysis of the parties' titles could be made in order to provide, in a comprehensive report, the necessary
information that will guide it in resolving the case completely, and not merely leave the determination of the
case to a consideration of the parties' more often than not self-serving evidence.43

Hi-lon Manufacturing, Inc. v. Commission on Audit, GR No. 210669, August 1, 2017


HI-LON MANUFACTURING, INC., Petitioner, v. COMMISSION ON AUDIT, Respondent.

Facts:
Sometime in 1978, the government, through the then Ministry of Public Works and Highways (now DPWH),
converted to a road right-of-way (RROW) a 29,690 sq. m. portion of the 89,070 sq. m. parcel of land located in
Page 126

Mayapa, Calamba, Laguna, for the Manila South Expressway Extension Project. The subject property was
registered in the name of Commercial and Industrial Real Estate Corporation ( CIREC) under Transfer
Certificate of Title (TCT) No. T-40999. Later on, Philippine Polymide Industrial Corporation (PPIC) acquired
the subject property, which led to the cancellation of TCT No. T-40999 and the issuance ofTCT No. T..:120988
under its name. PPIC then mortgaged the subject property with the De~elopment Bank of the Philippines
(DBP), a government financing institution, which later acquired the property in a foreclosure proceeding on
September 6, 1985. TCT No. T-120988, under PPIC's name, was then cancelled, and TCT No. T-151837 was
issued in favor ofDBP. Despite the use of the 29,690 sq. m. portion of the property as RROW, the government
neither annotated its claim or lien on the titles of CIREC, PPIC and DBP nor initiated expropriation
proceedings, much less paid just compensation to the registered owners.

Issue: Whether or not HI-LON is entitled to just compensation for the 29,690 sq. m. portion of the subject
property?

Held: NO.
Having determined that HI-LON or its predecessor-in-interest TGPI does not own the RROW in question, as it
has been the property of the Republic of the Philippines since its acquisition by the DBP up to the present, the
COA concluded that the proper valuation of the claim for just compensation is irrelevant as HI-LON is not
entitled thereto in the first place. Concededly, the 29,690 sq. m. portion of the subject property is not just an
ordinary asset, but is being used as a RROW for the Manila South Expressway Extension Project, a road
devoted for a public use since it was taken in 1978. Under the Philippine Highway Act of 1953, "right-of-way"
is defined as the land secured and reserved to the public for highway purposes, whereas "highway" includes
rights-of-way, bridges, ferries, drainage structures, signs, guard rails, and protective structures in connection
with highways. Article 420 of the New Civil Code considers as property of public dominion those intended for
public use, such as roads, canals, torrents, ports and bridges constructed by the state, banks, shores, roadsteads,
and others of similar character. Being of similar character as roads for public use, a road right-of-way (RROW)
can be considered as a property of public dominion, which is outside the commerce of man, and cannot be
leased, donated, sold, or be the object of a contract, except insofar as they may be the object of repairs or
improvements and other incidental matters. However, this RROW must be differentiated from the concept of
easement of right of way under Article 649 of the same Code, which merely gives the holder of the easement an
incorporeal interest on the property but grants no title thereto, 20 inasmuch as the owner of the servient estate
retains ownership of the portion on which the easement is established, and may use the same in such a manner
as not to affect the exercise of the easement.

Wee v. Mardo. GR No. 202414, June 4, 2014


JOSEPHINE WEE, Petitioner, vs. FELICIDAD MARDO, Respondent.

Facts:
Respondent Felicidad Mardo was granted a registered Free Patent covering a lot situated in Silang, Cavite. She
allegedly conveyed a portion of the said lot to petitioner, Josephine Wee, through a deed of absolute sale.
However, respondent refused to vacate and turn over the subject property claiming that the alleged sale was
falsified. Relying on the aforementioned deed of absolute sale, petitioner filed an Application for Original
Registration of the said parcel of land. The respondent opposed such application alleging 1] that she is the true
and lawful owner of the parcel of land; and 2] that petitioner’s deed of absolute sale is surreptitious. During the
pendency of the case, respondent managed to register the land in her name.

The RTC rendered a Decision granting the application of petitioner. The trial court held that Josephine Wee is
qualified to register the subject land in her name, and directed the Administrator of LRA to issue the
corresponding decree in her name based on the plan and technical description of said land as submitted by the
Page 127

applicant and the Register of Deeds of the Province of Cavite to issue title in her name. The Court of Appeals,
upon appeal by respondent, reversed the decision of the trial court, thereby denying the application for
registration by petitioner.

The CA held, among others, that petitioner was not able to comply with the requirement of possession and
occupation under Sec. 14 (1) of P.D. No. 1529. Her admission that the subject lot was not physically turned over
to her due to some objections and oppositions to her title suggested that she was not exercising any acts of
dominion over the subject property, an essential element in the requirement of possession and occupation
contemplated under Sec. 14 (1) of P.D. No. 1529.

Issue:
Whether or Not the adverse, open and notorious possession and occupation of a person and his
predecessor-in-interest over a parcel of land can defeat a certificate of title.

Ruling:
The petition deserves no merit.

In the case of Republic vs. Umali, this Court ruled that once a patent is registered and the corresponding
certificate of title is issued, the land ceases to be part of public domain and becomes private property over which
the Director of Lands has neither control nor jurisdiction. A public land patent, when registered in the
corresponding Register of Deeds, is a veritable Torrens title, and becomes as indefeasible upon the expiration of
one (1) year from the date of issuance thereof. Said title, like one issued pursuant to a judicial decree, is subject
to review within one (1) year from the date of the issuance of the patent. This rule is embodied in Section 103 of

PD 1529, which provides that:


Section 103. Certificates of title pursuant to patents. – Whenever public land is by the Government alienated,
granted or conveyed to any person, the same shall be brought forthwith under the operation of this Decree. x x x
After due registration and issuance of the certificate of title, such land shall be deemed to be registered land to
all intents and purposes under this Decree.

Accordingly, respondent’s registered patent in the corresponding Registry of Deeds is a veritable Torrens title
and becomes as indefeasible as a Torrens title upon the expiration of one (1) year from the date of its issuance.
For said reason, the order of the RTC directing the Administrator of LRA to issue a corresponding decree in
petitioner’s name is null and void. A land registration court has no jurisdiction to order the registration of land
already decreed in the name of another in an earlier land registration case. A second decree for the same land
would be null and void, since the principle behind the original registration is to register a parcel of land only
once.

Verily, once a title is registered, as a consequence either of judicial or administrative proceedings, the owner
may rest secure, without the necessity of waiting in the portals of the court sitting in the mirador de su casa to
avoid the possibility of losing his land. The certificate of title cannot be defeated by adverse, open and notorious
possession. Neither can it be defeated by prescription. As provided under Sec. 47 of PD 1529, no title to
registered land in derogation of the title of the registered owner shall be acquired by prescription or adverse
possession.

Lacbayan v. Samoy. Jr. GR No. 165427. March 2l, 2011


BETTY B. LACBAYAN, Petitioner, vs. BAYANI S. SAMOY, JR., Respondent.

Facts:
Page 128

During Betty Lacbayan and Bayani Samoy’s illicit relationship, they, together with three more incorporators,
were able to establish a manpower services company, by which they acquired 5 parcels of land, registered in
their names, ostensibly as husband and wife. Having parted ways eventually, both of them agreed to divide the
said properties and terminate their business partnership by executing a Partition Agreement.

Initially, Samoy agreed to Lacbayan's proposal that the properties in Malvar St. and Don Enrique Heights be
assigned to the latter, while the ownership over the three other properties will go to Samoy.
However, when Lacbayan wanted additional demands, Samoy refused.

Thus, Lacbayan filed a complaint for judicial partition of the said properties before the Quezon City RTC. In his
Answer, however, Samoy denied Lacbayan's claim of cohabitation and said that the properties were acquired
out of his own personal funds without any contribution from her.

Issues: Does an action for partition preclude a settlement on the issue of ownership?

Held: No.

While it is true that the complaint involved here is one for partition, the same is premised on the existence or
non-existence of co-ownership between the parties. Petitioner insists she is a co-owner five real estate
properties based on the TCTs covering the subject properties, pro indivison of the Respondent maintains
otherwise. Indubitably, therefore, until and unless this issue of co-ownership is definitely and finally resolved, it
would be premature to effect a partition of the disputed properties.

More importantly, the complaint will not even lie if the claimant, or petitioner in this case, does not even have
any rightful interest over the subject properties. A careful perusal of the contents of the so-called Partition
Agreement indicates that the document involves matters which necessitate prior settlement of questions of law,
basic of which is a determination as to whether the parties have the right to freely divide among themselves the
subject properties. Moreover, to follow petitioner's argument would be to allow respondent not only to admit
against his own interest but that of his legal spouse as well, who may also be lawfully entitled co-ownership
over the said properties. Respondent is not allowed by law to waive whatever share his lawful spouse may have
on the disputed properties. Petitioner herself admitted that she did not assent to the Partition Agreement after
seeing the need to amend the same to include other matters. Petitioner does not have any right to insist on the
contents of an agreement she intentionally refused to sign.

Fernando v. Acuna. GR No. 161030. Sept. 14, 2011


JOSE FERNANDO, JR., ZOILO FERNANDO, NORMA FERNANDO BANARES, ROSARIO FERNANDO
TANGKENCGO, HEIRS OF TOMAS FERNANDO, represented by ALFREDO V. FERNANDO, HEIRS OF
GUILLERMO FERNANDO, represented by Ronnie H. Fernando, HEIRS OF ILUMINADA FERNANDO,
represented by Benjamin Estrella and HEIRS OF GERMOGENA FERNANDO, Petitioners, vs. LEON
ACUNA, HERMOGENES FERNANDO, HEIRS OF SPOUSES ANTONIO FERNANDO AND FELISA
CAMACHO, represented by HERMOGENES FERNANDO, Respondents.

Facts:
At the heart of this controversy is a parcel of land registered in the names of Jose A. Fernando, married to
Lucila Tinio, and Antonia A. Fernando, married to Felipe Galvez, and located in San Jose, Baliuag, Bulacan.
When they died intestate, the property remained undivided. Petitioners herein are the heirs and successors-in-
interest of the deceased registered owners. However, petitioners failed to agree on the division of the subject
property amongst themselves, even after compulsory conciliation before the Barangay Lupon.
Page 129

Thus, petitioners, except for the heirs of Germogena Fernando, filed a Complaint for partition on April 17, 1997
against the heirs of Germogena Fernando. In the Complaint, plaintiffs alleged, among others, that they and
defendants are common descendants and compulsory heirs of the late spouses Jose A. Fernando and Lucila
Tinio, and the late spouses Antonia A. Fernando and Felipe Galvez. They further claimed that their
predecessors-in-interest died intestate and without instructions as to the disposition of the property left by them.
There being no settlement, the heirs are asking for their rightful and lawful share because they wish to build up
their homes or set up their business in the respective portions that will be allotted to them. In sum, they prayed
that the subject property be partitioned into eight equal parts, corresponding to the hereditary interest of each
group of heirs.

However, respondent Leon Acuna intervened in the action averring that in the Decision dated November 29,
1929 of the Cadastral Court of Baliuag, Bulacan, the portion of the property identified as Lot 1303 was already
adjudicated to several other persons who are the petitioners’ predecessor-in-interest. He likewise claimed that in
a 1930 Decision of the Cadastral Court, the portion identified as Lot 1302 was also already adjudicated to other
people as well.

Issue: Whether or not a title registered under the Torrens system, as the subject original certificate of title
is the best evidence of ownership of land and is a notice against the world.

Ruling: No.
As the records show, in the November 29, 1929 Decision of the Cadastral Court of Baliuag, Bulacan had
already been divided and adjudicated to spouses Jose A. Fernando and Lucila Tinio;
spouses Antonia A. Fernando and Felipe Galvez; spouses Antonio A. Fernando and Felisa Camacho; spouses
Jose Martinez and Gregoria Sison; and spouses Ignacio de la Cruz and Salud Wisco from whom respondent
Acuna derived his title.

In the decision, it would appear that petitioners’ ascendants themselves petitioned for the cadastral court to
divide Lot 1303 among the parties to the 1929 case and they were not allocated all the lots. Still, as the trial
court noted, the November 29, 1929 Decision was never fully implemented in the sense that the persons named
therein merely proceeded to occupy the lots assigned to them without having complied with the other directives
of the cadastral court which would have led to the titling of the properties in their names. Nonetheless, it is
undisputed that the persons named in the said November 29, 1929 Decision and, subsequently, their heirs and
assigns have since been in peaceful and uncontested possession of their respective lots for more than seventy
(70) years until the filing of the suit for partition on April 17, 1997 by petitioners which is the subject matter of
this case.
Section 47 of Presidential Decree No. 1529, otherwise known as the Property Registration Decree, states that
“[n]o title to registered land in derogation of the title of the registered owner shall be acquired by prescription or
adverse possession.” Thus, the Court has held that the right to recover possession of registered land is
imprescriptible because possession is a mere consequence of ownership.

However, the Court had recognized the jurisprudential thread regarding the exception to the foregoing doctrine
that while it is true that a Torrens title is indefeasible and imprescriptible, the registered landowner may lose his
right to recover possession of his registered property by reason of laches.

Thus, while a person may not acquire title to the registered property through continuous adverse possession, in
derogation of the title of the original registered owner, the heir of the latter, however, may lose his right to
recover back the possession of such property and the title thereto, by reason of laches.

In view of respondents’ decades long possession and/or ownership of their respective lots by virtue of a court
judgment and the erstwhile registered owners’ inaction and neglect for an unreasonable and unexplained length
Page 130

of time in pursuing the recovery of the land, assuming they retained any right to recover the same, it is clear that
respondents’ possession may no longer be disturbed. The right of the registered owners as well as their
successors-in-interest to recover possession of the property is already a stale demand and, thus, is barred by
laches.
Francisco v. Rojas. GR No. 16712.0. April23, 2014
RODOLFO V. FRANCISCO, Petitioner, v. EMILIANA M. ROJAS, AND THE LEGITIMATE HEIRS OF
JOSE A. ROJAS, NAMELY: JOSE FERDINAND M. ROJAS II, ROLANDO M. ROJAS, JOSE M. ROJAS,
JR., CARMELITA ROJAS–JOSE, VICTOR M. ROJAS, AND LOURDES M. ROJAS, ALL REPRESENTED
BY JOSE FERDINAND M. ROJAS II, Respondents.

Same; Reconveyance; In the action for reconveyance, the decree of registration is highly respected as
incontrovertible; what is sought instead is the transfer of the property wrongfully or erroneously registered in
another’s name to its rightful owner or to the one with a better right.

Facts: [Respondent] Emiliana M. Rojas is the widow of the late Jose Rojas, while the other [respondents] are
the children of the spouses. On the other hand, Rosalina V. Francisco, [petitioner] Rodolfo V. Francisco, and
Carmela V. Francisco, hereafter collectively referred to as the Franciscos, are the applicants for registration in
Land.

Subject of the controversy is a portion of 3,181.74 hectares, vast track of land, known as the Hacienda de
Angono, in Angono, Rizal. The entire hacienda used to be owned by one Don Buenaventura Guido y Santa Ana
upon whose death left a portion thereof, to his two (2) sons Francisco Guido and Hermogenes Guido. Sometime
in September 1911, Decreto No. 6145, covering the same hectare portion of Hacienda de Angono was issued in
favor of the brothers Francisco and Hermogenes. On the basis thereof, Original Certificate of Title over the
same hectares was issued in the names of the two (2) brothers.

Nine (9) years later, or sometime in 1942, the heirs of Francisco and Hermogenes adjudicated among
themselves the same hectares and transferred the one-half (1/2) portion thereof to Jose A. Rojas, predecessor-in-
interest of the [respondents] Rojases. Confusingly, some few months thereafter, the heirs of Guido y Santa Ana
requested the then Land Registration Commission (now, Land Registration Authority) to issue the
corresponding original certificate of title.The request, however, was denied by the said office.

Subsequently, the entire parcel of land covered by Decreto No. 6145 was subdivided into twenty-one (21) lots
and twenty-one (21) different certificates of title were issued in lieu of the reconstituted TCT No. 23377.
Thereafter, the heirs who executed the aforesaid document of extrajudicial settlement, including the now
spouses Jose Rojas and Emiliana Rojas, sold the property to Pacil Management Corporation (Pacil, for short),
and new titles were issued in favor of Pacil on June 26, 1976. Three (3) months later, or on August 26, 1976,
Pacil reconveyed all the 21 lots to the former owners.

“WHEREFORE, the Court hereby declares the Franciscos are the true and absolute owners of Lots 1, 2, 3 and
4. The aforequoted decision having become final and executory, the Franciscos filed with the same court a
petition for the issuance of a decree of registration. The court directed the Commissioner of Land Registration
to issue the desired decree. To complicate matters, it appears in the then Court of First Instance of Rizal, the
Republic of the Philippines, represented by the Solicitor General, filed a complaint for declaration of nullity of
Decreto No. 6145 and the owner’s duplicate copy of TCT No. 23377 against the heirs of Francisco Guido and
Hermogenes Guido, the spouses Jose Rojas and Emiliana Rojas, the Pacil Development Corporation and
Interport Resources Corporation, it being alleged in the same complaint that both the Decreto No. 6145 and the
owner’s copy of TCT No. 23377 were false, spurious and fabricated
Page 131

The Court held that, a decree of registration binds the land and quiets title thereto, is conclusive upon all
persons and cannot be reopened or revived after the lapse of one year after entry of the decree.

CA: Likewise DENIED the appeal

Issue: WHETHER OR NOT THE ASSAILED LAND REGISTRATION PROCEEDINGS IS THE


“APPROPRIATE PROCEEDING” CONTEMPLATED IN THIS HONORABLE COURT’S
PRONOUNCEMENT IN “GUIDO CASE”? IN THE ALTERNATIVE, WHETHER OR NOT AN
ACTION FOR ‘RECONVEYANCE’ BEING MAINTAINED BY THE RESPONDENTS IS THE
“APPROPRIATE PROCEEDING”?

Held: In disputing respondents’ contention that the “appropriate proceeding” should be an action for
reconveyance, petitioner states that such action may be proper but is still not an exclusive remedy. He maintains
that actual fraud in securing a title must be proved so as to succeed in an action for reconveyance, but the Court
already held in Guido that TCT No. 23377 is authentic and genuine; hence, it is assumed that there is no
infirmity or defect therein. Also, an action for reconveyance cannot be availed of like an application for
registration of land as it would be dismissed forthwith on the ground of prescription.

A land registration court has no jurisdiction to order the registration of land already decreed in the name of
another in an earlier land registration case. Issuance of another decree covering the same land is, therefore, null
and void. Truly, one of the appropriate legal remedies that should have been availed of by the Franciscos is an
action for reconveyance. Contrary to petitioner’s declaration, proof of actual fraud is not required as it may be
filed even when no fraud intervened such as when there is mistake in including the land for registration. In the
action for reconveyance, the decree of registration is highly respected as incontrovertible; what is sought instead
is the transfer of the property wrongfully or erroneously registered in another’s name to its rightful owner or to
the one with a better right.

An action for reconveyance resulting from fraud prescribes four years from the discovery of the fraud and if it is
based on an implied or a constructive trust it prescribes ten (10) years from the alleged fraudulent registration or
date of issuance of the certificate of title over the property.

However, an action for reconveyance based on implied or constructive trust is imprescriptible if the plaintiff or
the person enforcing the trust is in possession of the property. In effect, the action for reconveyance is an action
to quiet the property title, which does not prescribe.

In this case, the Franciscos claim to be in open, continuous, exclusive, and notorious possession and occupation
of the subject lots. It appears that they never lost possession of said properties, and as such, they are in a
position to file the complaint with the trial court to protect their alleged rights and clear whatever doubts has
been cast thereon.
Guaranteed Homes v. Valdez. GR No. 1 7153 1. Jan. 30, 2009
GUARANTEED HOMES, INC., Petitioner, vs. HEIRS OF MARIA P. VALDEZ, (EMILIA V. YUMUL and
VICTORIA V. MOLINO), HEIRS OF SEVERINA P. TUGADE (ILUMINADA and LEONORA P. TUGADE,
HEIRS OF ETANG P. GATMIN (LUDIVINA G. DELA CRUZ (by and through ALFONSO G. DELA CRUZ),
HILARIA G. COBERO and ALFREDO G. COBERO) and SIONY G. TEPOL (by and through ELENA T.
RIVAS and ELESIO TEPOL, JR.), AS HEIRS OF DECEDENT PABLO PASCUA, Respondents.

Petition for review on certiorari of CA decision and resolution. Justice Tinga


Page 132

Facts: CA reversed the order of RTC-Olongapo, which granted the motion to dismiss filed by petitioner
Guaranteed Homes. Respondents, descendants of Pablo Pascua, filed a complaint seeking reconveyance of a
parcel of land in Cabitaugan, Subic Zambales and covered by OCT No. 404 in the name of Pablo.They are
praying for damages. From the annexes attached in the complaint, it was ascertained that the OCT contained
several annotations which showed that the property had been sold by Pablo to Alejandria Marquinez and
Resituto Morales.

Also attached in the complaint, and averred by the respondents:


 Extrajudicial Settlement of a Sole Heir and Confirmation of Sales executed by Cipriano Pascua, Sr.,
declaring himself as the only heir of Pablo and confirming the sales made by Pablo to spouses Rodolfo;
 TCT Nos. T-8241 – issued in the name of Cipriano without cancellation of OCT No. 404 & not signed by
Register of Deeds;
 TCT No. T-8242 – issued in the name of spouses Rodolfo, which canceled T-8241;
 TCT No. T-10863 – issued in the name of petitioner when spouses Rodolfo sold the property to petitioner,
which canceled T-8242;
 Deed of Sale with Mortgage between spouses Albino Rodolfo and Fabia Rodolfo and Guaranteed Homes;
Jorge Pascua, Sr., son of Cipriano filed a petition with RTC-Olongapo for the issuance of a new owner’s
duplicate of OCT No. 404, which was denied. The trial court held that petitioner was already the owner of
the subject property, noting that the failure to annotate the subsequent property to it at the back of said
OCT did not affect its title to the property.

Petitioner filed a motion to dismiss the complaint on the following grounds:


 Action is barred by the Statute of Limitations (since more than 28 years have passed since the issuance
from T-10863 to the filing of the complaint);
 No cause of action, since petitioner is an innocent purchaser for value relying on the clean title of spouses
Rodolfo. RTC granted the motion. CA reversed and held that the respondents’ complaint for quieting of
title had not yet prescribed.

Hence, this petition.

Issue: Whether petitioner is an innocent purchaser for value, i.e. there is no need to go beyond the
registered title of spouses Rodolfo.

Held: Yes (there is no need for petitioner to go beyond the clean title presented to them). Petition granted. CA
decision reversed and set aside.

Ratio

The Court held that it is basic that a person dealing with registered property need not go beyond, but only has to
rely on, the title of his predecessor-in-interest. Since “the act of registration is the operative act to convey or
affect the land insofar as third persons are concerned,” it follows that where there is nothing in the certificate of
title to indicate any cloud or vice in the ownership of the property, or any encumbrance thereon, the purchaser is
not required to explore farther than what the Torrens title upon its face indicates in quest for any hidden defect
or inchoate right that may subsequently defeat his right thereto. In the present case, it is enough that petitioner
examined the latest certificate of title issued in the name of spouses Rodolfo. The petitioner as purchaser is not
bound by the original certificate but only by the certificate of title of the person from whom he had purchased
the property.
Page 133

Furthermore, registration in the public registry is considered a notice to the whole world. Every conveyance,
mortgage, lease, lien, attachment, order, judgment, instrument or entry affecting registered land shall be, if
registered, filed or entered in the Office of the Register of Deeds of the province or city where the land to which
it relates lies, be constructive notice to all persons from the time of such registering, filing or entering.
Lonoy v. Secretary of Agrarian Reform. GR No. 175049, Nov. 27. 2008
HEIRS OF SOFIA NANAMAN LONOY, namely, MANUEL N. LONOY, OSCAR N. LONOY, WARREN N. LONOY, EXCELINO
N. LONOY, EDGAR N. LONOY, VICTOR N. LONOY, APOLLO N. LONOY, GEMMA N. LONOY-SAMSON, HEIRS OF RODOLFO N.
LONOY (ISABEL A. LONOY, ISABELITA A. LONOY-YOUNG, WINONA A. LONOY, RODERICK A. LONOY, NANCY A. LONOY-
PAYNAEN, ROBERT LONOY, ROMMEL A. LONOY, RAFAEL A. LONOY, ZENAIDA LONOY-OPADA, HONEYLYN A. LONOY, MARITES
LONOY CABURNAY, and RODOLFO LONOY, JR.), HEIRS OF CORNELIA NANAMAN ADIS/ASEQUIA, namely, HEIRS OF ELSA N. ADIS,
BRICCIO N. ADIS, TOMAS N. ADIS, ROMY N. ADIS, JUSTINO N. ADIS, MERCITA N. ASEQUIA, and TOMASITA N. ASEQUIA, HEIRS OF
VICENTE NANAMAN (LUDEM NANAMAN, ET AL.), HEIRS OF MANUELA NANAMAN AMARGA, namely, HEIRS OF CLARITA
AMARGA-UBGUIA (VERLITO A. UBGUIA, DANILO A. UBGUIA, ASTERIO A. UBGUIA, and CARLO A. UBGUIA), HEIRS OF ACOLON
AMARGA (ALMIRANTE AMARGA, SPARTACUS AMARGA, MELVIN AMARGA, and RODRIGO AMARGA), ALONSO N. AMARGA,
HERDA N. AMARGA, DELOS MIMBA AMARGA-TOGONON, HEIRS OF ASCONA AMARGA UBAGAN (DEMOSTHENES A. UBAGAN,
ET AL.), HEIRS OF NICODEMO N. AMARGA (JIMMY AMARGA, MARIETTA AMARGA, BENIGNO AMARGA, NICODEMO AMARGA,
JR., ALMA AMARGA, FELIX AMARGA, ADOR AMARGA, LYDIA AMARGA, JUDY AMARGA, LOLOT AMARGA, and MADONNA
AMARGA), HEIRS OF ATANACIO NANAMAN AMARGA (GLORIOSA A. APOR, NESTOR AMARGA, NORVILLA AMARGA, GENITA
AMARGA, and GILMA AMARGA), HEIRS OF OLIVA AMARGA-BADELLES (JOSE I. BADELLES, JIMBO BADELLES, JOHNSON
BADELLES, ALITA BADELLES-JALAGAT, NINIAN BADELLES, JONA A. BADELLES, CEFERINO A. BADELLES, OLIVER BADELLES,
OHARA A. BADELLES, MARIA BADELLES, SARAH A. BADELLES, JEBA A. BADELLES, and MICHAELA A. BADELLES), and HEIRS OF
MANSUETO N. AMARGA (EDNA AMARGA – surviving spouse of JESSE AMARGA, DEÑA AMARGA-MAGHINAY, and MARLON
AMARGA), HEIRS OF GENARA NANAMAN SAKALL, namely, AMPARO SAKALL-DURANO, BENEDICTO N. SAKALL, ISABELITA N.
SAKALL, FRANCISCA SAKALL MARQUINA, HONORIO N. SAKALL, VIRGINIA SAKALL ESTANISLAO, and NORMA N. SAKALL,
HEIRS OF JULIETA NANAMAN, namely, HEIRS OF JAIME NANAMAN/RIVERA (ANASTASIA LAUGAM NANAMAN – surviving spouse,
DULSORA NANAMAN, and GUILLERMO NANAMAN), HEIRS OF PIO NANAMAN/ROA (WILMA NANAMAN, ALFREDO NANAMAN,
DELIA NANAMAN, SALVADOR NANAMAN, HEIRS OF RAUL NANAMAN, EVELYN NANAMAN, VIOLA NANAMAN, EDITHA
NANAMAN, PINKY NANAMAN, and ALEXANDER NANAMAN), HEIRS OF GREGORIO NANAMAN/DACAMPO (VICTOR NANAMAN,
VICENTE NANAMAN, GREGORIO NANAMAN, JR., and VIRGIE NANAMAN), and HEIRS OF ORLANDO NANAMAN (EMILIA G.
NANAMAN – surviving spouse, ALEX NANAMAN, EMMA NANAMAN, HEIRS OF GEORGINA NANAMAN, GEORGE NANAMAN,
RAMIL NANAMAN, and CAROLYN NANAMAN), HEIRS OF ROSARIO NANAMAN RUEDAS, namely, HEIRS OF BERNARDO N.
RUEDAS (JULIA RUEDAS, JONATHAN RUEDAS, MARLON RUEDAS, MARIVIC RUEDAS, EDITHA RUEDAS, and MARGIE RUEDAS-
POGOY), and HEIRS OF JOSE "FEBE" NANAMAN (SOCORRO NANAMAN, AIDA NANAMAN, LERMA NANAMAN-MORALES,
EDUARDO NANAMAN, JOSEFA NANAMAN, MARISA NANAMAN, ARTURO NANAMAN, and MARYFLOR NANAMAN), and ATTY.
ELPEDIO CABASAN as Administrator of the Intestate Estate of Gregorio Nanaman, petitioners, vs. CITY OF ILIGAN, HEIRS OF
JUAN NANAMAN, HEIRS OF LIMBANIA CABILI MERCADO, HEIRS OF MARIANO ANDRES CABILI,
respondents/unwilling co-petitioners.

Facts:
• Action for Reversion of Title
Spouses Gregorio Nanaman (Gregorio) and HilariaTabuclin (Hilaria) were the owners of a parcel of agricultural
land situated in Tambo, Iligan City, upon which they likewise erected their residence. Living with them on the
subject property were Virgilio Nanaman (Virgilio),Gregorios son by another woman.

When Gregorio died, Hilaria administered the subject property with Virgilio.Hilaria and Virgilio executed a
Deed of Saleover the subject property in favor of Jose C. Deleste (Deleste).

Upon Hilarias death, Juan Nanaman (Juan), Gregorios brother, was appointed as special administrator of the
estate of the deceased spouses Gregorio and Hilaria (joint estate).Edilberto Noel (Noel) was appointed as the
regular administrator of the joint estate.

The subject property was included in the list of assets of the joint estate. However, Noel could not take
possession of the subject property since it was already in Delestes possession. Thus, Noel filed before the Court
of First Instance (CFI),an action against Deleste for the reversion of title over the subject property to the Estate,
docketed as Civil Case No. 698.
Page 134

Through the years, Civil Case No. 698 was heard, decided, and appealed all the way to the Supreme Court in
Noel v. Court of Appeals. The Court rendered its Decision in Noel, affirming the ruling of the Court of Appeals
that the subject property was the conjugal property of the late spouses Gregorio and Hilaria, such that the latter
could only sell her one-half (1/2) share therein to Deleste. Consequently, the intestate estate of Gregorio and
Deleste were held to be the co-owners of the subject property, each with a one-half (1/2) interest in the same.

• Petition for Nullification of the Emancipation Patents (Heirs of Deleste)


Deleste passed away sometime in 1992.The Heirs of Deleste, filed with the Department of Agrarian Reform
Adjudication Board (DARAB) a petition seeking to nullify private respondents Emancipation Patents (EPs).

The Provincial Agrarian Reform Adjudicator (PARAD) rendered a Decisiondeclaring that the EPs were null and
void in view of the pending issues of ownership and the subsequent reclassification of the subject property into
a residential/commercial land.

On appeal, the DARAB reversed the ruling of the PARAD in its Decision. The DARAB held, that the EPs were
valid, since it was the Heirs of Deleste who should have informed the DAR of the pendency of Civil Case No.
698at the time the subject property was placed under the coverage of the Operation Land Transfer Program. The
Heirs of Deleste filed a Motion for Reconsideration of the aforementioned Decision, but the Motion was denied
by the DARAB in its Resolution dated 8 July 2004.

• Petition for Prohibition


A Petition for Prohibition, Declaration of Nullity of Emancipation Patents Issued by DAR and the
Corresponding [Original Certificates of Title] Issued by the [Land Registration Authority], Injunction with
Prayer for Temporary Restraining Order (TRO) was filed by petitioners with the Court of Appeals. Arguing that
they were deprived of their inheritance by virtue of the improper issuance of the EPs to private respondents
without notice to them, petitioners prayed that a TRO be forthwith issued, prohibiting the DAR Secretary, the
Land Registration Authority (LRA), the DARAB, the Land Bank of the Philippines (LBP), as well as the RTC,
from enforcing the EPs and OCTs in the names of private respondents . Petitioners further prayed that judgment
be subsequently rendered declaring the said EPs and the OCTs null and void.

The Court of Appeals dismissed the Petition.

Issues:
1. WHETHER OR NOT THE COURT OF APPEALS COMMITTED GRAVE ABUSE OF DISCRETION
AMOUNTING TO LACK OR EXCESS OF JURISDICTION IN HASTILY DISMISSING THE
PETITIONERS PETITION FOR PROHIBITION.
2. WHETHER OR NOT RESPONDENTS SECRETARY OF AGRARIAN REFORM, LRA, AND DARAB
ACTED WITHOUT OR IN EXCESS OF JURISDICTION IN REVIEWING [AND] OVERRULING
JUDICIAL DECISIONS CONSIDERING THAT THE POWER OF JUDICIAL REVIEW OVER ACTS OF
THE EXECUTIVE OR LEGISLATIVE BRANCH BELONGS TO THE JUDICIARY AND NOT VICE
VERSA.

Ruling:

Prohibition is a legal remedy, provided by the common law, extraordinary in the sense that it is ordinarily
available only when the usual and ordinary proceedings at law or in equity are inadequate to afford redress,
prerogative in character to the extent that it is not always demandable of right, to prevent courts, or other
tribunals, officers, or persons, from usurping or exercising a jurisdiction with which they have not been vested
by law.
Page 135

The writ of prohibition, is one which commands the person to whom it is directed not to do something which,
by suggestion of the relator, the court is informed he is about to do. The only effect of a writ of prohibition is to
suspend all action and to prevent any further proceeding in the prohibited direction.

In this case, the Petition for Prohibition filed by the petitioners reveal that the same is essentially more of an
action for the nullification of the allegedly invalid EPs and OCTs issued in the names of private respondents.
The writ of prohibition is only sought by petitioners to prevent the implementation of the EPs and OCTs. Such
EPs and OCTs had become indefeasible and incontrovertible by the time petitioners instituted their Petition for
Prohibition, and may no longer be judicially reviewed.

Private respondents EPs were issued in their favor and their OCTs were correspondingly issued and
subsequently registered with the Register of Deeds of Iligan City. Petitioners directly went to the Court of
Appeals, instead to the Regional Trial Court almost four (4) years after the issuance and registration thereof.
Petitioners failed to vindicate their rights within the one-year period from issuance of the certificates of title as
the law requires.

After the expiration of the one-year period, a person whose property has been wrongly or erroneously registered
in another’s name may bring an ordinary action for reconveyance, or if the property has passed into the hands of
an innocent purchaser for value, Section 32 of the Property Registration Decree gives petitioners only one other
remedy, i.e., to file an action for damages against those responsible for the fraudulent registration.

WHEREFORE, premises considered, the instant Petition for Review is hereby DENIED. No costs.
Cusi v. Domingo. GR No. 195825. Feb.27, 2013
SPOUSES ALFONSO AND MARIA ANGELES CUSI, Petitioners, vs. LILIA V. DOMINGO, Respondent.

Facts: The property in dispute was a vacant unfenced lot situated in White Plains, Quezon City and covered by
Transfer Certificate of Title (TCT) No. N-165606 issued in the name of respondent Lilia V. Domingo by the
Registry of Deeds of Quezon City. It had an area of 658 square meters. In July 1999, Domingo learned that
construction activities were being undertaken on her property without her consent. She soon unearthed the
series of anomalous transactions affecting her property. On July 18, 1997, one Radelia Sy (Sy), representing
herself as the owner of the property, petitioned the RTC for the issuance of a new owner’s copy of Domingo’s
TCT No. N-165606, appending to her petition a deed of absolute sale dated July 14, 1997 purportedly executed
in her favor by Domingo; and an affidavit of loss dated July 17, 1997, whereby she claimed that her bag
containing the owner’s copy of TCT No. N-165606 had been snatched from her on July 13, 1997 while she was
at the SM City in North EDSA, Quezon City. The RTC granted Sy’s petition on August 26, 1997. The Registry
of Deeds of Quezon City then issued a new owner’s duplicate copy of TCT No. N-165606, which was later
cancelled by virtue of the deed of absolute sale dated July 14, 1997, and in its stead the Registry of Deeds of
Quezon City issued TCT No. 186142 in Sy’s name. Sy subsequently subdivided the property into two, and sold
each half by way of contract to sell to Spouses Edgardo and Ramona Liza De Vera and to Spouses Alfonso and
Maria Angeles Cusi. The existence of the individual contracts to sell was annotated on the dorsal portion of Sy’s
TCT No. 186142 as Entry No. PE8907/N-186142, stating that the consideration of the sale was P1,000,000.00
for each set of buyers, or for a total of P2,000,000.00 for the entire property that had an actual worth of not less
than P14,000,000.00. TCT No. 186142 in the name of Sy was then cancelled by virtue of the deeds of sale
executed between Sy and Spouses De Vera, and between Sy and Spouses Cusi, to whom were respectively
issued TCT No. 189568 and TCT No. 189569. All the while, the transactions between Sy and the De Vera’s, and
between Sy and the Cusi’s were unknown to Domingo, whose TCT No. N-165606 remained in her undisturbed
possession. It turned out that the construction activities taking place on the property that Domingo learned about
were upon the initiative of the De Veras in the exercise of their dominical and possessory rights. Domingo
Page 136

commenced this action against Sy and her spouse, the De Vera’s and the Cusi’s in the RTC, the complaint being
docketed as Civil Case No. Q-99-39312 and entitled Lilia V. Domingo v. Spouses Radelia and Alfred Sy,
Spouses Alfonso G. and Maria Angeles S. Cusi, Spouses Edgardo M. and Ramona Liza L. De Vera, BPI Family
Savings Bank and The Register of Deeds of Quezon City, seeking the annulment or cancellation of titles,
injunction and damages.

Issue: Whether or not the petitioners are considered purchasers in good faith and for value thus protected for
their rights over the subject land.

Held: No. One of the guiding tenets underlying the Torrens system is the curtain principle, in that one does not
need to go behind the certificate of title because it contains all the information about the title of its holder. This
principle dispenses with the need of proving ownership by long complicated documents kept by the registered
owner, which may be necessary under a private conveyancing system, and assures that all the necessary
information regarding ownership is on the certificate of title. Consequently, the avowed objective of the Torrens
system is to obviate possible conflicts of title by giving the public the right to rely upon the face of the Torrens
certificate and, as a rule, to dispense with the necessity of inquiring further; on the part of the registered owner,
the system gives him complete peace of mind that he would be secured in his ownership as long as he has not
voluntarily disposed of any right over the covered land.

The Philippines adopted the Torrens system through Act No. 496, also known as the Land Registration Act,
which was approved on November 6, 1902 and took effect on February 1, 1903. In this jurisdiction, therefore,
“a person dealing in registered land has the right to rely on the Torrens certificate of title and to dispense with
the need of inquiring further, except when the party has actual knowledge of facts and circumstances that would
impel a reasonably cautious man to make such inquiry”.

The records also show that the forged deed of sale from Domingo to Sy appeared to be executed on July 14,
1997; that the affidavit of loss by which Sy would later on support her petition for the issuance of the duplicate
owner’s copy of Domingo’s TCT No. 165606 was executed on July 17, 1997, the very same day in which Sy
registered the affidavit of loss in the Registry of Deeds of Quezon City; that Sy filed the petition for the
issuance of the duplicate owner’s copy of Domingo’s TCT No. 165606; that the RTC granted her petition on
August 26, 1997; and that on October 31, 1997, a real estate mortgage was executed in favor of one Emma
Turingan, with the mortgage being annotated on TCT No. 165606 on November 10, 1997.

Being the buyers of the registered realty, the Cusi’s and the De Vera’s were aware of the aforementioned several
almost simultaneous transactions affecting the property. Their awareness, if it was not actual, was at least
presumed, and ought to have put them on their guard, for, as the CA pointed out, the RTC observed that “[t]hese
almost simultaneous transactions, particularly the date of the alleged loss of the TCT No. 165606 and the
purported Deed of Sale, suffice[d] to arouse suspicion on [the part of] any person dealing with the subject
property.” Simple prudence would then have impelled them as honest persons to make deeper inquiries to clear
the suspiciousness haunting Sy’s title. But they still went on with their respective purchase of the property
without making the deeper inquiries. In that regard, they were not acting in good faith.

Another circumstance indicating that the Cusis and the De Veras were not innocent purchasers for value was the
gross undervaluation of the property in the deeds of sale at the measly price of P1,000,000.00 for each half
when the true market value was then in the aggregate of at least P14,000,000.00 for the entire property. Even if
the undervaluation was to accommodate the request of Sy to enable her to minimize her liabilities for the capital
gains tax, their acquiescence to the fraud perpetrated against the Government, no less, still rendered them as
parties to the wrongdoing. They were not any less guilty at all. In the ultimate analysis, their supposed passivity
respecting the arrangement to perpetrate the fraud was not even plausible, because they knew as the buyers that
they were not personally liable for the capital gains taxes and thus had nothing to gain by their acquiescence.
Page 137

There was simply no acceptable reason for them to have acquiesced to the fraud, or for them not to have
rightfully insisted on the declaration of the full value of the realty in their deeds of sale. By letting their
respective deeds of sale reflect the grossly inadequate price, they should suffer the consequences, including the
inference of their bad faith in transacting the sales in their favor.

Alde v. Bernal, GR No. 169336, March 18, 2010


SPOUSES MELCHOR and SATURNINA ALDE, Petitioners, vs. RONALD B. BERNAL, OLYMPIA B.
BERNAL, JUANITO B. BERNAL, and MYRNA D. BERNAL, Respondents.

Facts: WON Mario has a better right over the subject land.
In 1957, Adriano Bernal, father of Ronald, Olympia, Juanito and Myrna had in his possession a property which
was later surveyed and designated as Cadastral Lot No. 1123, Cad 1119-D, Case 8 in 1992. In 1994, Adriano
secured a loan from Melchor and Saturnina Alde and turned over physical possession, occupation and
cultivation of 3 hectares of the property to Spouses Alde. Then Adriano had later sold the property to Spouses
Aldo.

On 18 October 1994, OCT No. AO-7236 was issued in the names of the Bernals. OCT No. AO-7236 originated
from CLOA No. 00073938 issued by the Department of Agrarian Reform pursuant to Republic Act No. 6657.
Then, in April 2002, respondents demanded from petitioners P50,000 as additional consideration for the
property. Respondents also informed petitioners, for the first time, of the existence of OCT No. AO-7236.
Petitioners rejected respondents’ request since they already bought the entire property in 1994 and requested
that respondents should turn-over to them OCT No. AO-7236.

Issue: WON the respondents can claim ownership over the disputed portions of the property.

Ruling
The respondents claim ownership of the property based on OCT No. AO-7236. However, a certificate of title is
not equivalent to title. In Lee Tek Sheng v. Court of Appeals: By title, the law refers to ownership which is
represented by that document [the Original Certificate of Title or the Transfer Certificate of Title]. Petitioner
apparently confuses certificate with title. Placing a parcel of land under the mantle of the Torrens system does
not mean that ownership thereof can no longer be disputed. Ownership is different from a certificate of title.
The TCT is only the best proof of ownership of a piece of land. Besides, the certificate cannot always be
considered as conclusive evidence of ownership. Mere issuance of the certificate of title in the name of any
person does not foreclose the possibility that the real property may be under co-ownership with persons not
named in the certificate or that the registrant may only be a trustee or that other parties may have acquired
interest subsequent to the issuance of the certificate of title. To repeat, registration is not the equivalent of title,
but is only the best evidence thereof. Title as a concept of ownership should not be confused with the certificate
of title as evidence of such ownership although both are interchangeable.

In this case, respondents cannot claim ownership over the disputed portions of the property absent any showing
of how they acquired title over the same. Accordingly, the property must be reconveyed in favor of petitioners.
An action for reconveyance is a legal and equitable remedy granted to the rightful owner of land which has been
wrongfully or erroneously registered in the name of another for the purpose of compelling the latter to transfer
or reconvey the land to him. However, since petitioners did not make a direct attack on the validity of OCT No.
AO-7236 and had not asked for the cancellation of the original certificate of title as required by Section 48 of
Presidential Decree No. 1529, this Court cannot cancel OCT No. AO-7236 and order the issuance of a new
certificate of title in the name of petitioners. Any direct attack on the validity of a Torrens certificate of title
must be instituted with the proper Regional Trial Court. This case originated in the Municipal Circuit Trial
Court. Even if the Court will consider petitioners’ counter-claim as a petition for the cancellation of OCT No.
Page 138

AO-7236 and, thus, a direct attack on the certificate of title, the MCTC still does not have jurisdiction over the
cancellation of a Torrens title.
Peralta v. Abalon. GR No. 183448, June 30, 2014
SPOUSES DOMINADOR PERALTA AND OFELIA PERALTA, Petitioners, vs. HEIRS OF BERNARDINA
ABALON, represented by MANSUETO ABALON, Respondents.

Facts:
The instant case before the RTC Legazpi City involved a parcel of land described as Lot 1679 consisting of
8,571 square meters covered by OCT No. (O) 16 and registered under the name of Bernardina Abalon. It was
fraudulently transferred to Restituto Rellama by executing a Deed of Sale and who, in turn, subdivided the
subject property and sold it separately to the other parties - Spouses Dominador and Ofelia Peralta (TCT No.
42252); and Marissa, Leonil and Arnel, all surnamed Andal (TCT No. 42482 and TCT No. 42821). Thereafter,
Spouses Peralta and the Andals individually registered the respective portions of the land they had bought under
their names. The heirs of Bernardina were claiming back the land, alleging that since it was sold under
fraudulent circumstances, no valid title passed to the buyers. On the other hand, the buyers, who were now title
holders of the subject parcel of land, averred that they were buyers in good faith and sought the protection
accorded to them under the law.

Issue: Whether a forged instrument may become the root of a valid title in the hands of an innocent
purchaser for value, even if the true owner thereof has been in possession of the genuine title, which is
valid and has not been cancelled.

Held: Yes. The established rule is that a forged deed is generally null and cannot convey title, the exception
thereto, pursuant to Section 55 of the Land Registration Act, denotes the registration of titles from the forger to
the innocent purchaser for value. Thus, the qualifying point here is that there must be a complete chain of
registered titles. This means that all the transfers starting from the original rightful owner to the innocent holder
for value – and that includes the transfer to the forger – must be duly registered, and the title must be properly
issued to the transferee. Contrary to what the Abalons would like to impress on us, Fule and Torres do not
present clashing views. In Fule, the original owner relinquished physical possession of her title and thus enabled
the perpetrator to commit the fraud, which resulted in the cancellation of her title and the issuance of a new one.
The forged instrument eventually became the root of a valid title in the hands of an innocent purchaser for
value. The new title under the name of the forger was registered and relied upon by the innocent purchaser for
value. Hence, it was clear that there was a complete chain of registered titles.

In the instant case, there is no evidence that the chain of registered titles was broken in the case of the Andals.
Neither were they proven to have knowledge of anything that would make them suspicious of the nature of
Rellama’s ownership over the subject parcel of land. Hence, we sustain the CA’s ruling that the Andals were
buyers in good faith. Consequently, the validity of their title to the parcel of the land bought from Rellama must
be upheld.
De la Cruz v. De laCruz. GRNo. L-61969,Ju1y 28, 1984
AGUSTINA DE LA CRUZ, ET AL., petitioners, vs. LUCIA DE LA CRUZ, IGLESIA NI KRISTO (Church of
Christ) and HONORABLE COURT OF APPEALS, respondents.
Prospero A. Crescini for petitioners.

Domingo v. Reed, GR No. 157701, Dec. 9, 2005


Spouses DANILO and G.R. No. 157701
Page 139

ALBERTA DOMINGO, and EDUARDO QUITEVES, Present: Petitioners, Panganiban, J., Chairman,
Sandoval-Gutierrez, - versus - Carpio Morales, Corona, and Garcia, JJ, GUILLERMO REED, Promulgated:
Respondent.
Ponce de Leon v. RFC, GR No. L-245718, Dec. 18, 1970
JOSE L. PONCE DE LEON, plaintiff-appellant, vs. REHABILITATION FINANCE CORPORATION,
defendant-appellant and third-party defendant-appellant, ROSALINA SORIANO, TEOFILA SORIANO and
REV. FR. EUGENIO R. SORIANO, third-party plaintiffs-appellants.

Facts:
1. On August 14, 1945, plaintiff Jose L. Ponce de Leonand Francisco Soriano, father of third-
party plaintiffsobtained a loan for P10k from the PNB Manila,mortgaging a parcel of land
situated at Barrio Ibayo,Municipality of Parañaque, Rizal, covered by OCT No.8094 of the land
records of Rizal Province in the nameof “Francisco Soriano, married to Tomasa Rodriguez”,as security for
the loan.
2. Jose L. Ponce de Leon filed with the RehabilitationFinance Corporation (RFC for short)
Manila, his loanapplication for an industrial loan, for putting up asawmill, in the amount
of P800k offering as security certain parcels of land, among which, was the parcelwhich
Ponce de Leon and Soriano mortgaged to thePNB. Jose Ponce De Leon, his wife
Carmela and,Francisco Soriano(father of third party plaintiff) filed fora loan from the RFC for
P495k. A deed of mortgagewas then executed in view of the loan, secured by aparcel of land
owned by Soriano. At the time thatFrancisco Soriano signed the mortgage deed hisspouse Tomasa
Rodriguez was already deadleaving as her heirs, her children none of whomsigned the said mortgage
deed or the promissorynote.
3. It was stipulated that part of the proceeds of themortgage loan shall be used to pay off
obligations. Inview of these conditions, the RFC paid Ponce deLeon's obligations to PNB, Cu
Unjieng Bros and ArturoColmenares. Various amounts were released to Poncede Leon from December
1951 to July 1952. Thechecks covering these releases were issued to Jose L.Ponce de Leon in view of
the authority given to him inwriting by Francisco Soriano and Carmelina Russell.
4. Allegedly, the loan was not paid. Because of this, RFCsought for extra-judicial foreclosure of the
mortgagedproperties (real estate+sawmill in Samar andequipment). RFC was the
purchaser of all themortgaged properties in the ensuing sheriff's sales,with the exception of
two parcels of land situated inBacolod City which were purchased by privateindividuals.
5. The Sheriff sold the land covered by OCT No. 8094 in the name of Francisco Soriano, married
toTomasa Rodriguez, and the deed of sale was executed by the sheriff in favor of the purchaser
the RFC, including all the other properties sold
6. Prior to the expiration of the one year periodredemption period. Francisco Soriano (through
hisheirs) offered to repurchase the Paranaque lot for 14kbut the bank(RFC) rejected the offer. RFC
scheduledthe public sale of the lot.
7. In 1956, Ponce filed the present action questioning thevalidity of the sherriff’s foreclose sale, and
requiring awrit of preliminary injunction to restrain RFC fromcarrying out its scheduled
sale.8.The Sorianos filed a 3rd party complaint contending thatthe mortgage in favor of the RFC and
promissory notesigned by Francisco Soriano lacked the latter's consentand was w/o consideration and
hence void as to himand his children; that the lot covered by OCT No. 8094in the name of Francisco
Soriano belonged to theconjugal partnership of the latter and his wife, TomasaRodriguez, now
deceased, and since the latter wasalready dead when the mortgage was executed andher
children who inherited her share have not signedthe mortgage contract and promissory note, at
least,the ½ share of the lot belonging now to the Sorianosisters and brothers, the third-party
plaintiffs, have notbeen legally included in the mortgage to the RFC sothe latter had not acquired
said one-half share in thesheriff's sale.9.TC dismissed Ponce’s complaint and held that
Page 140

beingregistered in the name of "Francisco Soriano, marriedto Tomasa Rodriguez," the property
covered by OCTNo. 8094(the Parañaque property) is presumed belongto the conjugal partnership
of said spouses, and the RFC having failed to offset this presumption, themortgage on and
the sale of the property by the sheriffare null and void as to one-half (½) thereof.

Issue: Whether the TC erred erred in holding that the Parañaqueproperty is presumed to belong to the
conjugal partnership of Mr. and Mrs. Francisco Soriano? YES

Held:
1. It appears that the property was registered in the nameof "Francisco Soriano, married to Tomasa
Rodriguez,"and based upon this fact alone without any proofestablishing satisfactorily that
the property had beenacquired during coverture, the lower court presumedthat it belongs
to the conjugal partnership of saidspouses.
2. We should not overlook the fact that the title to saidproperty was not a transfer certificate of title, butan
original one, issued in accordance with a decree which, pursuant to law, merely confirms a pre-
existing title. Said original certificate of title does not establish, therefore, the time of acquisition of the
Parañaque property by the registered owner thereof
3. Lower Court applied said presumption in Article 160 of our Civil Code, which reads:... All property of
the marriage is presumed to belong to the conjugal partnership, unless it be proved that it pertains
exclusively to the husband or to the wife. This provision must be construed in relation to Articles
153 to 159 of the same Code, enumerating the properties "acquired ... during the
marriage" that constitute the conjugal partnership. We have held that "the party who invokes this
presumption must first prove that the property in controversy was acquired during the marriage. In
other words, proof of acquisition during coverture is a condition sinequa non for the operation of the
presumption in favor of conjugal partnership.
4. The Sorianos have not succeeded in proving that theParañaque property was acquired "during
themarriage" of their parents. What is more, there issubstantial evidence to the contrary.
5. Gregorio Soriano testified that his first cousin,Francisco Soriano, had acquired said
propertyfromhis parents, long before he got married. Saidprosecution does not necessarily
warrant theconclusion that Gregorio Soriano was impelled by an"improper motive" in testifying
as he did. After all, theSorianos are, likewise, nieces of Gregorio Soriano andhe was not the party
allegedly accused by them.
6. This witness testified in a straightforward manner, anddisclosed a good number of details bearing
the ear-marks of veracity. His testimony was corroborated, notonly by Felipe Cuaderno, Jr. and OCT
No. 8094, but,also, by the testimony of 3rd plaintiff Rosalina Soriano.Felipe Cuaderno, Jr., an assistant
attorney and notarypublic of the RFC, before whom the deed of mortgagewas acknowledged, testified
that Francisco Sorianoassured him that the Parañaque property was "his ownseparate property,
having acquired it from his deceased father by inheritance and that his childrenhave nothing
to do with the property." This was, ineffect, confirmed by Rosalina Soriano, stated on cross-
examination, that her father "was born and ... raised" insaid property, so that contrary to her testimony in
chiefhe could not have told her that he and his wife hadbought it.
7. Needless to say, had the property been acquired bythem during coverture, it would have been
registered,in the name not of "Francisco Soriano, married toTomasa Rodriguez," but of the
spouses "FranciscoSoriano and Tomasa Rodriguez." In Litam vs. Espiritu,the words 'married to
Rafael Litam' written after thename of Marcosa Rivera, in each of the abovementioned
titles are merely descriptive of the civilstatus of Marcosa Rivera, the registered owner of
theproperties covered by said titles.
8. It is difficult to believe also that Sorianos did not knowthen of the mortgage constituted by Francisco
Soriano,on October 8, 1951, in favor of the RFC. In fact,Rosalina Soriano testified that
month, FranciscoSoriano and she conferred with the plaintiff, he statedthat the Parañaque property
Page 141

was mortgaged to theRFC, her father got angry at the plaintiff and said thathe had fooled him
(Francisco Soriano). Being aware ofsaid mortgage since October 1951, the Sorianos didnot
question its validity until January 12, 1957, whenthey filed in this cage their 3rd -party
complaint inintervention as regards, at least, 1/2 of the Parañaqueproperty, which they now claim
to be their mother'sshare in the conjugal partnership. Worse still, after theforeclosure sale in favor
of the RFC, they tried to edeem the property for P14,000, and, when the RFCdid not agree thereto,
they even sought the help of theOffice of the President to effect said redemption.
9. Their failure to contest the legality of the mortgage forover five (5) years and these attempts to redeem
the property constitute further indicia that the same belonged exclusively to Francisco Soriano,
not to the conjugal partnership with his deceased wife, Tomasa Rodriguez. Apart from the
fact that said attempts toredeem the property constitute an implied admission ofthe validity of its
sale and, hence, of its mortgage to the RFC, there are authorities to the effect that they bar the Sorianos
from assailing the same.
Litam v. Espirity, GR No. L-7644, Nov. 27, 1956
HENRY LITAM, ETC., ET AL., Plaintiffs-Appellants, vs. REMEDIOS R. ESPIRITU, as guardian of the
incompetent MARCOSA RIVERA, and ARMINIO RIVERA, Defendants-Appellees

PNB v. CA and Vitug, GR No. 57757, Aug. 31, 1987


PHILIPPINE NATIONAL BANK, petitioner, vs. THE HONORABLE COURT OF APPEALS, PRAGMACIO
VITUG AND MAXIMO VITUG, respondents.

Facts: Donata Montemayor is the administrator of 30 parcels of land of her late husband Clodualdo Vitug who
died intestate. Several portions of such land (TCT-2289, TCT-2887, and TCT-2888) were mortgaged to PNB as
security for certain loans availed by Salvador Vitug, Salvador Jaramilla and Pedro Bacani, respectively. The
loans were never paid so the bank foreclosed all the mortgaged properties. PNB as the highest bidder, purchased
the lots, and subsequently sold the same to the Vitugs and the Fajardos.

Meanwhile, Donata executed a contract of lease for a lot covered by TCT-2887- R to her sons Pragmacio and
Maximo Vitug. After a few years, the same brothers filed an action for partition and reconveyance with damages
in the CFI Pampanga against the PNB, the Vitugs, the Fajardos, and Marcelo Mendiola, the special
administrator of Donata’s intestate estate. They claimed that the 30 parcels of land form part of the conjugal
property of the spouses Donata and Clodualdo and they claim a share interest of 2/11 of 1/2 thereof. They
assailed the mortgage of said properties to the PNB and the subsequent public auction. They invoked the Vitug
vs. Montemayor case where the SC ruled on the conjugal nature of the 30 parcels of land.
Issues: (1) Can the PNB rely merely on the Torrens Certificate of Title covering Donata’s properties for
the processing of the respective mortgage loan applications?
(2) Is the earlier action for reconveyance and partition concerning the 30 lots real actions and binding
upon the PNB by virtue of the Vitug vs. Montemayor decision?

Ruling:
1. Yes. The PNB had sufficient reason to rely on the Torrens Certificate of Title of the mortgaged
properties. The SC ruled that in processing the loan applications, the PNB had the right to rely upon the
face of the certificate of title. Clearly, it appears that Donata (a widow) owns the properties and the PNB
had no reason to doubt her status and ownership. The PNB also found no liens or encumbrances
covering the properties. The clean facts reasonably cancel the need to make further inquiry. The Court
applied the well-known rule in jurisdiction that a person has a right to rely upon the face of the Torrens
Certificate of Title when dealing with a registered land. It is not necessary to inquire beyond its face,
except when such person has an actual knowledge of facts and circumstances that would prompt him to
Page 142

inquire further. The Court ruled that a Torrens title “concludes all controversy over ownership of the
land covered by a final degree of registration” and upon such registration, the person is assured of
ownership without going to court or sitting “at the veranda of his house” to avoid the fear of losing his
land.
2. No. The SC maintained that although actions for recovery of real property and for partition are real
actions, they are actions in personam which are binding only upon the persons who are parties thereto.
The PNB is not a party in the cited case and is therefore not bound by the decision. In addition, there is
no showing that the PNB was aware of the case decision when it extended the mortgage loan involving
the subject properties. The court settled that if the PNB knew that said properties were conjugal, it would
not have approved the mortgage application without securing the consent of the its co-owners.
Therefore, the PNB is considered as a purchaser for value in good faith when it sold the foreclosed
properties at a public auction.

The complaint is dismissed.


Republic v.Mendoza, GR No. 185091, Aug. 8, 2010
REPUBLIC OF THE PHILIPPINES, REPRESENTED BY THE DEPARTMENT OF EDUCATION DIVISION
OF LIPA CITY (FOR PANINSINGIN PRIMARY SCHOOL), Petitioner, v. PRIMO MENDOZA and MARIA
LUCERO, Respondents.

Barcelonav. Barcelona, GRNo. L-9014, Oc.31,1956 .

Buhat v. Besana, GR No. L-6746, Aug. 31,1954 .

De Pedro v. Romanasan, GR No. 158002, Feb. 28, 2005 .

Castores v. Afidchao, GR No. 151240,March31,2009 ,

Manotok v. Barque, GR No.162335, Dec. 18, 2008 .

Republic v. Samson-Tatad, GRNo. l\767j,April 5,2013 .

Bangis v. Adolfb, GR No. 190875, June 73,2012 .

Carvajal v. Court of Appeals, GR No.98328, Oct. 9,1997 .

Alfonso v. Oflice of the president, GR No. 150091, April 2, 2007 .

Republic v. Abrille, GR No. L-39248, May 7,1976

Potrebbero piacerti anche